Top Banner
1 © Pearson Education Limited 2006 Instructor’s Manual Financial Accounting & Reporting Tenth edition Barry Elliott and Jamie Elliott For further instructor material please visit: www.pearsoned.co.uk/elliott ISBN 0 273 70365 X Pearson Education Limited 2006 Lecturers adopting the main text are permitted to download the manual as required.
337
Welcome message from author
This document is posted to help you gain knowledge. Please leave a comment to let me know what you think about it! Share it to your friends and learn new things together.
Transcript
Page 1: elliottv10

1 © Pearson Education Limited 2006

Instructor’s Manual

Financial Accounting &

Reporting

Tenth edition

Barry Elliott and

Jamie Elliott

For further instructor material please visit:

www.pearsoned.co.uk/elliott

ISBN 0 273 70365 X

Pearson Education Limited 2006 Lecturers adopting the main text are permitted to download the manual as required.

Page 2: elliottv10

2 © Pearson Education Limited 2006

Pearson Education Limited Edinburgh Gate Harlow Essex CM20 2JE England

and Associated Companies around the world

Visit us on the World Wide Web at: www.pearsoned.co.uk

First published 2006

©Pearson Education 2006 The rights of Barry Elliott and Jamie Elliott to be identified as authors of this Work have been asserted by them in accordance with the Copyright, Designs and Patents Act 1988.

ISBN: 0 273 70365 X

All rights reserved. Permission is hereby given for the material in this publication to be repro-duced for OHP transparencies and student handouts, without express permission of the Publishers, for educational purposes only. In all other cases, no part of this publication may be reproduced, stored in a retrieval system, or transmitted in any form or by any means, electronic, mechanical, photocopying, recording, or otherwise without either the prior written permission of the Publishers or a licence permitting restricted copying in the United Kingdom issued by the Copyright Licensing Agency Ltd, 90 Tottenham Court Road, London W1T 4LP.

Page 3: elliottv10

Barry Elliott and Jamie Elliott: Financial Accounting and Reporting (tenth edition) – Instructor’s Manual

3 © Pearson Education Limited 2006

Contents Chapter 1 4 Chapter 2 13 Chapter 3 25 Chapter 4 38 Chapter 5 58 Chapter 6 64 Chapter 7 67 Chapter 8 73 Chapter 9 89 Chapter 10 109 Chapter 11 114 Chapter 12 130 Chapter 13 136 Chapter 14 143 Chapter 15 149 Chapter 16 153 Chapter 17 162 Chapter 18 172 Chapter 19 181 Chapter 20 189 Chapter 21 196 Chapter 22 211 Chapter 23 215 Chapter 24 233 Chapter 25 242 Chapter 26 253 Chapter 27 265 Chapter 28 275 Chapter 29 301 Chapter 30 318 Chapter 31 320 Chapter 32 329

Page 4: elliottv10

Barry Elliott and Jamie Elliott: Financial Accounting and Reporting (tenth edition) – Instructor’s Manual

4 © Pearson Education Limited 2006

C H AP T E R 1

Chapter 1: Question 1 – Jane Parker

(i) Cash budget (£000)

Jan Feb Mar Apr May June Total Initial capital 150.00 82.50 232.50

Customers . . . . 60.00 75.00 135.00

Total receipts 150.00 . . 82.50 60.00 75.00 367.50 Machinery 30.00 30.00

Motor vehicles 24.00 24.00

Premises 75.00 75.00

Drawings 1.20 1.20 1.20 1.20 1.20 1.20 7.20

Suppliers 30.00 48.00 60.00 60.00 60.00 258.00

Rates 1.20 1.20

Wages 2.25 2.25 2.25 2.25 2.25 2.25 13.50

General expenses 0.75 0.75 0.75 0.75 0.75 3.75

Insurance . . . . . 2.10 2.10

Total payments 132.45 35.40 52.20 64.20 64.20 66.30 414.75

Net cash flow 17.55 (35.40) (52.20) 18.30 (4.20) 8.70

Balance b/f – 17.55 (17.85) (70.05) (51.75) (55.95)

Balance c/f 17.55 (17.85) (70.05) (51.75) (55.95) (47.25) (47.25)

(ii) Cash flow summary statement (£000)

Realised operating cash flows for the period ended 30 June 20X1 Receipts from customers 135.00 Payments to:

Suppliers 258.00

Rates 1.20

Wages 13.50

General expenses 3.75

Insurance 2.10

278.55 (143.55)

Page 5: elliottv10

Barry Elliott and Jamie Elliott: Financial Accounting and Reporting (tenth edition) – Instructor’s Manual

5 © Pearson Education Limited 2006

For information only

Statement of financial position as at 30 June 20X1

£000 Capital – introduced 232.50

– withdrawn (7.20)

Net operating cash flows :Realised (143.55)

:Unrealised (7.80)

73.95

Premises (NRV) 75.00

Vehicles (NRV) 19.20

Machinery (NRV) 27.00

Net cash balance (47.25)

73.95

(iii) Further information re Jane Parker

(a) Nature of business linked to Parker’s business background, technical ability, special skills, know how, existing/terminated business involvement, contacts, associates, related parties.

(b) Type of business unit to be used and rationale for its selection.

(c) Sources of long and short-term capital.

(d) Products’ life cycle, cash flow projections over product life cycle.

(e) Initial investment in fixed assets and their terminal value at the end of the life cycle.

(f) Parker’s attitude to risk and how this affects the choice of discount rate and payback period.

Page 6: elliottv10

Barry Elliott and Jamie Elliott: Financial Accounting and Reporting (tenth edition) – Instructor’s Manual

6 © Pearson Education Limited 2006

Chapter 1: Question 2 – Mr Norman

(a) Purchases budget (£000)

Jan Feb Mar Apr May June Sales 15.00 20.00 35.00 40.00 40.00 45.00

Gross profit 3.00 4.00 7.00 8.00 8.00 9.00

Purchases 12.00 16.00 28.00 32.00 32.00 36.00

Payments 12.00 16.00 28.00 32.00 32.00

Notes:

• This is a start-up situation.

• Purchases = projected sales less a gross margin on sales at 20%.

• Goods are bought in month of sale; assume stocks remain constant.

(b) Cash flow statement (£000)

Jan Feb Mar Apr May June Total Initial capital 50.00 50.00

Cash Sales 7.50 10.00 17.50 20.00 20.00 22.50 97.50

Credit Sales - 7.50 10.00 17.50 20.00 20.00 75.00

57.50 17.50 27.50 37.50 40.00 42.50 222.50 Premises 80.00 80.00

Rent and rates 2.20 2.20 2.20 2.20 2.20 2.20 13.20

Suppliers 12.00 16.00 28.00 32.00 32.00 120.00

Commission 0.30 0.40 0.70 0.80 0.80 3.00

Wages 0.60 0.60 0.60 0.60 0.60 0.60 3.60

Insurance 3.50 - - - - - 3.50

86.30 15.10 19.20 31.50 35.60 35.60 223.30

Net cash flow (28.80) 2.40 8.30 6.00 4.40 6.90

Balance b/f - (28.80) (26.40) (18.10) (12.10) (7.70)

Balance c/f (28.80) (26.40) (18.10) (12.10) (7.70) (0.80) (0.80)

Page 7: elliottv10

Barry Elliott and Jamie Elliott: Financial Accounting and Reporting (tenth edition) – Instructor’s Manual

7 © Pearson Education Limited 2006

(c) Statements of operating cash flows and financial position

Realised operating cash flows for the period ended 30 June 20X8 £000 Receipts from customers 172.50

Payments to:

Suppliers 120.00

Rates 13.20

Wages 3.60

Commission 3.00

Insurance 3.50

143.30 29.20

Notes:

• The cash flow statement with summary attached is effectively a six-month cash budget showing the cash received, cash paid each month and the resulting month-end balances.

• It is necessary to separate sales and purchase transactions into cash and on credit and to identify clearly the month of receipt and payment.

• Commission is paid in the month after the sale is made, and all other cash flows are clearly indicated and allocated to specific months.

• Note that the format of the cash flow statement brings out key figures – for management decision and control, e.g.

• Month-end balances – assists in the control of liquidity

• cash deficiencies – identifies how much must be financed

• early warning – allows management to approach appropriate sources

• cash surpluses – identifies amounts to be invested on best terms.

Statement of financial position as at 30 June 20X8 £000

Capital – introduction 50.00

Net operating cash flows :Realised 29.20

:Unrealised (4.00)

75.20

Premises (NRV) 76.00

Net cash balance (0.80)

75.20

Page 8: elliottv10

Barry Elliott and Jamie Elliott: Financial Accounting and Reporting (tenth edition) – Instructor’s Manual

8 © Pearson Education Limited 2006

Notes:

• This statement shows net assets of £75,200

• make up: premises £76,000 less the negative cash balance £800.

• The negative cash balance indicates need for overdraft arrangements.

• The statement is based on cash flow concept

• it ignores accrual-based figures (£36,900 less £25,250)

• accruals are not regarded as real assets and liabilities

• critics of the cash flow concept would maintain that its utility has therefore been seri-ously diminished.

(d) Letter to bank requesting an overdraft facility should include

• The maximum overdraft facility of £28,800

• required at the end of January

• will be eliminated by July

• Overdraft will fall progressively as per the cash budget.

• It might be practical to request a limit of £30,000

• for the full six-month period

• reducing to £15,000 thereafter to allow for contingencies. The facility only to be called on as required.

• Refer to the Cash Budget to support the request

• confirm that it is based on the most likely scenario

• agree a repayment schedule.

• Specify that collateral security is available in form of premises if it should be required.

• If not an existing customer

• give outline details of business background

• explain future plans.

Page 9: elliottv10

Barry Elliott and Jamie Elliott: Financial Accounting and Reporting (tenth edition) – Instructor’s Manual

9 © Pearson Education Limited 2006

Chapter 1: Question 3 – Fred and Sally

(i) (a) Decision to close in winter

April May June July Aug Sept Oct Nov Dec Jan Feb March £ £ £ £ £ £ £ £ £ £ £ £ Opening balance 100 12,100 29,860 35,120 40,380 45,640 50,900 45,160 33,660 34,660 35,660 36,660

Sales (Wk1) 14,000 19,760 19,760 19,760 19,760 19,760 5,760

Boards (custom-built) 10,000 10,000 10,000 10,000 10,000 10,000

14,100 31,860 59,620 64,880 70,140 75,400 66,660 55,160 33,660 34,660 35,660 36,660

Salary in Lanzarote: Fred and Sally: income 500 500 500 500 500 500 Staff: income 500 500 500 500 500 500 Purchases 16,000 16,000 16,000 16,000 16,000 16,000 Materials for boards (custom-built)(Wk2) 6,500 6,500 6,500 6,500 6,500 6,500 Wages 1,000 1,000 1,000 1,000 1,000 1,000 Rent 500 500 500 500 500 500 Misc. costs 500 500 500 500 500 500 12,100 29,860 35,120 40,380 45,640 50,900 45,160 33,660 34,660 35,660 36,660 37,660

£ Wk1 (£120,000/6) = 20,000 Wk 2 Under this alternative Per board Total – 6,000 (paid by credit card) Materials £500 14,000 (cash received in month) Sails (not Dryline) 150 + 5,760 (last month’s credit card sales less 4%) 650 × 10 = £6,500 19,760

Page 10: elliottv10

Barry Elliott and Jamie Elliott: Financial Accounting and Reporting (tenth edition) – Instructor’s Manual

10 © Pearson Education Limited 2006

(i) (b) Decision to remain open in winter and keep Dryline agency

April May June July Aug Sept Oct Nov Dec Jan Feb March £ £ £ £ £ £ £ £ £ £ £ £ Opening balance 100 11,5839,41017,237 24,064 30,891 37,718 32,295 40,528 38,761 37,994 37,227

Sales 14,000 19,76019,76019,760 19,760 5,760 5,760 5,760

576 1,400 1,976 1,976 1,976 1976

Boards 1,000 1,000 10,000 10,000 10,000 10,000 10,000 10,000 1,000 1,000 1,000

1,000

15,676 32,343 40,170 46,997 53,824 60,651 54,878 44,271 42,504 41,737 40,970 40,203

Purchases (Wk3/4) 1,493 14,933 14,933 14,933 14,933 14,933 14,933 1,493 1,493 1,493 1,493 1,493

Boards 600 6,000 6,000 6,000 6,000 6,000 6,000 600 600 600 600 600

Wages 1,000 1,000 1,000 1,000 1,000 1,000 1,000 1,000 1,000 1,000 1,000 1,000

Rent 500 500 500 500 500 500 500 500 500 500 500 500

Misc costs 500 500 500 500 500 500 150 150 150 150 150 150

Closing balance 11,583 9,410 17,237 24,064 30,891 37,718 32,295 40,528 38,761 37,994 37,227 36,460

Wk 3 Wk4

20,000 × 2,000 ×

60% 60%

Non Dryline 12,000 × 1,200 ×

80% 80%

9,600 + 8,000 × 2/3 960 + 800 × 2/3

Dryline 5,333 533

14,933 1,493

Page 11: elliottv10

Barry Elliott and Jamie Elliott: Financial Accounting and Reporting (tenth edition) – Instructor’s Manual

11 © Pearson Education Limited 2006

Notes supporting (i) (a)

Assuming closure during the winter months means that:

• there are no sales from October to March

• salaries of £1,000 per month are received from October to March

• a start-up situation arises each April with consequential effect on cash from sales of both custom-built and non-custom-built boards.

Cash from sales is calculated as follows:

Non-custom-built (i) 70% received immediately = 14,000 (ii) 30% via credit card finance less 4% (£6,000 – 240) = 5,760

19,760 However, only £14,000 is received in first month and £5,760 in October.

Custom-built Received per month – two months in arrears = £10,000

Cash payments are calculated as follows:

Non-custom-built 80% of the sales value is paid two months after purchase

Custom-built Materials cost £650 each; total £6,500 per month; paid 2 months after purchase

Other fixed costs Wages, rent, miscellaneous costs at uniform monthly rate of £2,000 for 6 months

Projected cash balances show an upward trend to September and then they fall away.

Notes supporting (i) (b)

Cash receipts are calculated as follows:

Non-custom-built Receipts are in two cycles: April to September and October to March. April cash received is £14,576 made up of 70% of £20,000 being the April sales plus 96% of March’s credit card sales (i.e. 96% of £600). From May to September (inclusive) cash receipts will be the same as calculated on the clo-sure assumption i.e. £19,760 per month. October receipts are £7,160 i.e. £5,760 from September plus £1,400 from October sales with £1,976 again for five subsequent months.

Custom-built Cash receipts from June to November will be £10,000 per month as under the closure as-sumption.

Page 12: elliottv10

Barry Elliott and Jamie Elliott: Financial Accounting and Reporting (tenth edition) – Instructor’s Manual

12 © Pearson Education Limited 2006

In the winter months there will be a further £6,000 except that customers took an unex-plained average of three months in winter, receipts are nil in December.

Cash payments are calculated as follows:

Non-custom-built From May to October purchases are £14,933 and 10% of this for the remaining six months

Custom-built As for closure assumption

Other fixed costs Wages, rent, miscellaneous costs at uniform monthly rate of £2,000 for 6 months

(ii) Additional information required in order to advise

At first glance closing in the winter generates more cash but:

1. What about loss of future earnings from Dryline?

2. Is there any potential damage to customer goodwill?

3. Will this allow competition to creep into market?

Notes relating to part (a):

• This is a start-up situation.

• Purchases = projected sales less a gross margin on sales at 20%.

• Goods are bought in month of sale; assume stocks remain constant.

We need to know more about the business e.g:

• Objectives

• What are Fred/Sally’s objectives?

• Is it a short-term pleasant lifestyle with high income, or long-term market maximisa-tion?

• Demand

• state of market

• whether demand for Dryline sails is expanding or contracting

• possibility of other agencies

• possibility of opening in other areas

• possibility of increasing winter sales

• higher profile for custom-made boards e.g. outline in Lanzarote

• whether or not the custom-made boards are independent of the rest of the business i.e. a different market.

Page 13: elliottv10

Barry Elliott and Jamie Elliott: Financial Accounting and Reporting (tenth edition) – Instructor’s Manual

13 © Pearson Education Limited 2006

C H AP T E R 2

Chapter 2: Question 1 – Jane Parker

(a) Cash budget (€000)

Jan Feb Mar Apr May June Total Initial capital 150.00 75.00 225.00

Customers 60.00 75.00 75.00 210.00

Total receipts 150.00 60.00 75.00 150.00 435.00 Machinery 30.00 30.00

Motor vehicles 24.00 24.00

Premises 75.00 75.00

Drawings 1.50 1.50 1.50 1.50 1.50 1.50 9.00

Suppliers 30.00 48.00 60.00 60.00 60.00 258.00

Rates

Wages 2.25 2.25 2.25 2.25 2.25 2.25 13.50

General expenses 0.75 0.75 0.75 0.75 0.75 3.75

Insurance 26.40 26.40

132.75 34.50 52.50 64.50 64.50 90.90 439.65 Net cash flow 17.25 (34.50) (52.50) (4.50) 10.50 59.10

Balance b/f – 17.25 (17.25) (69.75) (74.25) (63.75)

Balance c/f 17.25 (17.25) (69.75) (74.25) (63.75) (4.65) (4.65)

All balances are overdrawn except for January 20X1

Feb Mar Apr May June o/d 17.25 69.75 74.25 63.75 4.65

Note: No entries will be made for the 20X0/X1 local taxes that are paid in Feb 20X2 – this situation arose because Jane Parker had assumed that the business would only pay the taxes from the start of the tax year e.g. 1.4.20X1. However, there will be an entry in the profit and loss and balance sheet.

Page 14: elliottv10

Barry Elliott and Jamie Elliott: Financial Accounting and Reporting (tenth edition) – Instructor’s Manual

14 © Pearson Education Limited 2006

(b) Jane Parker – profit and loss account for six months ended 30.6.20X1

€000 €000

Sales [60.00 + (5 × 75.00)] 435.00

Purchases 378.00

Closing inventory (30.00)

Cost of sales 348.00

Gross profit 87.00

Wages 13.50

General expenses 4.50

Local taxes [1.1.X1 – 30.6.X1] 4.00

Insurance 13.20

Depreciation

– Vehicles 2.40

– Machinery 1.50 39.10

Net profit 47.90

Budgeted balance sheet as at 30 June 20X1

Capital 225.00

Net profit 47.90

Less: Drawings (9.00)

263.90

Non-current assets Premises 75.00

Vehicles 24.00

Less: Depreciation 2.40 21.60

Machinery 30.00

Less: Depreciation 1.50 28.50

Current assets Inventory 30.00

trade receivables [3 × 75.00] 225.00

Insurance 13.20 268.20

Page 15: elliottv10

Barry Elliott and Jamie Elliott: Financial Accounting and Reporting (tenth edition) – Instructor’s Manual

15 © Pearson Education Limited 2006

Current liabilities Trade payables 120.00

Local taxes [1.1.X1 – 30.6.X1] 4.00

Bank overdraft 4.65

General expenses 0.75 (129.40)

Net current assets 138.80

263.90

(c) Possible action to deal with exceeding agreed overdraft limit

• Approach bank to re-negotiate the overdraft or arrange a loan facility for an agreed term.

• The amount and period that additional facilities are required depends on preparing a pro-jected cash flow statement for a longer period taking into account future plans e.g. owner’s drawings requirement, any additional capital expenditure.

• In particular, consider alternatives such as:

– leasing vehicles and/or machinery

– mortgaging the property

– getting debts in quicker

– introducing more capital

– obtaining or providing loan capital.

Page 16: elliottv10

Barry Elliott and Jamie Elliott: Financial Accounting and Reporting (tenth edition) – Instructor’s Manual

16 © Pearson Education Limited 2006

Chapter 2: Question 2 – Mr Norman

(a) Purchases budget ($000)

Jan Feb Mar Apr May Jun Sales units 1.65 2.20 3.85 4.40 4.40 4.95

– Closing inventory 0.55 0.96 1.10 1.10 1.24

+ Closing inventory 0.55 0.96 1.10 1.10 1.24 1.38

Purchases units 2.20 2.61 3.99 4.40 4.54 5.09

Purchases Sales $000 $000

Jan (2,200 × 40) 88.00 82.50 (1,650 × 50)

Feb (2,610 × 40) 104.40 110.00 (2,200 × 50)

Mar (3,990 × 40) 159.60 192.50 (3,850 × 50)

Apr (4,400 × 40) 176.00 220.00 (4,400 × 50)

May (4,540 × 40) 181.60 220.00 (4,400 × 50)

Jun (5,090 × 40) 203.60 247.50 (4,950 × 50)

913.20 1,072.50

(b) Cash flow forecast (£000)

Jan Feb Mar Apr May June Total Initial capital 150.00 150.00

Cash Sales 41.25 55.00 96.25 110.00 110.00 123.75 536.25

Credit sales 41.25 55.00 96.25 110.00 110.00 412.50

191.25 96.25 151.25 206.25 220.00 233.75 1098.75 Premises 80.00 80.00

Commission 1.65 2.20 3.85 4.40 4.40 16.50

Suppliers 88.00 104.40 159.60 176.00 181.60 709.60

Administration 8.00 8.00 8.00 8.00 8.00 8.00 48.00

Wages 17.00 17.00 17.00 17.00 17.00 17.00 102.00

Insurance 0.35 0.35

Total payments 105.35 114.65 131.60 188.45 205.40 211.00 956.45

Net cash flow 85.90 (18.40) 19.65 17.80 14.60 22.75

Balance b/f – 85.90 67.50 87.15 104.95 119.55

Balance c/f 85.90 67.50 87.15 104.95 119.55 142.30

Page 17: elliottv10

Barry Elliott and Jamie Elliott: Financial Accounting and Reporting (tenth edition) – Instructor’s Manual

17 © Pearson Education Limited 2006

(c) Budgeted profit and loss account for six months ended 30 June 20X8

$000 $000 Sales 1072.50

Purchases 913.20

Closing inventory [1,380 units × £40] (55.20)

Cost of sales 858.00

Gross profit 214.50

Wages 102.00

Administration 48.00

Commission [2% of 1072.50] 21.45

Insurance 0.18

Amortisation of lease 8.00

179.63

Net profit 34.87

Budgeted balance sheet as at 30 June 20X8

$000 $000 Capital 150.00

Net profit 34.87

184.87 Non-current assets Leasehold premises 80.00

Less amortisation (8.00)

72.00

Current assets

Inventory 55.20

Trade receivables 123.75

Prepayments – insurance .17

Cash 142.30

321.42

Current liabilities

Trade payables 203.60

Commission 4.95

208.55

Net current assets 112.87

184.87

Page 18: elliottv10

Barry Elliott and Jamie Elliott: Financial Accounting and Reporting (tenth edition) – Instructor’s Manual

18 © Pearson Education Limited 2006

(d) Investment of surplus funds

• Acid test ratio At the end of the first six months’ trading Norman’s balance sheet shows that the acid test ratio is 1.28:1 (266.22/208.55) – this is higher than the basic 1:1 ratio but it should be com-pared with that of similar businesses in the same industry to establish a norm. It would appear however that the business has surplus funds to invest.

• Amount to invest A projected cash flow statement is required taking into account future plans re owner’s drawing requirements, future capital commitments and working capital criteria e.g. debtor collection and creditor payment terms.

• Period to invest The projected cash flow will give an indication of the period of the investment e.g. it could range from overnight on the money market to term investments.

The important aspect is that the owner should be aware of the projected cash flows so that return on surplus funds can be maximised.

Page 19: elliottv10

Barry Elliott and Jamie Elliott: Financial Accounting and Reporting (tenth edition) – Instructor’s Manual

19 © Pearson Education Limited 2006

Chapter 2: Question 3 – The Piano Warehouse Company Limited

(a) Conventional profit calculation

(i) (ii) (iii) 4 sold 2 part made 2 repaired/sold £ £ £ £ £ Sales 8,000 3,000

Materials 2,000 900 1,800

Labour 2,800 800 400

Overheads 800 100

Cost of production 5,600 1,800 2,200

Less: inventory – 1,800 – Cost of sales 5,600 2,200

Profit 2,400 800

Comments on profit calculation:

(i) 4 pianos sold: profit is simply sales less cost. (ii) 2 partly completed pianos:

• No profit has been recognised

• Delay recognition until sale to comply with matching concept

• Work-in-progress carried forward.

Match with revenue when sales take place

presumably in the following year. (iii) Rebuilt pianos: profit has been taken in full

• Assumes no default on hire purchase payments.

• An alternative procedure that could be considered would be to

provide for unrealised profit on instalments outstanding

create a provision of £533 [2/3 of £800]

this reduces profit from £800 to £267 [1/3 of £800].

(b) It has been assumed that

• the work-in-progress is held at cost; and

• no profit has been recognised.

• The payment in advance of £900 will be shown as a creditor in the balance sheet.

Page 20: elliottv10

Barry Elliott and Jamie Elliott: Financial Accounting and Reporting (tenth edition) – Instructor’s Manual

20 © Pearson Education Limited 2006

Given the facts that

• there is an assured market for work-in-progress; and

• the pianos are 50% complete a case could be made out for attributing a % of the total profit of £900 (£4,500 – £ 3,600 estimated cost to produce). Except in the simplest cases, the matching process can be highly subjective.

The conventional approach is to recognise revenue

• at the point of sale or, alternatively

• on completion of production or receipt of payment.

The core determinant is the identification of the completion of the earnings cycle as sig-nalled by a critical event e.g. production, delivery, receipt of payment. Applying these alternatives to The Piano Warehouse Company and assuming the critical event is: (i) On sale/delivery Recognise profit from the sale of 4 pianos (ii) During production Recognise profit from the 50% complete pianos (iii) On receipt of payment Restrict the profit from the 2 rebuilt pianos to the proportion of cash received. A more detailed consideration in IAS 18 relating to the sale of goods suggests the criti-cal event for revenue recognition hinges on 2 conditions:

• There has been a transfer to the buyer of the significant risks and rewards of owner-ship, and

• no significant uncertainty exists concerning the sale price, costs, likelihood of rejec-tion and return.

(c) The significant cost conventions are:

• Accruals for recognising profit

• Matching for associating costs incurred and to be incurred in arriving at a profit fig-ure

• Prudence in attributing/not attributing profit on work-in-progress.

(d) The profit for the year could be increased by the recognition of a proportion of the profit on the two pianos in the course of production with an estimate of the amount earned to date.

The maximum figure would normally be £450 [50% of the estimated total profit] but this would be dependent upon being reasonably certain that the future costs will not ex-ceed the estimate. Conversely, it could be argued that the £800 profit on the rebuilt pianos should be reduced to £267.

Page 21: elliottv10

Barry Elliott and Jamie Elliott: Financial Accounting and Reporting (tenth edition) – Instructor’s Manual

21 © Pearson Education Limited 2006

Chapter 2: Question 4 – FRRP and Wiggins Group

FRRP membership

The Chairman of the Panel is Richard Sykes QC and the Deputy Chairman Matthew Patient CBE. There are currently 15 other Panel members drawn from a broad spectrum of commerce and the professions. Individual cases are normally dealt with by specially constituted groups of 5 or more members.

FRRP remit

The remit of the Financial Reporting Review Panel is to examine the annual accounts of public and large private companies to see whether they comply with the requirements of the Compa-nies Act 1985. Within this framework a main focus is material departures from accounting standards where such a departure results in the accounts in question not giving a true and fair view as required by the Act.

Action if accounts are defective

Where a company’s accounts are defective the Panel will, wherever possible, endeavour to se-cure their revision by voluntary means, but if this approach fails it is empowered to make an application to the court under section 245B of the Companies Act 1985 for an order compelling their revision. To date no court applications have been made, though in some instances the nec-essary steps have been at an advanced stage.

FRRP is reactive not proactive

The Panel does not itself monitor or actively initiate scrutinies of company accounts for possible defects, but acts on matters drawn to its attention, either directly or indirectly. The Panel’s re-sponsibilities do not extend to the directors’ report, summary financial statements or interim statements.

Wiggins: Revenue recognition

Company policy

It is the company’s accounting policy to recognise revenue in respect of commercial property sales on exchange of contract. In the 1999 accounts however, two sales had been recognised on the basis of non-binding heads of agreement and a third in respect of a contract dated after the end of the 1999 accounting period. In the 2000 accounts, the directors accepted that these treat-ments were not appropriate and adjusted the 1999 comparatives by the £21.5m turnover incorrectly recognised.

Page 22: elliottv10

Barry Elliott and Jamie Elliott: Financial Accounting and Reporting (tenth edition) – Instructor’s Manual

22 © Pearson Education Limited 2006

FRRP questions raised re contracts

In those accounts, the directors did not address other concerns previously expressed by the Panel in connection with these contracts. One of the contracts was conditional upon the company ob-taining planning permission on terms satisfactory to the purchaser without which he had certain rights not to proceed. A second contract had the appearance of a financing transaction rather than an outright sale which, under FRS 5, should not be recognised until the risks and rewards of ownership pass at a future date. In the particular circumstances, the Panel was of the view that neither contract could be recognised in the 2000 accounts.

The directors have now accepted the Panel’s view and, in the revised accounts, have adjusted for these and other sales in earlier years, applying the same principles.

Revenue recognition

Company policy in 1999 accounts

The 1999 accounts contained an accounting policy for turnover in the following terms: ‘Commercial property sales are recognised at the date of exchange of contract, providing the Group is reasonably assured of the receipt of the sale proceeds.’

FRRP accepted policy in itself was not objectionable

The Panel accepted that this wording was similar to that used by many other companies and was not on the face of it objectionable. In reviewing the company’s 1999 accounts the Panel noted that the turnover and profits recognised under this policy were not reflected in similar inflows of cash; indeed, operating cash flow was negative and the amount receivable within debtors of £46m represented more than the previous two years’ turnover of £44m. As a result, the Panel enquired into the detailed application of the policy.

FRRP enquired into detailed application of the policy where contracts not exchanged

This enquiry established that revenue in respect of the sale of Manston Park and the Northern Grass area of Manston Airport had been recognised at 31 March 1999 on the strength of non-binding Heads of Agreement which were not turned into contracts until 29 July 1999. The enquiry also established that the sale contract for Fairfield had not been signed until 1 April 1999. In the Panel’s view the manner in which these transactions had been accounted for was not in accordance with the stated accounting policy and was unacceptable.

The Panel also expressed its concern that the chairman’s statement for 1999 compounded the error in the accounts by stating that during the year the most important achievements were the exchange of contracts with MEPC for the sale and development of Manston Park and Lon-don Manston Airport when the contract with MEPC for Manston had, in fact, not been exchanged during that year.

Page 23: elliottv10

Barry Elliott and Jamie Elliott: Financial Accounting and Reporting (tenth edition) – Instructor’s Manual

23 © Pearson Education Limited 2006

Company response

The company accepted that the 1999 accounts were in error in respect of these transactions and decided to recognise turnover on exchange of contracts in accordance with its stated ac-counting policy which it did in the 2000 accounts by amending the 1999 comparatives.

FRRP enquired into detailed application of the policy where contracts were exchanged

However, the Panel had also questioned whether it would be correct to recognise revenue in respect of two of these transactions even after the contracts had been exchanged in the following year. The contract for the sale of Fairfield, concluded on 1 April 1999, was conditional upon the company’s subsequent fulfilment of a material condition: namely, that planning permis-sion had been obtained on terms satisfactory to the purchaser and without which the purchaser had certain rights not to proceed. Furthermore, the contract for the sale of the Northern Grass area of Manston appeared to have the characteristics of a financing deal rather than an outright sale and, under FRS 5, should not be recognised until the risks and rewards of ownership pass at a future date. The company’s decision to recognise turnover and profit in respect of the conditional Fairfield contract and the Northern Grass transaction in its 2000 accounts was made without discussion with the Panel and in spite of the Panel’s reservations. The directors justified their recognition of revenue from the sale of Fairfield on the basis that they believed the planning permission was ‘virtually certain’ to be obtained in due course. The Panel noted that the company had first recognised revenue in respect of Fairfield in its 1997 and 1998 accounts and had had to reverse those amounts in its original 1999 accounts when the contracts had lapsed. The contracts signed on 1 April 1999 replaced these original contracts. On the principle of reflecting sales of property that are subject to planning permission, the Panel had two main concerns, which are interrelated. The first is that, if the company has still to perform a significant amount of work in order to satisfy the condition, it has not yet com-pleted the earning process sufficiently to entitle it to recognise the revenue at the balance sheet date. The second is that the outcome of a conditional contract is necessarily uncertain, and unless that uncertainty has been reduced to an acceptable level by the time that the accounts have been finalised, in general, the prudent course would be not to recognise the conditional contract until the condition is satisfied. The Panel accepts that there may be specific instances where it might be appropriate to recognise a sale even though it remains conditional; for exam-ple, where all the work required to satisfy the conditions has been performed and the relevant costs have been incurred before the year-end and the relevant conditions satisfied before the ac-counts are issued, it may be reasonable to recognise the sale. Those basic steps had not been effected in the case of Fairfield before the 2000 accounts were issued. The company has now accepted the Panel view. The amount of turnover recognised in the 2000 accounts in error and now to be corrected amounts to £37m.

Revisions required to the accounts – turnover restated

As indicated in its interim statement, the company has also acknowledged that applying these principles, £2.34m turnover included in the 1996 accounts and subsequently reversed in 1997

Page 24: elliottv10

Barry Elliott and Jamie Elliott: Financial Accounting and Reporting (tenth edition) – Instructor’s Manual

24 © Pearson Education Limited 2006

and 1998 should not have been recognised in the 1996 accounts. The accounts for 1996 to 1998 are being revised to correct these entries. A similar adjustment is being made by the company to its 1995 accounts.

Revisions required to the accounts – compliance with FRS 5

As to the sale of Northern Grass the Panel considered that the substance of the transaction re-flected a development being financed by the purchaser with the company paying an interest cost on monies received from the purchaser and retaining certain risks and rewards for the time be-ing. Accordingly, in compliance with FRS 5, the transaction should have been treated as a financing arrangement until the interest payments ceased, either on completion of the develop-ment or on 30 January 2002, whichever is the earlier. The company has now accepted the Panel view. The amount of turnover recognised in the 2000 accounts in error and now to be corrected amounts to £5m.

Debtors

The Companies Act 1985 requires the amounts of debtors that are receivable after more than one year to be separately disclosed. In the 1999 accounts £10m of the anticipated proceeds from the sale of Manston Park was wrongly included in debtors due within 12 months. The company agreed to correct this error in the 2000 accounts but in the event recognition of the sale was reversed in its entirety.

Page 25: elliottv10

Barry Elliott and Jamie Elliott: Financial Accounting and Reporting (tenth edition) – Instructor’s Manual

25 © Pearson Education Limited 2006

C H AP T E R 3

Chapter 3: Question 1 – Jim Bowater

(a) Refer to Question 2 (a) below for description of underlying theory

(b) Jim Bowater ideal economic income model:

Investment of £36,000, cost of capital 20%

(i) Jim’s economic income (£) for each of the three years is:

31 Dec 20X5 6,828 31 Dec 20X6 6,695 31 Dec 20X7 6,833 (ii) Jim’s economic capital will be preserved at the 1 January 20X5 level of £34,144 provided:

• he reinvests excess actual income of £672 on 31 December 20X5 and £34,107 at 31 De-cember 20X7, generating a return of 20%

• an excess of actual income of £695 at 31 December 20X6 created, in effect, a cumulative injection of capital of (672 – 695) £23

• to maintain his income of 20% p.a. will necessitate an investment of £34,167 from the pro-ceeds of the proposed sale.

Workings

(A) Ideal economic income (i.e. conditions of certainty) Period C Kt Kt–1 Ye C – Ye 20X5 t0 – t1 7,500 33,472 (b) 34,144 (a) 6,828 672

20X6 t1 – t2 6,000 34,167 (c) 33,472 (b) 6,695 (695)

20X7 t2 – t3 41,000 34,167 (c) 6,833 34,167

54,500 20,356 34,144

(a) t0 – t1 7,500 + 6,000 + 41,000 = 34,144

1.2 1.22 1.23

(b) t1 – t2 6,000 + 41,000 = 33,472

1.2 1.22

(c) t2 – t3 41,000 = 34,167

1.2

Page 26: elliottv10

Barry Elliott and Jamie Elliott: Financial Accounting and Reporting (tenth edition) – Instructor’s Manual

26 © Pearson Education Limited 2006

(B) Reinvestment under certainty to maintain 20% p.a. income Economic income from: original investment reinvestment total economic income t0 – t1 6,828 – 6,828

t1 – t2 6,695 (20% 672) 133 (approx.) 6,828

t2 – t3 6,833 (20% 23) (5) (approx.) 6,828

Page 27: elliottv10

Barry Elliott and Jamie Elliott: Financial Accounting and Reporting (tenth edition) – Instructor’s Manual

27 © Pearson Education Limited 2006

Chapter 3: Question 2 – Hicks’s Concept of Income; Spock

(a) Hick’s economic model of income and capital

Hicks’s economic model of income and capital is based on his concept of ‘welloffness’.

• Welloffness is: the maximum income enjoyed by the individual without depleting that indi-vidual’s capital stock.

• It is based on the precept of consumption which embraces the opportunity for consump-tion as well as actual consumption.

• As an extension of Fisher’s original model: it takes savings into account.

It is an ex ante model in that it usually measures expected income in advance of the time period concerned.

• Measurement of capital is necessitated in order to compute income.

• Income is the difference between opening and closing valuations of capital stock.

• The capital stock is computed by utilising the concept of present values.

• This concept adopts the idea of compound interest in order to compensate for the time ele-ment between cash flows.

Limitations

In the field of accountancy there are serious practical limitations in measuring the accountant’s version of income and capital e.g.

• Subjectivity: the present value factor, often referred to as the discount cash flow element, is subjective.

• It requires the use of an interest rate and, as such, depending upon personal inclinations, it can utilise opportunity cost of capital, or the return on existing capital employed within a business entity, or contemporary short-term interest rates such as that charged on bank overdrafts, or the average rate pertaining in the current economic climate, or a speculative rate as assessed on the basis of perceived risk involved.

• Unrealised and realised flows: the model uses a mix of unrealised and realised cash flows. As a measure it is thus not of practical value in determining taxation liability and dividend policy.

• Financial strategy: attainment of flows as per a financial strategy is an integral part of the calculations. Targets are rarely achieved with precision. Variations from target destroy the model’s accuracy. Predictions are invariably unachievable with absolute accuracy.

• Windfalls: windfall flows cannot be foreseen and consequently cannot be incorporated within the model.

• Balance sheet values: balance sheet valuations of net assets or capital employed concern aggregations of individually valued assets and itemised liabilities.

It is not easy to apply the concept of present values across a range of individual assets and li-abilities for balance sheet discount purposes.

Page 28: elliottv10

Barry Elliott and Jamie Elliott: Financial Accounting and Reporting (tenth edition) – Instructor’s Manual

28 © Pearson Education Limited 2006

(b) Calculate Spock’s ideal economic income using Hicks’s theorem

Economic capital value of the business at KO Year Cash Flow DCF Factor PV 1/(1+r)n £ £ K1 400 0.909 364

K2 500 0.826 413

K3 600 0.751 451

400 0.751 300

1,900 1,528

Economic value at X0 i.e. the beginning of the year is £1,528 [Note: initial capital was £1,000 therefore subjective goodwill is £528]

Economic capital value of the business at K1 Year Cash Flow DCF Factor PV 1/(1+r)n £ £ K1 400 1.000 400

K2 500 0.909 455

K3 600 0.826 496

400 0.826 330

1,900 1,681

Economic value at K1 = £1,681. So Y for Y1 = £1,681 – £1,528 = £153

Rate of income return = £153 / £1,528 = 10%

Economic capital value of the business at K2 Year Cash Flow DCF Factor PV 1/(1+r)n £ £ K1 400 1.100 440

K2 500 1.100 500

K3 600 0.909 546

400 0.909 363

1,900 1,849

Economic value at K2 = £1,849. So Y for Y2 = £1,849 – £1,681 = £168

Rate of income return = £168/£1,681 = 10%

Page 29: elliottv10

Barry Elliott and Jamie Elliott: Financial Accounting and Reporting (tenth edition) – Instructor’s Manual

29 © Pearson Education Limited 2006

Economic capital value of the business at K3

Year Cash Flow DCF Factor PV 1/(1+r)n £ £ K1 400 1.121 484

K2 500 1.100 550

K3 600 1.000 600

400 1.000 400

1,900 2,034

Economic value at K3 = £2,034. So Y for Y3 = £2,034 – £1,849 = £185

Rate of income return = £185/£1,849 = 10%

Note rate of income return is a constant 10%.

Page 30: elliottv10

Barry Elliott and Jamie Elliott: Financial Accounting and Reporting (tenth edition) – Instructor’s Manual

30 © Pearson Education Limited 2006

Chapter 3: Question 3 – Jason

(a) (i) Accounting income for Jason (using historical cost concept and applying conventional accounting by compiling an Income Statement account).

Income statement for the year ended 31 December,20X1 £ £ Opening inventory 10,000 Sales less purchases 36,200

Gross profit c/d 41,700 Closing inventory 15,500

51,700 51,700

Depreciation 5,000 Gross profit b/d 41,700

Net profit 36,700 41,700 41,700

(Note: The Sales less purchases figure of £36,200 is derived from the debtors/creditors account below.)

So accounting profit = £36,700 (assumes traditional concept of going concern).

Workings are shown below in T account form

Cash Book 1 Jan 20X1 Balance 135,000 1.1.20X1 Purchase of business 130,000

Legal costs 5,000

135,000 135,000

20X1 Debtors/creditors 40,000 31 Dec 20X1 Drawings 25,000

Balance 15,000

40,000 40,000

1 Jan 20X2 Bal b/d 15,000

Purchase of business 1 Jan 20X1 Cash 130,000 1 Jan 20X1 Premises 100,000

Stock 10,000

Debtors 4,000

______ Goodwill 16,000

130,000 130,000

Shop premises 1.1.20X1 Purchase of

business 100,000 31 Dec 20X1 Balance 105,000

Page 31: elliottv10

Barry Elliott and Jamie Elliott: Financial Accounting and Reporting (tenth edition) – Instructor’s Manual

31 © Pearson Education Limited 2006

Cash

Legal costs capitalised 5,000

105,000 105,000

Depreciation 31 Dec 20X1 Balance c/d 5,000 31 Dec 20X1 P&L a/c 5,000

5,000 5,000

1 Jan 20X2 Balance b/d 5,000

Inventory 1 Jan 20X1 Purchase of

business 10,000 31 Dec 20X1 P&L a/c 10,000

31 Dec 20X1 P&L a/c 15,500

Goodwill 1 Jan 20X1 Purchase of

business 16,000

Capital 31Dec 20X1 Drawings 25,000 1 Jan 20X1 Cash 135,000

Balance c/d 146,700 31 Dec 20X1 P&L a/c 36,700

171,700 171,700

1 Jan 20 × 2 Balance b/d 146,700

Trade receivables/trade payables account

[prepared in order to derive net sales less purchases] 1 Jan 20X1 Purchase of 31 Dec 20X1 Cash balance 40,000

business:

receivables 4,000

20X1 Sales – purchases = balancing figure 36,200 31Dec 20X1 Payables c/d 5,000 31 Dec 20X1 Receivables c/d 5,200

45,200 45,200

1 Jan 20X2 Receivables b/d 5,200 1 Jan 20 × 2 Payables b/d 5,000

Drawings

31 Dec 20X1 Cash 25,000 31 Dec 20X1 Capital a/c 25,000

Page 32: elliottv10

Barry Elliott and Jamie Elliott: Financial Accounting and Reporting (tenth edition) – Instructor’s Manual

32 © Pearson Education Limited 2006

Balance sheets

(NA0) (NA1) At 1 Jan 20X1 At 31 Dec 20X1

Premises 100,000 100,000

Add capitalisation

of legal costs 5,000 5,000

105,000 105,000

Less depreciation 5,000

105,000 100,000

Goodwill 16,000 16,000

Inventory 10,000 15,500

Trade receivables 4,000 5,200

Cash 40,000

Less drawings 25,000 15,000

Trade payables (5,000)

135,000 146,700

So profit = NA1 – NA0 + Drawings = 146,700 – 135,000 + 25,000 = £36,700 as confirmed by the profit and loss account above. Comparing the opening and closing balance sheets and allowing for drawings will enable profit to be derived but it is usual for accounting profit to be shown via a profit and loss account.

• It has been assumed that the traditional historical cost concept applies.

• It was intended that the legal costs be capitalised giving a fair value at 1 January 20X1 of £105,000

• Thus depreciation is £5,000 (£105,000 – £100,000)

• Alternatively the £5,000 could have been treated as an expensed cost (i.e. written off in the profit and loss account)

• The net profit would remain as £36,700, the depreciation having been replaced by the legal costs.

• It was assumed that opening balance sheet values represent fair values (i.e. cost) of the indi-vidual assets concerned.

(ii) Realisable income

Y0-1 = Net RV1 – Net RVo + drawings = £136,200 – £135,000 + £25,000 = £26,200

Page 33: elliottv10

Barry Elliott and Jamie Elliott: Financial Accounting and Reporting (tenth edition) – Instructor’s Manual

33 © Pearson Education Limited 2006

Workings: Net realisable values At 1 Jan 20X1 At 31 Dec 20X1

Premises 85,000 105,000

Goodwill 16,000 16,000

Inventory 20,000 10,000

Trade receivables 5,200 4,000

Cash 40,000

Less drawings 25,000 15,000

Trade payables (5,000)

Net realisable values 136,200 135,000

Assumptions:

(a) The realisable values are not based on an enforced sale.

(b) Goodwill would possess a realisable value equivalent to its original cost in an enforced sale.

(c) The entity is capable of being sold as a business entity in order to realise goodwill.

Note: Some commentators might dispute the validity of goodwill because the concept of realis-able income contravenes the going concern concept. In this situation the realisable income would be £10,200.

(iii) Economic income ex ante

Ye = C1 + (K1t – Kt – 1)

= 25,000 + (142,361 – 139,467) = £27,894 income for 20X1

Assumptions:

(a) The difference of £4,467 between the actual cost of opening capital of £135,000 and its pre-sent value of £139,467 is to be treated as subjective goodwill.

(b) The anticipated drawings represent expected cash flows.

(c) The discount factor does not vary over the timespan.

(d) The cash flows predicted will materialise.

(e) Only the original capital of a present value of £139,467 needs to be maintained.

(f) All the price levels are constant.

Page 34: elliottv10

Barry Elliott and Jamie Elliott: Financial Accounting and Reporting (tenth edition) – Instructor’s Manual

34 © Pearson Education Limited 2006

Workings:

Kt – 1 = capital at 1 January 20X1 ex ante

CF 25,000 25,000 25,000 + 150,000 = ——– = —–—– + —–—– + ––––––––––––––– (1+r)n (1.2) (1.2)2 (1.2)3

= £139,467 K1

t = capital at 31 December 20X1 ex ante

= CF 25,000 25,000 + 150,000 ——– = —–—– + —–—––––––––––

(1+r)n (1.2) (1.2)2

= £142,361

An extension of the tabulated workings might be helpful: Year C1 K1

t Kt –1 Ye W Ye+W C1 – (Ye+W)

Y0–Y1 X1 25,000 142,361 139,467 27,894 27,894 (2,894)

Y1–Y2 X2 35,000 175,000 175,000 35,000 32,639 67,639 (32,639)

Y2–Y3 X3 210,000 175,000 35,000 35,000 175,000

270,000 97,894 130,533 139,467

Workings:

Y1–Y2 20X2 (2) 175,000 175,000 (1) Y2–Y3 20X3 175,000 (2)

35,000 35,000 + 175,000 (1) ––––– + ––––––––––––––– = £175,000

(1.2) (1.2)2

35,000 + 175,000 (2) –––––––––––––––– = £175,000

(1.2)

(iv) Economic income ex post

Ye = C + (Kn – Kn – 1) = 35,000 + (175,000 – 175,000) = £35,000

Page 35: elliottv10

Barry Elliott and Jamie Elliott: Financial Accounting and Reporting (tenth edition) – Instructor’s Manual

35 © Pearson Education Limited 2006

Assumptions:

(a) The difference of £40,000 between the actual cost of opening capital of £135,000 and its present value of £175,000 is to be treated as subjective goodwill.

(b) The discount factor is not subject to change.

(c) Price levels are constant.

(d) Cash flows for years following 20X1 will be as predicted.

(e) All flows occur at the year-end.

Workings:

CF 35,000 35,000 35,000 + 175,000 W1 = –––– = –––––– + –––––– + –––––––––––––– (1+r)n (1.2) (1.2)2 (1.2)3

= £175,000

CF 35,000 35,000 + 175,000 W2 = ––––– = ––––– + ––––––––––––––

(1+r)n (1.2) (1.2)2

= £175,000

A tabular extension of the workings might be helpful:

Year C1 Kn Kn – 1 Ye W Ye+W C1 – (Ye+W)

Y0–Y1 35,000 175,000 W2 175,000 W1 35,000 35,000

Y1–Y2 35,000 175,000 W3 175,000 35,000 35,000

Y2–Y3 35,000 -- 175,000 35,000 35,000

175,000 ______

280,000 105,000 _____ ______

35,000 + 175,000 W3 = –––––––––––––– = £175,000

1.2

(b) Evaluation of the income figures

(i) Accounting income (£36,700)

As an indicator of performance

• Based on actual transactions in this respect it is objective.

• However, it also utilises subjective data (e.g. depreciation) which incorporates an ele-ment of estimation into the results for the year.

Page 36: elliottv10

Barry Elliott and Jamie Elliott: Financial Accounting and Reporting (tenth edition) – Instructor’s Manual

36 © Pearson Education Limited 2006

• If such subjective data are substantial as a proportion of total costs, then the resultant profit or loss would be of reduced reliability.

• Being based on historical cost it can be misleading as an indicator of real profit in times of changing price levels.

• It ignores unrealised capital gains/losses in pursuit of the going concern concept.

• It could be said therefore that on the one hand the figure is incorrect; but

• on the other it is realistic because there is no intention to realise the net assets.

• The balance sheet is not a valuation statement. Consequently, profit expressed as a return on capital employed may be incorrect as an indicator of performance.

As an aid in decision making

• It is historic and history may not be a guide to the future.

• Circumstances of trade, costs and setting prices may be subject to factors not encoun-tered by the results to date.

• However, historical trends over years may be of considerable assistance.

• It does not enable precise comparisons to be made with the return yield of other businesses as historical costs can mean differing values across trade and industry as inflation develops.

• In general, accounting income has a considerable degree of authenticity because

• of its objective nature

• it is traditional and it is understood

• it can be of assistance as an indicator of performance and as an aid in decision mak-ing if it is used as a base figure capable of amendment in the light of:

• subjective content

• a changing price economy and anticipated future commercial trends regarding costs and sales.

(ii) Realisable income (£26,200)

As an indicator of performance

• It avoids the subjective assessment of depreciation and in this sense its measured income can be said to be realistic, but it embraces unrealised capital gains and losses which can be said to be irrelevant when the intention to sell does not exist.

• If the going concern concept is paramount then as an indicator realisable-value-based profit is unrealistic.

• Realisable values are subjective.

As an aid to decision making

• It can be said to equate asset values with opportunity cost which is relevant when consider-ing the going concern versus the cash realisation potential of disposal.

Page 37: elliottv10

Barry Elliott and Jamie Elliott: Financial Accounting and Reporting (tenth edition) – Instructor’s Manual

37 © Pearson Education Limited 2006

(iii) Economic income ex ante (£27,894)

As indicator of performance

• Very subjective figure in terms of future cash flows in respect of amount, timing and dis-count factor. These effects can make it impractical to implement as a system.

• However, it can accommodate inflation by taking account of changing price levels when forecasting the future cash flows

• As an indicator it is predictive and thus windfall gains can occur in this system when antici-pated cash flows are changed by new circumstances.

• It is a guide to prudent conduct as it represents maximum consumption for a defined period without eroding capital.

As an aid to decision making

• By attempting to value a business at different time points it takes account of a strict capital maintenance concept via a time value of money

• thus the possibility of profit distributions being excessive and consequently eroding the capital is restricted

• this is not so with historical cost.

• The adoption of a discounting factor enables cash flows to be adjusted to take account of risk.

• Whilst these two qualities are perhaps too subjective in terms of valuing the entire business entity they can be of considerable assistance when considering investment in the individual asset where choice amongst alternatives or the option of buying or renting exists.

(iv) Economic income ex post (£35,000)

As indicator of performance

• Accuracy is dependent upon the validity of the forecasting cash flows as with the ex ante system.

• However, unlike that model adjustments can be made to past as well as future capital values.

• Thus as an indicator of performance it has the potential to better the ex ante concept.

• By adjusting past as well as future cash flows due to windfall elements it tends to have characteristics akin to traditional accounting. The figure of £35,000 is close to the tradi-tional accounting figure of £36,200.

As an aid to decision making

• Expectations can change over time thereby affecting income and capital calculations.

• Windfall gains and losses can influence the calculations and thus inhibit confidence in the reliability of the measure. Will the profit be £35,000 next year?

• However absolute accuracy is not pretended by the model; as with the ex ante measure its intention is to give guidance only.

Page 38: elliottv10

Barry Elliott and Jamie Elliott: Financial Accounting and Reporting (tenth edition) – Instructor’s Manual

38 © Pearson Education Limited 2006

C H AP T E R 4

Chapter 4: Question 1 – Shower Ltd 20X3 HC CPP/PLA RC CoCoA £ £ £P £P £ £ £ (iv) (i) (ii) (iii) (iv) Sales (8,000 units) 20,000 240/120 40,000 20,000 20,000

Inventory (4,000 units) 4,000 240/100 9,600 HC × 150/100 6,000

Purchase (6,000 units) 9,000 240/120 18,000 HC × 150/150 9,000

13,000 27600 15,000

C Inventory (2,000 units) 3,000 10,000 240/120 6,000 21,600 HC × 150/150 3,000 12,000 10,000

10,000 18,400 8,000 10,000

Sundry expenses 5,000 240/120 10,000 HC × 150/150 5,000 5,000

Depn £6,000/5 1,200 240/100 2,880 HC × 200/100 2,400 0,000

3,800 5,520 600 5,000

Monetary gains Realised holding Price variation

gains adjustments

Loan FA FA

(8,000 × 240) – 8,000 2,400–1,200 1,200 6,000–2,000 (4,000)

(100) 11,200 Inventory Inventory

12,000–10,000 2,000 3,000–5,100 2,100

Page 39: elliottv10

Barry Elliott and Jamie Elliott: Financial Accounting and Reporting (tenth edition) – Instructor’s Manual

39 © Pearson Education Limited 2006

Monetary Losses Unrealised holding Capital Maintenance

Gains

(6,000 × 240) – 6,000 FA

(120) (6,000) 9,600–4,800 4,800 2,000 × 240

Inventory 100

5,100–3,000 2,100 –2,000 (2,800)

PLA Net Income 10,720 10,700 300

Page 40: elliottv10

Barry Elliott and Jamie Elliott: Financial Accounting and Reporting (tenth edition) – Instructor’s Manual

40 © Pearson Education Limited 2006

Balance sheets as at 31 December 20X3

HC CPP/PLA RCA CoCoA

Share capital 2,000 × 240/100 4,800 2,000 2,000

Retained earnings 3,800 PLA 10,720 600 300

Realised holding 3,200

Unrealised holding 6,900

Capital maintenance

reserve 2,000 × 240 2,800

100

–2,000

Loan 8,000 8,000 8,000 8,000

£13,800 £23,520 £20,700 £13,100

Non-current asset 6,000 240/100 14,400 200/100 12,000

Depn 1,200 4,800 240/100 2,880 11,520 200/100 2,400 9,600 NRV 2,000

Inventory 3,000 240/120 6,000 255/150 5,100 NRV 5,100

Cash 6,000 6,000 6,000 6,000

£13,800 £23,520 £20,700 £13,100

Page 41: elliottv10

Barry Elliott and Jamie Elliott: Financial Accounting and Reporting (tenth edition) – Instructor’s Manual

41 © Pearson Education Limited 2006

Chapter 4: Question 2 – Toy plc

(a)

1. Cost of sales adjustment

• This is the amount by which the historic cost of goods sold and charged in the historical cost income statement falls short of the replacement cost of those goods as at the date of sale.

• The CCA model requires all costs, assets, revenues and liabilities to be reported at their cur-rent entry value.

• The COSA is therefore an additional charge to the income statement account intended to bring the historical cost of sales to their replacement cost equivalent.

• The COSA is regarded as a realised holding gain in the sense that it is a gain relating to real-ised assets (i.e. sold inventory) and one made during the asset holding period.

2. Additional depreciation

• Is an amount charged to the income statement to make charge for year equate to that related to the replacement cost.

3. MWCA

• This is the gain or loss from holding monetary working capital.

• If prices are rising MWCA will be a gain if net monetary liabilities are held and a charge if net monetary assets are held.

4. Gearing adjustment

• Is an amount that is based on the proportion of the above charges that accrues to ordinary shareholders because there are lenders who bear a proportion of the charges.

5. Accumulated current cost depreciation

• This is to provide sufficient retention of funds to allow for the replacement of the fixed as-set.

6. Current cost reserve

• This is a revaluation reserve where all holding gains (realised as well as unrealised) are credited in order to avoid the distribution of such gains and therefore reserve enough funds to replace assets at their current replacement costs as they are consumed.

Page 42: elliottv10

Barry Elliott and Jamie Elliott: Financial Accounting and Reporting (tenth edition) – Instructor’s Manual

42 © Pearson Education Limited 2006

• The composition of the CCA reserve account is likely to include realised items as follows:

• debits: backlog depreciation gearing adjustment

• credits: COSA monetary working capital adjustment.

and the following unrealised items:

• closing inventory revaluation increase

• non-current asset revaluation increase.

(b)

It quantifies cost of sales and depreciation after allowing for changing price levels so that the trading results are free of inflationary elements and provide a clearer picture of management performance.

• Resources are maintained by eliminating the risk of disturbing profits out of real capital.

• Time series and inter firm comparisons are more indicative of management performance.

Page 43: elliottv10

Barry Elliott and Jamie Elliott: Financial Accounting and Reporting (tenth edition) – Instructor’s Manual

43 © Pearson Education Limited 2006

Chapter 4: Question 3 – Parkway plc

(a) Monetary working capital

Making and stating assumptions:

• COSA provides for the maintenance of inventory levels in times of inflation.

• There is a view that MWC is also an integral part of daily operating activities and should be treated similarly via a provision out of revenue, from any detrimental impact caused by ris-ing price levels.

• However, a consensus does not exist.

• in that some commentators maintain that MWC is not a part of the operating capital and so should be ignored when considering the operating capital maintenance concept,

• apparently in the belief that investment in such items as debtors is not an essential in-gredient of day-to-day operations.

• Even where critics accept MWC is a part of operating activities, varying views exist as to which assets and liabilities should be included in the MWC calculation. Conflicting views are:

• MWC should embrace monetary assets only; or

• all monetary assets less all monetary liabilities should be taken into account; or

• only short-term monetary liabilities should be accepted into the calculation; that long-term monetary liabilities should be part of the gearing adjustment; or

• even short-term monetary liabilities should be ignored; or

• only monetary assets and liabilities that have been generated by operating activities should be involved and thus these should be segregated from other monetary assets and monetary liabilities.

Usual inclusions:

• In spite of ongoing contentious debate, there is general acceptance to include the following items as part of MWC:

• trade receivables, including prepayments, trade bills receivable and VAT recoverable on trade purchases

• trade payables, including accruals, trade bills payable and VAT payable on turnover; and

• any stock not subject to COSA.

Usual exclusions:

• receivables and trade payables arising from fixed assets sold, bought or under construction or those arising out of any other non-trading activities;

• any cash or bank balances; and

Page 44: elliottv10

Barry Elliott and Jamie Elliott: Financial Accounting and Reporting (tenth edition) – Instructor’s Manual

44 © Pearson Education Limited 2006

• certain investments such as long-term and short-term investments. The former will be treated as fixed assets and the latter as cash and bank balances.

Some critics formulate a case for including all or a portion of liquid resources as part of MWC:

• If cash is essential to support day-to-day ordinary operations (e.g. a retail supermarket), then such cash is part of the MWC.

• Similarly if part of a bank balance or overdraft is subject to temporary but material changes as a reaction to fluctuations in levels of stock, trade debtors, trade purchases or sales then it should be treated as MWC.

• Any surplus will become part of the gearing adjustment.

Taking account of the above scenarios the following MWCA calculation involves the assump-tions stated below and corresponding reasons for making them i.e. that:

• MWC is part of day-to-day ordinary operating activities.

• ‘Trade receivables’ are substantial

• at £60,000 this is almost 50% of the capital invested in fixed assets (£126,000)

• they amount to 63% of inventories (after eliminating an average profit content in debt-ors of 16% of sales i.e. £118,000/738,000 × 100 based on the year-end debtors figure of £60,000 i.e. 84% of £60,000/£80,000 × 100) = 63%.

• If COSA is considered necessary in respect of inventories of £80,000 then so too is MWC in respect of trade receivables, inclusive of profit, of £60,000.

• Total inventories of £80,000 are all subject to COSA.

• Trade payables, being also substantial at £90,000, are deemed essential to the entity’s daily operating activities. Trade payables amount to an average 37 days’ credit i.e. [((£80,000 + 70,000)/2) / £738,000] × 365 during 20X8.

• Short-term investments are not essential to MWC, i.e. they do not constitute a provision of finance for, say, imminent investment in trade receivables as part of a marketing strategy to stimulate sales by increasing terms of credit to customers.

• Cash and bank balances, and any part thereof, are essential to day-to-day ordinary activities.

• The rate of credit given and taken remains unchanged over the period.

• Inventories have been charged out on the basis of FIFO and the inventory price level index is appropriate to the MWCA.

• Inventory movements have been evenly spread throughout the year.

Calculation of MWCA

30.6.20X8 1.X.20X7 Change

Trade receivables 60,000 40,000

Trade payables (90,000) (60,000)

(30,000) (20,000) (10,000)

Page 45: elliottv10

Barry Elliott and Jamie Elliott: Financial Accounting and Reporting (tenth edition) – Instructor’s Manual

45 © Pearson Education Limited 2006

Adjustment to average price levels:

30,000 × 160 – 20,000 × 160

180 140

= 26,667 – 22,857

= Volume change (3,810)

Reduction in MWC 6,190

(b) Critical evaluation of the influence of MWCA

• The concept of a MWCA acknowledges the existence of the interaction of physical assets and monetary assets by allowing for the protection of MWC against erosion by inflation in the same way that COSA protects capital in stocks consumed.

• The provision for additional MWC supplements the provision for extra depreciation and COSA in maintaining the capital substance of the entity.

• The calculation is not over-prudent as it takes cognisance of the protection granted by credit suppliers in their indirect funding of credit customers.

• The inclusion of monetary assets and trade payables within the inflation protection frame-work reduces the risk of an excess dividend being paid. This could threaten the going concern by overlooking the impact of inflation on the monetary working funds.

• The concept recognises the lag between realising a sale and realising the resultant cash.

• Changes in credit periods between that granted to trade receivables and that given by sup-pliers can be affected by inflation in that impact may otherwise remain hidden if the MWCA were not applied.

• However, a point of criticism is that if trade creditors become unstable in terms of credit given and credit received, the MWCA calculation increases in complexity and may not be so readily understood by the users of the accounts.

• A further criticism lies in the determination of any cash floats and bank balance movements deemed to be part of MWC by some business entities. These may be very subjective and, consequently, inaccurate or prone to abuse by the compilers.

Page 46: elliottv10

Barry Elliott and Jamie Elliott: Financial Accounting and Reporting (tenth edition) – Instructor’s Manual

46 © Pearson Education Limited 2006

Chapter 4: Question 4 – Raiders plc

(a) All in £000s

(i) Cost of goods sold

Note: Closing inventory purchased on average on 31 December.

Average index is index at 30.9.X4 i.e. 150

Alternatively calculate the average of indices at 1.4.X4 and 31.3.X5

(138 + 162 ) = 150 ( 2 )

HC ×××× Revaluation = Current ratio cost

Inventory 1.4.X4 9,600 150/133 10,827

Purchases 39,200 150/150 39,200

48,800 50,027

Inventory 31.3.X5 11,300 150/156 10,865

COGS 37,500 39,162

(ii) Inventory figure in balance sheet

Balance sheet value 11,300 162/156 11,735

(iii) Equipment depreciation charge

HC ×××× Revaluation = Current cost ratio 1.4.X4 31.3.X5

Purchased 1.4.X2 16,000 180/145 19,862

1.4.X3 20,000 180/162 22,222

1.4.X4 21,600 180/180 21,600

63,684

× 200/180 70,760

CC depreciation =15% × [63,684 + 70,760] = 10,083

[ 2 ]

Alternatively calculate as: 15% × £70,760 = 10,614

Page 47: elliottv10

Barry Elliott and Jamie Elliott: Financial Accounting and Reporting (tenth edition) – Instructor’s Manual

47 © Pearson Education Limited 2006

(iv) Balance sheet value of equipment

Purchase Current cost ×××× 200 Gross CC Accumulated date @ 1.4.X4 180 depreciation

1.4.X2 19,862 22,069 (45%) 9,931

1.4.X3 22,222 24,691 (30%) 7,407

1.4.X4 21,600 24,000 (15%) 3,600

70,760 20,938

Net balance sheet value = 70,760 – 20,938 = £49,822

(b) Evaluation of incremental informational content

Discuss users and their decisions and how current cost number should improve predic-tions and control. Should also refer to recent empirical evidence and may discuss ongoing controversy within (and outside) ASB.

(c) Consider power of providers; cost; economic companies, etc.

Page 48: elliottv10

Barry Elliott and Jamie Elliott: Financial Accounting and Reporting (tenth edition) – Instructor’s Manual

48 © Pearson Education Limited 2006

Chapter 4: Question 5 – Smith plc

(i) CPP requires the restatement of the income statement and balance sheet in terms of pur-chasing power of money at the end of the accounting period in units of CPP.

It is rather like translating the historic figures into another currency.

• CPP accounts are derived from the historic accounts by applying the general price index, and are issued as supplementary statements aimed at the shareholders.

The intention is to ensure that shareholders’ capital is maintained in terms of general pur-chasing power and distributions would be restrained during a period of general inflation.

• CPP accounts are objective/factual because they are

• based on HC figures updated to year end values;

• as such can be audited.

They show gains and losses on monetary items not incorporated into basic CCA model.

(ii) (a) Restate the income statement in £CPP

CPP income statement for the year ended 31.12.20X8 HC£000 Index CPP£000

Sales 2,000 236/228 2,070

Cost of sales

Inventory 320 236/216 350

Purchases 1,680 236/228 1,739

2,000 2,089

Closing inventory 280 236/232 (285)

1,720 1,804

Gross profit 280 266

Depreciation 20 236/120 39

Admin. expenses 100 236/228 104

120 143

Net profit 160 123

Page 49: elliottv10

Barry Elliott and Jamie Elliott: Financial Accounting and Reporting (tenth edition) – Instructor’s Manual

49 © Pearson Education Limited 2006

(b) Restate the closing balance sheet in £CPP

Balance sheet as at 31.12.20X8 HC£000 Index CPP£000

Non-current assets

Land and buildings 1,180 236/120 2,321

Net current assets

Inventory 280 236/232 284

Trade receivables 160 160

Cash/bank 120 120

560 564

Less Trade payables (140) (140)

420 424

Net total assets 1,600 2,745

Equity 1,600 2,745

(c) Restate the opening balance sheet in £CPP (as at 31.12.20X8 rate)

Balance sheet as at 31.12.20X7 HC£000 Index CPP£000

Non-current assets

Land and buildings (net) 1,200 236/120 2,360

Net current assets

Inventory 320 236/216 350

Trade receivables 80 236/220 86

Cash/bank 40 236/220 43

440 479

Less Trade payables (200) 236/220 (215)

240 264

Net total assets 1,440 2,624

Equity (balancing figure) 1,440 2,624

(d) Calculation of monetary loss as at 31/12/20X8

Equity [balance] at 31.12.20X7 in CPP£000 2,624

Equity [balance] at 31.12.20X8 in CPP£000 2,745

Increase 121

Profit per income statement in CPP£000 123

Monetary loss 2

Page 50: elliottv10

Barry Elliott and Jamie Elliott: Financial Accounting and Reporting (tenth edition) – Instructor’s Manual

50 © Pearson Education Limited 2006

(e) Reconciliation of monetary loss as at 31.12.20X8

HC£000 Index CPP£000 Net monetary liabilities at 31.12.20X7 (80) 236/220 (86)

Increase 220 236/228 228

Net monetary assets at 31.12.20X8 140 142

Monetary loss [140 – 142] 2

Net monetary liabilities are made up as follows:

31.12.20X7 31.12.20X8 £000 £000

Trade receivables 80 160

Bank 40 120

Trade payables (200) (140)

(80) 140

Page 51: elliottv10

Barry Elliott and Jamie Elliott: Financial Accounting and Reporting (tenth edition) – Instructor’s Manual

51 © Pearson Education Limited 2006

Chapter 4: Question 6 – Aspirations Ltd

Income statement for year ended 31 December 20X1, prepared on RCA basis

Sales 868,425 Purchases 520,125

Less: Inventory at 31.12.X1 24,250

495,875

Add: Cost of sales adjustment (W1) 7,717

Adjusted cost of sales 503,592

Adjusted gross profit 364,833

Expenses 95,750

Depreciation (W2) 33,000

128,750

Operating gain 236,083

Balance sheet as at 31st December 20X1 – RCA Basis

Non-current assets Freehold property (W3) 975,000

Depreciation (W3) 9,750

965,250

Office equipment 465,000

Depreciation 23,250

441,750

Current assets

Inventories at replacement cost 24,250

Trade receivables 253,500

Cash 1,090,300

1,368,050

Less: Current Liabilities

Payable within 1 year 116,250

Net current assets 1,251,800

Less: Non-current liabilities

Payable after 1 year 500,00

751,800

2,158,800

Page 52: elliottv10

Barry Elliott and Jamie Elliott: Financial Accounting and Reporting (tenth edition) – Instructor’s Manual

52 © Pearson Education Limited 2006

Issued share capital

1,500,000 ordinary shares at £1 each 1,500,000

Holding gains (W4) 422,717

Retained earnings 236,083

2,158,800

Workings for RCA Model

W1 Cost of sales adjustment (COSA)

HCA Indexed RCA Difference

Initial stock 1.1.X1 34,375 × 130/115 38,859 4,484

Purchases 485,750 × 130/ 130 485,750 –

520,125 524,609 4,484

Closing inventory 31.12.X1 24,250 × 130/ 150 21,017 3,233

495,875 503,592 7,717

The calculation has utilised the device of averaging. The user of an average index assumes that inventory was consumed on average at a price applying midway through the financial period. The increase in the cost of sales due to upward-moving price levels is £7,717. Purchases have been acquired evenly throughout the year, apart from the initial inventory, therefore the histori-cal cost also represents average current cost and thus will not require any amendment. The advantages of averaging are those of speed and convenience.

W2 Depreciation

Depreciation is being based on the year-end replacement cost. HCA Indexed RCA Depn Depn

Property 6,500 × 127/110 7,505

Equipment 18,750 × 145/125 21,750

25,250 29,255

As far as the balance sheet is concerned, however, the cumulative depreciation for one year based on year-end values would still have to be £33,000. The difference of £3,745 (£33,000–29,255) would constitute backlog depreciation for the current year. This is an important aspect of the calculation if average price level movements are used to determine depreciation. At first sight many students find the concept of backlog depreciation for the current year as distinct from previous years more difficult to understand.

Page 53: elliottv10

Barry Elliott and Jamie Elliott: Financial Accounting and Reporting (tenth edition) – Instructor’s Manual

53 © Pearson Education Limited 2006

W3 Revaluation of non-monetary items at balance sheet date

HCA Indexed RCA Difference

Freehold property 650,000 × 165/110 975,000 325,000

Office equipment 375,000 × 155/125 465,000 90,000

1,025,000 1,440,000 415,000

Inventory 24,250 × 145/145 24,250

The index of 145/145 used to convert the inventory to RCA is not strictly correct. Inventories of £24,250 were part of purchases for the year and as such were not bought on the last day of the year. However we have assumed that the inventory was bought in the closing days of the year and is tending towards the specific price level measured at 145. If the inventory had been bought much earlier and the amount was material then it would be necessary to ascertain the index at the date of purchase.

W4 Holding gains:

£ On stocks consumed (W1) 7,717

On stocks carried at the year-end nil

On fixed assets (W3) 415,000

422,717

Income statement for year ended 31.12 20X1, GPP basis £ £

Sales (W5) 952,466

Initial inventory (W6) 43,287

Purchases (W6) 532,758

Less: Inventory 31 December (W6) (24,250)

Cost of sales (W6) 551,795

GPP gross 400,671

Expenses (W7) 105,016

Depreciation (W8) 31,796

136,812

GPP net profit before loss on monetary items 263,859

Less: Loss on monetary items (W10) 142,003 GPP net profit after loss on monetary items 121,856

Page 54: elliottv10

Barry Elliott and Jamie Elliott: Financial Accounting and Reporting (tenth edition) – Instructor’s Manual

54 © Pearson Education Limited 2006

Balance sheet as at 31st December 20X1, GPP Basis Non-current assets: Cost Depn

Freehold property (W9) 818,518 8,185 810,333

Office equipment (W9) 472,222 23,611 448,611

1,290,740 31,796 1,258,944 Current assets:

Inventories at GPP valuation (W6) 24,250

Trade receivables 253,500

Cash 1,090,300

1,368,050

Less: Current Liabilities

Payable within 1 year 116,250

Net current assets 1,251,800

Less: Non-current liablities

Payable after 1 year 500,000

751,800

2,010,744 Issued share capital

1,500,000 ordinary shares fully paid 1,888,888

Retained earnings 121,856

2,010,744

Workings (W): General or current purchasing power model With the GPP model historic pounds must be converted into general purchasing power pounds as at the end of the financial year. Where sales are generated and costs incurred evenly throughout the year, we may convert the historic pounds by using an average general price in-dex. However, where substantial outlays of cash are involved on a particular day, as in the case of fixed assets and initial acquisition of inventories it will be more precise to utilise the index applying at that date, if available.

HCA Adjustment GPP/CPP

W5 Sales 868,425 × 170 952,466

155

W6 Initial inventory acquired 34,375 × 170 43,287

135

Purchases 485,750 × 170 532,758

155

520,125 576,045

Page 55: elliottv10

Barry Elliott and Jamie Elliott: Financial Accounting and Reporting (tenth edition) – Instructor’s Manual

55 © Pearson Education Limited 2006

Closing inventory (24,250) × 170

170 = no change (24,250)

Cost of sales 495,875 551,795

Inventory assumed acquired on or close to December 31

W7 Expenses 95,750 × 170 105,016

155

W8 Depreciation 25,250 × 170 31,796

135

W9 Fixed assets HCA HCA NBV Index CPP CPP cost Depn cost Depn £ £ £ CPP£ CPP£ CPP£

Freehold 650,000 6,500 643,500 × 170/135 818,518 8,185 810,333

Equipment 375,000 18,750 356,250 × 170/135 472,222 23,611 448,611

1,025,000 25,250 999,750 1,290,740 31,796 1,258,944

W10 Gain or loss on monetary items

£ Change in trade receivables during year: 253,500

Change in cash occurring during year:

In hand at 31 December 20X1 1,090,300

Received 1 January 20X1 1,500,000

Less payments – non-current assets (1,025,000)

– inventory (34,375)

In hand at 1 January 20X1 440,625

Change (increase) during year 649,675

Change in payables occurring during year

Trade payables (increase) (116,250)

Other payables – Loans (increase) (500,000)

(616,250)

Change in monetary assets occurring during year 286,925

Page 56: elliottv10

Barry Elliott and Jamie Elliott: Financial Accounting and Reporting (tenth edition) – Instructor’s Manual

56 © Pearson Education Limited 2006

So:

Loss on holding net monetary assets during CPP:

year’s inflation

286,925 × (170 – 155)/155 27,767

Add loss on holding cash during the year

i.e. balance at 1 January was held for full year and excluded

from the above calculation:

440,625 × (170 – 135)/135 114,236

142,003

Income statement for year ended 31.12.20X1 – NRV basis £

Sales 868,425

Purchases 520,125

Less: Inventory at 31.12.X1 24,250

Cost of sales 495,875

Gross profit 372,550

Expenses 95,750

Depreciation (W11) 35,000

130,750

Operating gain 241,800

Holding gain (W12) 18,188 259,988

Balance sheet as at 31 December 20X1 – NRV basis

£ £ Non-current assets

Freehold property (W11) 640,000

Equipment (W11) 350,000

Current assets

Inventories at NRV (W12) 42,438

Trade receivables 253,500

Cash 1,090,300

1,386,238

Less: Current Liabilities

Payable within 1 year 116,250

Net current assets 1,269,988

Page 57: elliottv10

Barry Elliott and Jamie Elliott: Financial Accounting and Reporting (tenth edition) – Instructor’s Manual

57 © Pearson Education Limited 2006

Less: Non-current liabilities

Payable after 1 year 500,000

769,988

1,759,988

Issued share capital

1,500,000 ordinary shares at £1 each 1,500,000

Retained earnings 259,988

1,759,988

W11 Reduction in value of non-current assets at 31 December 20X1

Freehold Equipment Total £ £ £

HCA 650,000 375,000 1,025,000

Less: NRV at 31.12.X1 640,000 350,000 990,000

10,000 25,000 35,000

The reduction in value is treated as depreciation

W12 Holding gain in inventory at 31 December 20X1

NRV = cost + profit content of 75%

£24,250 + 75% of £24,250 42,438

Less: Cost 24,250

18,188

Page 58: elliottv10

Barry Elliott and Jamie Elliott: Financial Accounting and Reporting (tenth edition) – Instructor’s Manual

58 © Pearson Education Limited 2006

C H AP T E R 5

Chapter 5: Question 1 – Membership of FRC, ASB and FRRP

FRC members

The membership of the Council is designed to include wide and balanced representation at the most senior levels of preparers, auditors and users of accounts and others interested in them. The members are:

Chairman

Sir Bryan Nicholson Sir Bryan has been Chairman of the Financial Reporting Council since 2001; Pro-Chancellor and Chair of the Council, Open Uni-versity since 1996; Chairman of Education Development International plc since 2004 and non-executive director of Equi-tas Holdings Ltd since 1996.

Born in 1932, Sir Bryan joined Unilever as a management trainee following National Service as a Second Lieutenant in the Royal Army Service Corps (RASC) and graduated with Second Class Honours from Oriel College, Oxford, where he read Politics, Phi-losophy and Economics (PPE). Having progressed within sales and marketing management at Unilever he moved first to the Jeyes Group as Sales Manager and then to the Remington divi-sion of Sperry Rand in 1964 as Sales Director. In 1966 he became General Manager in Australia and returned to Europe in 1969 as Managing Director for the UK and France. He joined Rank Xerox in 1972 as Director, Operations, Rank Xerox (UK) Limited becoming Chairman in 1979. He also supervised the European Subsidiaries of Rank Xerox.

In 1984 the Government invited him to become Chairman of the Manpower Services Commission (MSC) and he was knighted in 1987 for his work at the MSC. In October 1987 he became Chairman and Chief Executive of the Post Office for five years until the end of December 1992. He was Chairman of BUPA from 1992 to 2001, Chairman of Varity (Europe) Limited from 1992 to 1996 and Chairman of The Cookson Group Plc from 1998 to 2003. Sir Bryan was also Chairman of the Council for National Academic Awards (CNAA) from 1988 to 1991 and of the National Council for Vocational Qualifications (NCVQ) from 1990 until 1993. He was Chancellor of Sheffield Hallam Univer-sity from 1992 to 2001. Sir Bryan was President of the Confederation of British Industry (CBI) from 1994 to 1996. He was a member of the National Economic Development Council (NEDC) from 1985 to 1992. He is a past President of the Oriel

Page 59: elliottv10

Barry Elliott and Jamie Elliott: Financial Accounting and Reporting (tenth edition) – Instructor’s Manual

59 © Pearson Education Limited 2006

Society and in 1989 was elected an Honorary Fellow of Oriel in recognition of his services to the College.

Deputy Chair

Barbara Thomas Barbara Thomas received her BA from the University of Penn-sylvania. She then went on to NYU Law School where she received a JD degree with honors. Thereafter she practised cor-porate and securities law in New York, becoming a partner of Kaye, Scholer, Fierman, Hays & Handler in 1978.

In 1980 she was appointed youngest ever Commissioner of US Securities and Exchange Commission and an informal spokes-man for its accounting division. In 1983 she moved to Hong Kong as the first woman main board director of a London mer-chant bank, Samuel Montagu & Co. Ltd. In 1993 she came to the UK as an executive director of News International plc. She sub-sequently led a buy-in of Scotia Haven Food Group and then of Whitworths Food Group.

Currently she is Chairman of Private Equity Investor plc and Deputy Chairman of Friends Provident plc, as well as a non-executive director of Capital Radio plc and Quintain Estates and Development plc, among others. She is also a Trustee of the Royal Academy of Arts and of The Wallace Collection, and a member of the Governing Body of the School of Oriental and African Studies. In addition, she is Chairman of the Professional Standards Advisory Board of the Institute of Directors.

Directors

Sir John Egan President of the CBI

David Illingworth Chairman of the Consultative Committee of Accountancy Bodies and President of the Institute of Chartered Accountants in Eng-land and Wales

Vacancy Investor Community Representative

Members (ex-officio)

Sir John Bourn KCB Chairman, Professional Oversight Board for Accountancy

Richard Fleck Chairman, Auditing Practices Board

Mike Fogden Chairman, Accountancy Investigation and Discipline Board

Mary Keegan Chairman, Accounting Standards Board

Bill Knight Chairman, Financial Reporting Review Panel

Page 60: elliottv10

Barry Elliott and Jamie Elliott: Financial Accounting and Reporting (tenth edition) – Instructor’s Manual

60 © Pearson Education Limited 2006

Members

Charles Allen-Jones Formerly Senior Partner, Linklaters & Alliance

Mike Barnes Head of Technical Development, Audit Commission

Scott Bell CBE Formerly Group Managing Director, The Standard Life Assur-ance Company

Sir Victor Blank Chairman, GUS plc and Chairman of Trinity Mirror plc

Sir John Bond Group Chairman, HSBC Holdings plc

Martin Broughton Chairman, British American Tobacco plc

Sir David Clementi Chairman, Prudential plc

Don Cruickshank Formerly Chairman of the London Stock Exchange

Michael Foot CBE Managing Director, Deposit Takers & Markets Directorate, Fi-nancial Services Authority

Stephen Haddrill Director General, Fair Markets Group (Government nominee)

Sir Derek Higgs Senior Adviser in the UK, UBS Warburg

Douglas Kerr Group Finance Director, CPL Industries Ltd

Rory Murphy Joint General Secretary, UNIFI

Paul Myners CBE Chairman, Guardian Media Group plc

Richard Pearson Senior Partner, PKF

Colin Perry Chairman, LTE Scientific Ltd

Ian Plaistowe Formerly Chairman, Auditing Practices Board

Sir Nigel Rudd Chairman, Boots Group plc and Pilkington plc

Vincent Sheridan Chief Executive, VHI Healthcare

Sir Robert Smith Chairman, The Weir Group plc

Rosemary Thorne Group Finance Director, Bradford & Bingley plc

Graham Ward Senior Partner, Global Energy and Utilities, Pricewaterhouse-Coopers

Observers

Sir John Bourn KCB Comptroller & Auditor General, National Audit Office

Peter Brierley Head of Domestic Finance Division, Bank of England (Bank of England nominee)

Sir Andrew Likierman Managing Director, Financial Management, Reporting and Au-dit, HM Treasury, and Head of the Government Accountancy Service

Secretary

Ann Wilks CBE

Page 61: elliottv10

Barry Elliott and Jamie Elliott: Financial Accounting and Reporting (tenth edition) – Instructor’s Manual

61 © Pearson Education Limited 2006

ASB members

Membership of the ASB is limited to a maximum of ten, of whom two (the chairman and the technical director) are full-time members and the remainder are part-time members. Appoint-ments to the ASB are made by an Appointments Committee which comprises the FRC chairman and deputy chairman together with three members of council.

Chairman

Mary Keegan On 1 January 2001 Mary Keegan, then head of the Global Corpo-rate Reporting Group of PricewaterhouseCoopers (PwC), succeeded Sir David Tweedie as full-time Chairman of the ASB. She joined Price Waterhouse (now PwC) in 1974, becoming, in 1985, the first woman admitted to partnership as an auditor in the UK firm. In 1991 she took charge of PW’s UK technical function and in 1993 joined the group running the firm’s European audit practice. She formalised the firm’s support for the International Accounting Standards Committee (IASC). She served on the UITF 1993–99 and was a founder-member of IASC’s Standing Interpretations Committee. She served on the Technical Commit-tee of the Hundred Group of Finance Directors 1996–2000. From 1990 she actively contributed to the work of the ICAEW, includ-ing membership of its Council. From 1997 to 2000 she represented the UK accountancy bodies on the Council of the Fédération des Experts Comptables Européens (FEE) and was a vice president of FEE.

Technical Director

Andrew Lennard

Members

Michael Ashley Partner, KPMG

Douglas Flint Group Finance Director, HSBC Holdings plc

Huw Jones Director of Corporate Finance, M&G Investment Management Limited

Roger Marshall Partner, PricewaterhouseCoopers

Isobel Sharp Partner, Deloitte & Touche

John Smith Director of Finance, Property & Business Affairs, British Broad-casting Corporation

Jonathan Symonds Chief Financial Officer, AstraZeneca plc

Page 62: elliottv10

Barry Elliott and Jamie Elliott: Financial Accounting and Reporting (tenth edition) – Instructor’s Manual

62 © Pearson Education Limited 2006

Observers

Allan Cook CBE Member of the European Financial Reporting Advisory Group’s Technical Expert Group

Bernadette Kelly Director, Company Law and Investigations, Department of Trade and Industry

Sir Andrew Likierman Managing Director, Financial Management, Reporting and Au-dit, HM Treasury, and Head of the Government Accountancy Service

Professor Geoffrey Liaison member of the International Accounting

Whittington CBE Standards Board

Secretary

Charles Bridge

FRRP members

Chairman

Bill Knight Bill Knight is a solicitor and a former Chairman of the Law Soci-ety’s Company Law Committee. He was senior partner at Simmons & Simmons until 2001. He is currently Deputy Chair-man of Council at Lloyd's of London and Chairman of the Enforcement Committee of the General Insurance Standards Council.

Deputy Chairman

Ian Brindle Ian Brindle BA Econ FCA retired from PricewaterhouseCoopers on 30 June 2001 having been appointed the Senior Partner of Price Waterhouse in 1991, and the Chairman of Pricewater-houseCoopers on the merger in 1998. Before joining the Accounting Standards Board in 1993 he served as a founder member of the Board's Urgent Issues Task Force. He was previ-ously a member of the Auditing Practices Committee, becoming its Chairman in 1990. He was a member of the Council of the In-stitute of Chartered Accountants in England and Wales from 1994 to 1998.

Members

Rupert Beaumont Formerly Partner, Slaughter and May

Sir John Bourn KCB Comptroller & Auditor General, National Audit Office

Stephen Box Formerly Finance Director, The National Grid Group plc

Page 63: elliottv10

Barry Elliott and Jamie Elliott: Financial Accounting and Reporting (tenth edition) – Instructor’s Manual

63 © Pearson Education Limited 2006

Michael Brindle QC Barrister

Richard Delbridge Formerly Group Chief Financial Officer, NatWest Group

Martin Eadon Partner, Deloitte

John Grieves Formerly Senior Partner, Freshfields

Gordon Hamilton Partner, Deloitte

Andrew Higginson Finance Director, Tesco plc

Robert Hildyard QC Barrister

Nigel Macdonald Formerly Partner, Ernst & Young

David Mallett Formerly Group Head of Finance, Standard Chartered Bank

Ron Paterson Formerly Partner, Ernst & Young

Andrew Popham Partner, PricewaterhouseCoopers

George Rose Finance Director, BAE Systems plc

Rosemary Thome Group Finance Director Bradford & Bingley plc

Tony Wedgwood Formerly Partner, KPMG

Secretary

Ann Wilks CBE

Director, Panel Operations

Carol Page

Page 64: elliottv10

Barry Elliott and Jamie Elliott: Financial Accounting and Reporting (tenth edition) – Instructor’s Manual

64 © Pearson Education Limited 2006

C H AP T E R 6

Chapter 6: Question 1 – Financial Statements from Different Countries

Most companies will begin their accounting policies note with an explanation of the general ac-counting convention (e.g. US GAAP or IASs). The first point that should be considered is whether the overall convention is what might be expected given the origin and/or listing of the company. Secondly, although the companies are based in different countries, they may be using the same overall accounting convention. Students might comment that this is helpful as it al-lows greater comparability. The next stage of the analysis should involve a comparison of the accounting policies for deal-ing with specific accounting items. Things to look out for might include:

• Level of detail – does one country give more than the other? Is this related to the level of regulation or is it down to the individual company?

• Which items are treated in the same way?

• Which items are treated differently?

• What is the effect of the differences?

• Is there a common thread in the differences? For example, might one set of regulations seem to be protecting a particular user group (e.g. creditors) or might there be some other underlying assumption (e.g. earnings should not be volatile)?

Chapter 6: Question 2 – Web Exercise using EDGAR

The approach to this question should probably be similar to that for Question 1. However, this question allows students to measure the relative importance of the differences in policies. Stu-dents will identify the main areas of difference from the numerical reconciliations, but will need to refer to the narrative disclosures to understand the reasons.

This exercise would work well with groups, asking different members of the group to look at particular countries. Tutors may wish to steer students towards particular countries to allow representation of different parts of the world. It is worth remembering that disclosures that sat-isfy the SEC’s requirements need not necessarily be easy for an outsider to follow. If students are studying the book in the order it is presented, then care should be taken that they do not get bogged down with the detail of different methods of accounting at this stage – they will have an opportunity to address this later.

Page 65: elliottv10

Barry Elliott and Jamie Elliott: Financial Accounting and Reporting (tenth edition) – Instructor’s Manual

65 © Pearson Education Limited 2006

Chapter 6: Question 3 – Taxation and Financial Reporting

The chapter refers to the relationship between tax and financial reporting fairly briefly. A full answer to this question would probably require some further reading. Suggestions might in-clude: Haller, A. (1992) ‘The relationship of financial and tax accounting in Germany: a major reason

for accounting disharmony in Europe’, International Journal of Accounting, Vol. 27, pp. 310–23.

Hoogendoorn, M.N. (1996) ‘Accounting and taxation in Europe – a comparative overview’, European Accounting Review, Vol. 5 (Supplement), pp. 783–94.

Lamb, M., Nobes, C.W. and Roberts, A.D. (1998) ‘International variations in the connections between tax and financial reporting’, Accounting and Business Research, Vol. 28 (3), pp. 173–89.

Nobes, C.W. (2003) A Conceptual Framework for the Taxable Income of Businesses and How to Apply It under IFRS, Certified Accountants Educational Trust (this can be downloaded from the ACCA website: www.accaglobal.com).

Aisbitt, S. (2002) ‘Tax and accounting rules: some recent developments’, European Business Review, Vol. 14 (2), pp. 92–7.

The question hinges on whether the tax and commercial accounts are (or will be) based on the same figures (congruence). Some advantages and disadvantages of congruence are set out be-low.

Congruence between tax and commercial accounts

Interest group Advantages Disadvantages

(a) Preparers Only one set of information

required

Tax considerations might

drive commercial decisions

(b) Users Clear relationship between

figures in accounts and tax

charge – no need for de-

ferred tax

Accounts do not necessarily

reflect economic reality –

they are prepared so as to

minimise tax liabilities

(c) Tax authorities Clear information that has

been fully audited

May become involved in ac-

counting issues rather than

concentrating on macroeco-

nomic policy issues

If there is (or will be) a high degree of congruence, then tax authorities need to know the extent to which income under IASs will differ from income under current regulations in order to esti-mate their expected share of that income. Further action may be required (e.g. in terms of grants or changes in tax rates) to achieve policy objectives. In the UK, the Inland Revenue is aiming for transition to IASs to be ‘tax neutral’.

Page 66: elliottv10

Barry Elliott and Jamie Elliott: Financial Accounting and Reporting (tenth edition) – Instructor’s Manual

66 © Pearson Education Limited 2006

In the longer term international bodies (e.g. the EU) may impose the tax base, e.g. consolidated IAS financial statements. However, the financial reporting lobby (e.g. Nobes, 2003) would op-pose that on the basis that 'tax pollution' of financial statements is undesirable due to the differing needs of the user groups (tax authorities versus investors).

Page 67: elliottv10

Barry Elliott and Jamie Elliott: Financial Accounting and Reporting (tenth edition) – Instructor’s Manual

67 © Pearson Education Limited 2006

C H AP T E R 7

Chapter 7: Question 1 – MCRV Ltd

(a) Operations Statement for year ended 31.12.20X8

£ £ Turnover 200 Opening stock at market price 50

Purchases at cost 80

130

Closing stock at market price 66

Charge for goods sold 64 Contribution 136

Salaries 30

Interest paid 9

39

Wealth created by operations 97

(b) Statement of changes in wealth

£

Increase in wealth due to operations 97

Increase in value of fixed assets 40

Decrease in value of long-term loans 20

Realisable increase in net assets 157

Changes in wealth:

• The net assets have increased from 195 to 352.

• Fixed asset values reviewed at net realisable value £270 overall at the end of the year.

• Long-term loan value reviewed at net realisable value £70 at the end of the year due to rise in yields on long-dated stocks.

• Movements are set out in W1 (cash flow) and W2 (worksheet). (c) The value of long-term loans is affected by changes in the prevailing interest rates. In the example the yields have increased. If they had decreased, the value of the loans would have been increased.

Page 68: elliottv10

Barry Elliott and Jamie Elliott: Financial Accounting and Reporting (tenth edition) – Instructor’s Manual

68 © Pearson Education Limited 2006

W1: Cash flow statement £

Cash held at start of year 10

Cash flow from operations *66 Long-term loan finance raised 30

106

Fixed assets acquired 30

Cash held at end of year 76

* MCRV does not prescribe the amount of detail which might be shown here. The conclusions of the feasibility study

were that it would be appropriate to report inflows and outflows in the level of detail shown in the following cash book

summary.

Notes on transactions during year

Cash book: £ £ Cash received during year

– credit customers 190

– long-term loans 30

220

Cash paid during year – credit suppliers 85

– fixed assets 30

– salaries 30

– loan interest 9

154

Net increase in cash included in cash flow statement 66

Journal: £ £ DR Debtors 200

CR Sales 200

DR Stock 80

CR Creditors 80

Credit purchases for year

DR Cost of goods sold 70

CR Stock 70

Goods issued for sales

DR Stock 6

CR Cost of goods sold 6

Uplift in closing stock value from cost to net realisable value

Page 69: elliottv10

Barry Elliott and Jamie Elliott: Financial Accounting and Reporting (tenth edition) – Instructor’s Manual

69 © Pearson Education Limited 2006

W2: Worksheet for information

Assets and

liabilities at

start of year

Cash

transactions

Journal Changes in

wealth

Operations Assets and

liabilities at

end of year

Fixed assets Stock Debtors Cash Creditors Long-term loan Net assets Sales Cost of sales Salaries Interest paid Operations

Dr Cr 200 50 30 10 35 60 195

Dr Cr 30 190 220 154 85 30 30 9

Dr Cr 86 70 200 80 200 70 6

Inc Dec 40 20 157 97

Dr Cr 200 64 30 9 97

Dr Cr 270 66 40 76 30 70 352

290 290

374 374

356 356

157 157

200 200

452 452

(d) The ratio will differ to the extent that NRV differs from historical costs.

Page 70: elliottv10

Barry Elliott and Jamie Elliott: Financial Accounting and Reporting (tenth edition) – Instructor’s Manual

70 © Pearson Education Limited 2006

Chapter 7: Question 2 – Conceptual Framework

(a) Statements of Principles or Financial Reporting which set out the concepts that underlie the preparation and presentation of financial statements for external users have been widely de-veloped. Their primary purpose is to provide a coherent frame of reference for standard setters to use in the development and review of accounting standards. In particular the framework provides a basis for choosing between alternative accounting treatments.

(b) In practice the conceptual framework has provided standard setters with a framework for developing standards rather than providing a frame of reference for practitioners in resolv-ing questions in the absence of a specific promulgated standard. Auditors are under pressure to accept practices which are commercially convenient to the client in the absence of a stan-dard, e.g. selecting favourable revenue recognition criteria, adopting merger accounting where possible, massaging income in times of recession.

Page 71: elliottv10

Barry Elliott and Jamie Elliott: Financial Accounting and Reporting (tenth edition) – Instructor’s Manual

71 © Pearson Education Limited 2006

Chapter 7: Question 3 – Fairness

(a) There is an overriding requirement that financial statements should give a true and fair view of the financial position, performance and financial adaptability of an enterprise. In the UK, the ASB considers it to be a dynamic concept whose content will evolve in response to mat-ters such as advances in accounting and changes in business practice. The Board considers that the evolution of the interpretation of the concept will be influenced over time by the accounting standards and other statements that the Board issues.

It is an important concept in the UK because it allows companies to override statutory re-quirements. In such a case the company is required to include a note to the accounts giving particulars of any such departure, the reasons for it and its effect. The use of the override has been considered at various times by the Financial Reporting Review Panel, e.g.

• FRRP Press Notice 42 – Sutton Harbour The Financial Reporting Review Panel considered the 1995 accounts of Sutton Harbour Holdings plc. The Panel accepted the directors’ justification for their departure from the provisions of Statement of Standard Accounting Practice (SSAP) 4 in the particular circum-stances of the company.

• AIM Group The Financial Reporting Review Panel considered the Report and Accounts of AIM Group plc for the year ended 30 April 1998 and discussed them with the company’s directors. The primary matter at issue was the departure from compliance with Financial Reporting Stan-dard (FRS) 7 Fair Values in Acquisition Accounting and the use of the true and fair override following the company’s acquisition of certain of the assets and business of Hunting plc.

(b) This raises the question of the feasibility of general purpose accounts to satisfy the infor-mation needs of non-equity shareholders. Discussion should embrace the interests of each of the user groups and consider the effectiveness of current measurement systems (HC/CPP/RCA/NRVA) and disclosure requirements e.g. socio-economic information, three bottom lines, environmental reporting.

Page 72: elliottv10

Barry Elliott and Jamie Elliott: Financial Accounting and Reporting (tenth edition) – Instructor’s Manual

72 © Pearson Education Limited 2006

Chapter 7: Question 4 – Control of Standard Setting

There is no unique answer to this question – it may be approached in a number of ways, e.g.

PRO arguments Technical requirements. These are now so complex as transactions have become more complex, e.g. financial instruments, that accountancy international professional firms are the only profes-sional group with competence in many of the areas that will require standards in the future.

Globalisation. National standard setters do not have the breadth that exists within the interna-tional firms.

Accountability requirements. Standards will be set that are feasible based on current expertise and costs.

Liability. Given that the main liability lies with the professional firms it is important that they are actively involved in formulation of standards.

CON arguments Enforcement. The major requirement is for effective enforcement of existing standards which can only be achieved by making the international firms accountable.

Lack of independence. The firms are too closely allied with the client and are therefore inclined to accept measurement and disclosure practices which do not comply with existing international standards.

Investor confidence. This depends on transparency – the existence of standards; their effective enforcement and adequate monitoring of audit performance.

In practice, this requires the active participation of all parties: the companies preparing accounts, the accounting standard setters, the profession and user groups.

Page 73: elliottv10

Barry Elliott and Jamie Elliott: Financial Accounting and Reporting (tenth edition) – Instructor’s Manual

73 © Pearson Education Limited 2006

C H AP T E R 8

Chapter 8: Question 1 – Old NV

(a) Income Statement (internal) for the year ended 31 December 20X1 (€000)

Sales 12,050

Less: returns 350 11,700

Inventory at 1.1. 20X1 825

Purchases 6,263

Carriage on purchases 13

Less: returns (313) 5,963

6,788

Inventory at 31.12.20X1 1,125

5,663

Depreciation of plant 313 5,976

Gross Profit 5,724

Administration:

Wages 738

Administration expenses [286–12] 274

Directors’ remuneration 375

Selling:

Salesmen’s salaries 800

Distribution:

Distribution expenses 290

Depreciation of vehicles 187

Carriage 125

Financial:

Goodwill impairment 177

Audit fee 38

Debenture interest 25

Rent receivable (100) 2,929

2,795

Tax 562

Profit for year 2,233

Page 74: elliottv10

Barry Elliott and Jamie Elliott: Financial Accounting and Reporting (tenth edition) – Instructor’s Manual

74 © Pearson Education Limited 2006

(b) Income statement for publication

Income Statement of Old NV for the year ended 31 December 20X1 €000 €000 Sales 11,700

Cost of sales 5,976

Gross profit 5,724

Distribution costs W1 1,402

Administrative expenses W2 1,602

Other operating income (100)

2,904

Trading profit 2,820

Interest payable 25

Profit on ordinary activities before tax 2,795

Income tax 562

Profit on ordinary activities after tax 2,233

W1 Salesmen’s salaries 800

Distribution expenses 290

Depreciation of vehicles 187

Carriage 125 1,402

W2 Wages 738

Administrative expenses 274

Directors’ remuneration 375

Goodwill impairment 177

Audit fee 38 1,602

There will be a disclosure note as follows:

Profit on ordinary activities after tax is after charging

Goodwill impairment 177

Audit fee 38

Depreciation 500

Directors’ remuneration 375

Page 75: elliottv10

Barry Elliott and Jamie Elliott: Financial Accounting and Reporting (tenth edition) – Instructor’s Manual

75 © Pearson Education Limited 2006

Balance Sheet of Old NV as at 31 December 20X1 €000 €000

Non-current assets

Intangible assets [1062 – 177] 885

Property, plant and equipment Note 1 1,074

Current assets Inventories 1,125

Receivables 3,875

Cash at bank and in hand 1,750

Prepayments 12

6,762

Current liabilities Payables 738

Provision for income tax 562

Accrued charges 63

Dividends proposed 362

1,725

Net current assets 5,037

Total assets less current liabilities 6,996

Non-current liabilities Debentures 250

6,746 Capital and reserves

Ordinary shares of €1 each 3,125

Preference shares of €1 each 625

Share premium 250

Retained earnings Note 2 2,746

6,746

Page 76: elliottv10

Barry Elliott and Jamie Elliott: Financial Accounting and Reporting (tenth edition) – Instructor’s Manual

76 © Pearson Education Limited 2006

Disclosure notes to show make-up of balance sheet items

Note 1: Property, plant and equipment

Property, plant and equipment Motor Plant vehicles Total €000 €000 €000

Cost At 1.1.20X1 1,200 1,125 2,325

Additions 362

Disposals

At 31.12.20X1 1,562 1,125 2,687

Accumulated depreciation At 1.1.20X1 738 375 1,113

Charge for year 313 187 500

At 31.12.20X1 1,051 562 1,613

Net book value At 31.12.20X1 511 563 1,074

At 31.12.20X0 824 750 1,574

Working: Accrued expenses €000

Audit fee 38

Debenture interest 25

Note 2: Movements on reserves €000

Retained earnings at 1.1.20X1 875

Amount transferred from income statement 2,233

Dividends proposed (362)

Balance at 31.12.20X1 2,746

Page 77: elliottv10

Barry Elliott and Jamie Elliott: Financial Accounting and Reporting (tenth edition) – Instructor’s Manual

77 © Pearson Education Limited 2006

Chapter 8: Question 2 – HK Ltd

(a) Income Statement for year ended 30 June 20X1

$000 $000 Turnover 381,600

Cost of sales

Per trial balance 318,979

+ Hire 2,400 + depreciation 799

– Insurance 150 + inventory loss 250 3,299 322,278

Gross profit 59,322

Administration expenses

Per trial balance 9,000 + Directors 562 +

Bad Debt 157 + Auditor remuneration 112 9,831

Distribution costs 35,100 44,931

14,391

Profit on disposal of fixed assets 536

Profit before tax and interest 14,927

Interest payable [454 + 151 tax on interest] 605

14,322

Other operating income 17

Profit before tax 14,339

Income tax at 35% 4,887

Tax on profit on disposal 461 5,348

Profit for the year 8,991

Note: Depreciation consists of Buildings 94 + Plant 619 + Fixtures 86

Balance sheet as at 30 June 20X1

(b) Intangible non-current assets

Goodwill 480 Tangible non-currrent assets Freehold land 2,880 Freehold buildings 4,680 Aggregate depreciation 648 4,032 Plant and machinery 3,096 Aggregate depreciation 1,857 1,239 Fixtures and fittings 864 Aggregate depreciation 259 605 Current assets

Page 78: elliottv10

Barry Elliott and Jamie Elliott: Financial Accounting and Reporting (tenth edition) – Instructor’s Manual

78 © Pearson Education Limited 2006

Inventory [11,794 – 500 obsolescence] 11,294 Receivables [7,263 + 250 inventory sale + 150 insurance] 7,663 Bank 11,561 30,518 Current liabilities Payables 2,591 Dividends 486 Tax [4,887 + 461] 5,348 8,425 Net current assets 22,093 31,329 Non-current liabilities 9% loan 7,200 24,129 Deferred Income – Government grant (see Note) 68 24,061 Capital Ordinary shares 50c each 3,600 9% preference shares of $1 each 5,400 Reserve for increased cost of plant 310 Retained earnings [6,364 + 780 Revaluation now realised – 310 transfer to Increased Cost of Replacement Reserve + 8,991 profit for the year – 1,074 dividends] 14,751 24,061

Note: The grant could be deducted from the cost of the plant under IAS 20.

(c) The usefulness of the non-current asset schedule

(1) The column headings allow the user to see the type of non-current assets owned by the business. This can give helpful initial indications, for example:

Realisability – intangible assets might be more difficult to sell than property.

Appreciation – land is more likely to appreciate than office equipment.

Depreciation – licences are subject to amortisation and possible fall in value due to com-petion

Security – land and buildings are more likely to be accepted as security for loans and over-drafts than intangible assets.

Page 79: elliottv10

Barry Elliott and Jamie Elliott: Financial Accounting and Reporting (tenth edition) – Instructor’s Manual

79 © Pearson Education Limited 2006

(2) The carrying values may be at cost or revaluation. If at cost it may be that the balance sheet gives too low an indication of current market val-ues – this is often an important consideration if existing shareholders are assessing a takeover offer.

(3) The accumulated depreciation figure when related to the cost gives an indication of the age of the assets and possible need for capital outlays to replace with cash flow implications.

(4) Disposals may be an indication that there is replacement occurring which could indicate growth or maintenance of existing capacity. If no replacement then consider implications for future capacity or other reason e.g. change of direction, disposal of non profit making parts of the business.

Page 80: elliottv10

Barry Elliott and Jamie Elliott: Financial Accounting and Reporting (tenth edition) – Instructor’s Manual

80 © Pearson Education Limited 2006

Chapter 8: Question 3 – Basalt plc

(a) Income statement for the year ended 31.12.20X0 (£000)

Turnover (962 – 27 returns) 935

Cost of sales Note 1 460

Gross profit 475

Distribution costs Note 2 218

Administrative expenses Note 3 118

139

Other operating income (i.e. rent receivable) 7

Profit on ordinary activities before tax 146

Tax on profit of ordinary activities 58

Profit on ordinary activities after tax 88

Retained profits brought forward 55

143

Proposed ordinary dividend 75

Retained profits carried forward 68

£000

Note 1: Opening inventory 66

Purchases 500

Carriage inwards 9

Returns out (25)

Closing inventory (90)

460

Note 2: Warehouse wages 101

Salesmen’s salaries 64

Distribution expenses 6

Hire of vehicles 19

Depreciation 28 [7/11 of 20% of £220,000]

218

Note 3: Admin. wages 60

Admin. expenses 10

Directors’ remuneration 30

Auditors’ remuneration 2

Depreciation (4/11) 16

118

Page 81: elliottv10

Barry Elliott and Jamie Elliott: Financial Accounting and Reporting (tenth edition) – Instructor’s Manual

81 © Pearson Education Limited 2006

Balance Sheet as at 31 December 20X0 (£000) Non-current assets Tangible assets

[cost 220 – Dep’n b/f 49 – Dep’n for year 44] 127

Current assets Inventory 90

Trade receivables 326

Cash at bank 62

478

Liabilities

Amounts falling due within 1 year:

Trade payables 66

Other payables [Audit 2 + Corporation tax 58 +

Dividends 75] 135

201

Net current assets 277

Total assets less current liabilities 404 Capital and reserves

Called-up share capital 300

Share premium a/c 20

General reserve 16

Retained earnings 68

404

(b) (i) Directors’ report must deal with certain matters by law, e.g.

• Proposed dividends

• Likely future developments in the company’s business

• Principal activities of the company

• Political and charitable contributions

• Must be consistent with other statements – reviewed by auditors

(ii) Chairman’s report

• May be highly personalised review of the business, its developments and the environ-ment in which it operates

• Not subject to audit

(iii) Auditors’ report expresses an opinion as to whether the financial statements give a ‘true and fair view’.

Page 82: elliottv10

Barry Elliott and Jamie Elliott: Financial Accounting and Reporting (tenth edition) – Instructor’s Manual

82 © Pearson Education Limited 2006

Chapter 8: Question 4 – Raffles Ltd

(a) Income Statement for the year ended 31 December 20X6

$ Revenue 1,628,000

Cost of sales [W1] 1,098,400

Gross profit 529,600

Administration expenses [W2] (71,050)

Distribution expenses (32,800)

Operating profit [2] 425,750

Income from fixed asset investments [5] 6,000

Interest payable and similar charges [6]

[18,000 Deb + 3000] (21,000)

Exceptional costs [7] (150,000)

Profit before tax on ordinary activities 260,750

Tax on ordinary activities [8] 115,750

Profit after tax on ordinary activities 145,000

W1: Cost of sales

$ Per question 1,100,000

Less: Realisable on obsolescent inventory (5,600)

Add: depreciation 14,000

Less: capital item incorrectly posted (10,000)

1,098,400

W2: Administration expenses

Per question 206,300

Less: restructuring expense (150,000)

Less: provision written back (1,500)

Audit: fee 7,000

Goodwill impairment 2,500

Depreciation 6,750

71,050

Page 83: elliottv10

Barry Elliott and Jamie Elliott: Financial Accounting and Reporting (tenth edition) – Instructor’s Manual

83 © Pearson Education Limited 2006

Balance sheet as at 31 December 20X6 Non-current assets Intangible [1] 40,000

Property, plant and equipment [2] 341,250

Investments [3] 130,000

511,250

Current assets

Inventory [156,360 + Inventory obsolescence realisable 5,600] 161,950

Receivables [179,830 + 1500 Provision written back] 181,330

343,280

Current liabilities [4] (210,530)

Net current assets 132,750

Total assets less current liabilities 644,000

Non-current liabilities [5] (180,000)

464,000

Share capital 250,000

Revaluation reserve 25,000

Retained earnings [b/f 98,000 + 91,000] 189,000

464,000

(b) Notes to the income statement

1 Accounting policies General Depreciation Exceptional items

2 Operating profit is stated after charging audit fee 7,000

3 Staff costs Purchasing 6 45,000 Distribution 3 22,500 Administration 1 7,500 10 75,000

4 Directors’ emoluments Chairman nil Highest paid director £19,800 Other directors earn salaries in the range of £15,000 – £20,000 £

5 Income from non-current asset investments 6,000

Page 84: elliottv10

Barry Elliott and Jamie Elliott: Financial Accounting and Reporting (tenth edition) – Instructor’s Manual

84 © Pearson Education Limited 2006

6 Interest payable and similar charges £ Bank interest 3,000 Debenture interest paid and payable 18,000 21,000

7 Exceptional costs Restructuring 150,000 Less: Tax relief (45,000) 105,000

8 Taxation on ordinary activities Tax on profits [165,000 – 45,000 from Note 7] 120,000 Overprovision (4,250) 115,750

9 Dividends £ Interim at 3.6p 18,000 Proposed at 7.2p 36,000 54,000

Notes to balance sheet

Non-current assets

1 Intangible assets – Goodwill

Cost at 1 January 20X6 50,000

Impairment

At 1 January 20X6 7,500

Charge for year included in Administration expenses 2,500

At 31 December 20X6 10,000

Net book value at 31 December 20X6 40,000 Net book value at 1 January 20X6 42,500

2 Property, plant and equipment

Land/ Plant/ Fixtures Total blgs machinery Cost

At 1.1.20X6 225,000 20,000 78,000 323,000

Additions 90,000 90,000

Disposals (24,000) (24,000)

Revaluation 25,000 25,000

At 31.12.20X6 250,000 110,000 54,000 414,000

Page 85: elliottv10

Barry Elliott and Jamie Elliott: Financial Accounting and Reporting (tenth edition) – Instructor’s Manual

85 © Pearson Education Limited 2006

Depreciation

At 1.1.20X6 9,000 16,000 37,500 62,500

Charge 3,000 11,000 6,750 20,750 Disposals (10,500) (10,500)

At 31.12.20X6 12,000 27,000 33,750 72,750

NBV 31.12.20X6 238,000 83,000 20,250 341,250 NBV 1.1. 20X6 220,000 4,000 40,500 260,500

3 Investments

Listed on recognised stock exchange £130,000

Notes relating to Liabilities, Post balance sheet events and Capital

4 Current liabilities

Bank overdraft 12,700

Trade payables 32,830

Taxation 120,000

Dividends 36,000

Accrual – interest 9,000

210,530

5 Non-current liabilities

10% Debentures redeemable 2003 £180,000

(i) Provisions for liabilities – Lawsuit details.

(ii) Post balance sheet events – Details re investment in Diat P’or.

(iii) Share capital

Details of authorized capital

Reserves Revaluation Income

statement

At 1.1.20X6 – 98,000

Movement during year 25,000 145,000

Less dividends (Note 9) (54,000)

At 31.12.20X6 189,000

Page 86: elliottv10

Barry Elliott and Jamie Elliott: Financial Accounting and Reporting (tenth edition) – Instructor’s Manual

86 © Pearson Education Limited 2006

Comments

Provision

• is an amount retained from profit

• provides for a loss or liability likely to be incurred but amount is uncertain

• is shown in balance sheet after creditors or deducted from assets e.g. bad debt provision

Reserve

• is a realised or unrealised gain which has not either legally or at the company’s discretion been distributed as dividends, i.e.

• is retained in the business, e.g. retained profits, share premium, revaluation reserve

• is shown in the balance sheet after share capital

Liability

• is an obligation in the future requiring the transfer of assets, e.g. cash payment or provision of services to other entities

• entails a probable future sacrifice

• may also include the amount owed to the owners of the business

• is reported under current or long-term liabilities

Contingent liability

• is a condition that exists at the balance sheet date where the outcome will be determined by a future uncertain event appears as a note to the balance sheet

Page 87: elliottv10

Barry Elliott and Jamie Elliott: Financial Accounting and Reporting (tenth edition) – Instructor’s Manual

87 © Pearson Education Limited 2006

Chapter 8: Question 5 – Phoenix plc

(a) Income Statement for year ended 30 June 20X7

£000 Revenue 6,465

Cost of sales [4,165 +280 dep] (4,445)

Gross profit 2,020

Distribution cost (669)

Administration expense [1,126 + 31 dep + 415] (1,572)

Operating loss (221)

Exceptional item:

Gain on disposal of associate 75 Dividend received 80

Loss before taxation (66)

Taxation (96)

Loss after taxation (162)

(b) Balance sheet as at 30 June 20X7

Property, plant and equipment 4,159

Investment 365

Current assets Inventory 1,468

Trade receivables 947

Cash at bank 175

Current liabilities (868)

Net current assets 1,722

6,246

Share capital and reserves

Share capital 4,500

Share premium 500

Revaluation reserve 1,170

Retained earnings 76

6,246

Page 88: elliottv10

Barry Elliott and Jamie Elliott: Financial Accounting and Reporting (tenth edition) – Instructor’s Manual

88 © Pearson Education Limited 2006

(c) Statement of Movement of Property, plant and equipment

L&B P&M F&F Total Balance b/f 2,400 1,800 620 4,820 Disposal (150) (150)

Revaluation reserve 160 160

Balance c/f 2,250 1,960 620 4,830

Accumulated depreciation Balance b/f 540 360 900 Revaluation reserve (540) (540)

P&L charge 280 31 311 Balance c/f 280 391 671

WDV at 30.6.20X7 2,250 1,680 22 4,159

Current assets Trade receivables 947

Creditors Trade payables 566

Taxation 122

Dividend proposed 180

868

Balances in revaluation reserve and retained earnings are made up as follows: Revaluation reserve Retained earnings Balance b/f 600 488

Plant and machinery revaluation 700

Transfer on disposal (30) 30

Transfer – additional depreciation (100) 100

Loss for year (162)

Dividends (380)

Balance c/f 1,170 76

(d) Statement of Recognised Gains and Losses

Loss after taxation (162)

Revaluation gain 700

Total recognised gain 538

Page 89: elliottv10

Barry Elliott and Jamie Elliott: Financial Accounting and Reporting (tenth edition) – Instructor’s Manual

89 © Pearson Education Limited 2006

C H AP T E R 9

Chapter 9: Question 1 – Springtime Ltd

(a) Income Statement for the year ended 31 March 20X4

Continuing Discontinued operations operations Total Turnover 30,000 5,000 35,000

Cost of sales 19,000 4,000 23,000

Gross profit 11,000 1,000 12,000

Distribution costs (3,065) (425) (3,490)

Administrative costs (1,225) (15) (1,240)

6,710 560 7,270

Closure costs (350) (350)

Operating profit 6,710 210 6,920 Income from fixed asset investment 1,200

8,120

Taxation [3200 – 200 + 150] 3,150

Profit for the year 4,970

Note: Tax: Income Tax 3,200 – Overprovision 200 + Transfer to deferred tax account 150

Workings

Continuing Discontinued Total Distribution costs Delivery costs 900 300 1,200

Dep’n – vans 40 40

Dep’n – stores equip. 50 50

Storeroom costs 1,000 1,000

Delivery staff 700 700

Directors 75 25 100

Storeroom staff 300 100 400

3,065 425 3,490

Page 90: elliottv10

Barry Elliott and Jamie Elliott: Financial Accounting and Reporting (tenth edition) – Instructor’s Manual

90 © Pearson Education Limited 2006

Admin. costs Audit 30 30

Depreciation – cars 10 10

Office expenses 800 800

Directors 300 300

Office staff 85 15 100

1,225 15 1,240

Note: as allowed under IFRS 5, disclosures are given on the face of the income statement

(b) IFRS 5 has required companies to disclose in detail activities that are discontinued. This disclosure is both numerical and narrative and provides a full explanation of the activities to be discontinued, when the discontinuance should occur and the financial effect of the dis-continuance.

This information is useful to users in enabling them to interpret the future performance of the enterprise and assessing the performance of management over the period. When consid-ering the future performance of an enterprise only the continuing operations should be considered as it is only these that will continue into future periods. The management per-formance can be assessed to some extent by having knowledge of discontinuing activities because the users will be able to judge whether the management decision to discontinue is a good one.

Users can also get benefits from the disclosure in understanding the future strategic direc-tion of the business. By discontinuing activities the management may be refocusing the business towards more core areas and this would be seen through the disclosures.

(c) Reasons why accounting policy notes are important to users.

• The underlying reason for a change might be more important than the change itself, e.g.

a reduction in the life over which assets are depreciated could indicate a threat from technological changes within the sector; or

a structural change, e.g. repositioning in the market.

• Inter-company comparison requires ideally that two companies should have similar account-ing policies, or that their accounting policies should be amended to make them uniform, e.g. restating all non-current assets at current replacement costs.

• This is important when comparing ROCE, and a policy note about the use of modified his-torical accounts, together with reconciliations to historical cost profit required by some national standards (eg FRS 3 in the UK) might improve the interpretation of differences.

Page 91: elliottv10

Barry Elliott and Jamie Elliott: Financial Accounting and Reporting (tenth edition) – Instructor’s Manual

91 © Pearson Education Limited 2006

Chapter 9: Question 2 – Olive A/S

(a) Income Statement for year ended 30 September 20X4

Revenue 3,460

Cost of sales W1 (1,557.1)

Gross profit 1,902.9 Distribution cost W2 (362)

Administration expenses W3 (917.9)

Operating profit 623 Exceptional items: Gain – disposal of fixed assets 6 Dividend received 45

Interest and similar charges (30)

644

Taxation (Sch 1) (197)

Profit for the year 447

(b) Balance Sheet as at 30 September 20X4

Non-current assets Intangible assets 425

Tangible assets (Sch 2) 1,480

Investments 248

Current assets

Inventory [364 + 40] 404

Receivables (Sch 3) 599

Cash and bank 38

Current liabilities (Sch 4) (636)

Net current assets 405 2,558

Non-current liabilities 12% debentures 500

Net capital employed 2,058

Share capital: ordinary shares of £1 each 600

Share premium account 30

Retained earnings (Sch 5) 1,055

Revaluation reserve 373

2,058

Page 92: elliottv10

Barry Elliott and Jamie Elliott: Financial Accounting and Reporting (tenth edition) – Instructor’s Manual

92 © Pearson Education Limited 2006

Schedule 1: Taxation charge

Income tax 185

Underprovision 20X3 (140 – 128) 12

197

Schedule 2: Statement of Movement of Non-current Assets Land and Plant and Fixtures Prepay- Buildings Machinery and fittings ments Total Balance b/f 600 520 80 – 1,200

Revaluation 300 – – – 300

Acquisitions – 320 40 60 420

Disposal – (240) – – (240)

Balance c/f 900 600 120 60 1,680

Balance b/f 80 160 26 – 266

Revaluation (80) – – – (80)

Income charge 15 54 11 – 80 Disposal – (66) – – (66)

Balance c/f 15 148 37 – 200

WDV 30.9.X4 885 452 83 60 1,480 WDV 30.9.X3 520 360 54 – 934

Schedule 3: Receivables Trade receivables 584

Prepaid rent 15

599

Schedule4: Current liabilities Trade payables 296

Debenture interest (3 months) 15

20X3 Income tax 140

20X4 Income tax 185

636

Page 93: elliottv10

Barry Elliott and Jamie Elliott: Financial Accounting and Reporting (tenth edition) – Instructor’s Manual

93 © Pearson Education Limited 2006

Schedule 5: Statement of Movement of Reserves

Share Revaluation Income premium A/c reserve statement Balance b/f 150 – 661

Formation expenses w/off (120) – –

Profit for the year – – 447

Dividend paid – – (60)

Revaluation gain – 380 –

Transfer – extra depreciation – (7) 7

Balance c/f 30 373 1,055

Notes

1 Expenses charged in the year includes the following: Depreciation written off €80,000 Directors’ emoluments €180,000 Directors’ pension €18,000 €198,000 Audit fees and expenses €65,000

2 Company employs 646 persons, of whom 428 work at the factory and the rest at the head office.

3 Land and Buildings were revalued during the year by Messrs XYZ, Chartered Valuers, at open market value on existing use basis and the surplus recorded in a Revaluation Reserve.

4 Administration expenses includes an exceptional item of €60,000 being the underprovision for a claim that arose in a previous year.

Workings

W1 Cost of Sales Inventory on 1.10.20X3 211

Purchases 925

Carriage inwards 162

Depreciation – Building 9

Depreciation – Machinery [18 + 28 + 8] 54

Salaries [55% of 820] 451

Pension cost [10% of 451] 45.1

Heat and light [80% of 80] 64

Inventory 30.9.20X4 (364)

1557.1

Page 94: elliottv10

Barry Elliott and Jamie Elliott: Financial Accounting and Reporting (tenth edition) – Instructor’s Manual

94 © Pearson Education Limited 2006

W2 Distribution cost Advertising 112

Sales commission 92

Bad debts 158

362

W3 Administration expenses Depreciation – Buildings 6

Depreciation – Fixtures and equipment [8 + 3] 11

Underprovision for litigation 60

Salaries 369

Directors’ emoluments 180 549

Pension costs [10% of 549] 54.9

Heat and Light 16

Audit fees and expenses 65

Stationery 28

Other administrative expenses 128

917.9

Page 95: elliottv10

Barry Elliott and Jamie Elliott: Financial Accounting and Reporting (tenth edition) – Instructor’s Manual

95 © Pearson Education Limited 2006

Chapter 9: Question 3 – Cryptic plc

(a)

• Company is wise to depreciate buildings because IAS 16 requires the depreciation of all assets with finite life.

• This can be treated as a change in accounting policy; however, the guidance is not clear. If it is a change of policy, the material amount of backlog depreciation up to 30.6.X3 should be treated as a prior period adjustment, and explained in a note to published accounts.

(b)

• The company is not merely permitted, it is encouraged to undertake a periodical review of its estimate of UEL of fixed assets.

Necessary for avoiding situation where assets already fully written off continue in use and assist in earning income.

• The change in the estimate of UEL is not a change in accounting policy and hence the im-pact of the change would not qualify to be treated as prior period adjustment.

• The written-down value of the machinery on 30.6.20X4 of £288,000 should be written off

over the remainder of its revised UEL (of two years)

on the same policy (straight-line method).

the current year’s charge will be £144,000 and this will be included in the figure of Cost of Sale.

• Since the current year’s charge of £144,000 exceeds the normal charge of £48,000 by a sub-stantial amount, the difference may be reported in a note to accounts as an exceptional item.

(c) Statement of Movement of Property, Plant and Equipment

Land and Plant and Furniture, buildings machinery tools etc. Total £000 £000 £000 £000

Balance b/f 600 480 380 1,460

Acquisitions 150 – – 150

Disposals – – (80) (80)

Balance c/f 750 480 300 1,530

Page 96: elliottv10

Barry Elliott and Jamie Elliott: Financial Accounting and Reporting (tenth edition) – Instructor’s Manual

96 © Pearson Education Limited 2006

Acc. depreciation

Balance b/f – 192 95 287

Prior period adj 64 – – 64 Income statement charge 9 144 18 171 Disposal – – (9) (9)

Balance c/f 73 336 104 513

WDV on 30.6.20X4 677 144 196 1,017

WDV on 30.6.20X3 600 288 285 1,173

(d) Income Statement for the year ended 30 June 20X5

£000 £000 Revenue 2,285.0

Cost of sales (Note 1) (1,466.2)

Gross profit 818.8

Distribution cost (Note 1) ( 60.6)

Administration expenses (Note 1) (281.2)

Operating Profit: 477.0

Dividend income 24.0

Finance cost (24.0)

Unusual items: Results of discontinued operations (192) Fundamental reorganisation (145) Disposal of fixed assets (7) (344.0) 133.0

Taxation (Note 2) (33.0)

Profit for the year 100.0

Balance Sheet as at 30 June 20X5

Non-current assets Property, plant and equipment (Ans c) 1,017

Non-current asset investments 240

Page 97: elliottv10

Barry Elliott and Jamie Elliott: Financial Accounting and Reporting (tenth edition) – Instructor’s Manual

97 © Pearson Education Limited 2006

Current assets Inventory (Sch 3) 590

Receivables 475

Cash and bank balance 29

Current liabilities (Note 3) (497) 597

1,854

Non-current liabilities

Deferred tax (Note 2) (174)

Preference shares @ £1 each (200)

1,480 Share capital

Ordinary shares @ 50p each 1,000

Share premium account 150

Retained earnings 330

1,480 Workings

Cost of sales Distribution Administration Inventory

Raw materials 112

Work-in-progress 76

Finished goods 264

Purchases 1,200

Depreciation

Machinery 144

Buildings 3.6 0.9 4.5

Furniture 18.0

Salaries 288 18 54

Rent 72 18 30

Electricity 3.6 7.2 25.2

Advertising 65

Factory power 48

Stationery 12

Other administration

expenses [468–310] 158

Audit fee 18

Inventory

Raw materials (172)

Work-in-progress (54)

Page 98: elliottv10

Barry Elliott and Jamie Elliott: Financial Accounting and Reporting (tenth edition) – Instructor’s Manual

98 © Pearson Education Limited 2006

Finished Goods (364)

1,621.2 109.1 319.7

Less: discontinued operations (155.0) (48.5) (38.5)

1466.2 60.6 281.2

Notes (1) Discontinued operations

The results of discontinued operations are made up from: ‘000 Revenue 215

Cost of sales (155)

Distribution expenses (48.5)

Administrative expenses (38.5)

Costs of cancelling contracts (165)

(192)

(2) Taxation

Income tax 65,000

Overprovision for 20X4 taxation (21,000)

Deferred tax (11,000)

33,000

Deferred tax Balance b/f 185,000

Taxation for the year (11,000)

174,000

(3) Current liabilities

Audit fee 18

Sales tax [(2,875 × 15/115) – (1,380 × 15 /115) – 165] 30

Trade payables 360

Tax 65

Dividend (charged as interest) 24

557

Page 99: elliottv10

Barry Elliott and Jamie Elliott: Financial Accounting and Reporting (tenth edition) – Instructor’s Manual

99 © Pearson Education Limited 2006

Items to be disclosed

Expenses charged in the year

Expenses charged in the year includes the following:

Depreciation £171,000

Auditor’s remuneration £18,000

Unusual item:

Depreciation charged on machinery includes an exceptional item of £96,000 arising from the revision of estimated useful life of machinery.

Prior period adjustment

The company has decided to depreciate buildings. The effect of this change in policy of £64,000 has been charged against Retained Earnings b/f.

Page 100: elliottv10

Barry Elliott and Jamie Elliott: Financial Accounting and Reporting (tenth edition) – Instructor’s Manual

100 © Pearson Education Limited 2006

Chapter 9: Question 4 – Reporting Financial Performance

(a) Statement of Changes in Equity

The Statement of Changes in Equity is of benefit to both investors and lenders to a company in a number of ways. The statement highlights the way that shareholders’ funds have changed over a period, and also the gains and losses recognised in the financial statements in the period that have not been charged or credited to the income statement.

Investors will use the information in the statement to understand how the financial position of the company has changed. This will help the investors to understand whether the performance of the company has been good or poor.

Investors will also see gains and losses that are not recognised in the income statement. For ex-ample the company could have been holding property that is increasing in value in the period, and this will be seen in the statement if the company has revalued in the period.

Lenders will use the information in the statement to help assess the financial position of the company with a view to lending to it. The recognition of revaluation gains in the statement will help a lender to decide whether a company has a sufficient asset base to give security to loans.

It can be argued that all the information that is in the Statement of Changes in Equity is already available in other disclosures in the financial statements. The statement, however, more clearly presents the information and therefore it is of benefit to users.

Segmental disclosures

Segmental disclosures provide information about the performance and position of an enterprise by reference to its business activities and geographical locations. The disclosures are quite ex-tensive for primary segments and less extensive for secondary segments. The primary segmentation can either be by business activities or geographical location; it is the one that has the biggest impact on business risk.

Investors want to make decisions on whether to buy, sell or hold shares in a company. Investors will need to understand how risky the investment is therefore as the return required will vary depending on the level of risk. Risk can either be financial risk or business risk, and segmental disclosures give information about the business risk that a company faces. If a company oper-ates in a number of different business lines, for example, it will be exposed to less business risk than an enterprise that operates in only one.

Investors can also use segmental disclosures to help to assess the quality of management. The investors will be able to see the performance of management in all of the entity’s trading activi-ties and therefore they will be able to see if the performance is poor in any area. Lenders will use the information in a similar way but to make decisions on whether to lend to the enterprise. The level of business risk to a lender may have an influence on the rate of inter-est that is set by the lender and the other terms of the loan.

Page 101: elliottv10

Barry Elliott and Jamie Elliott: Financial Accounting and Reporting (tenth edition) – Instructor’s Manual

101 © Pearson Education Limited 2006

Discontinued activities

IFRS 5 requires companies to present information about their continuing and discontinued ac-tivities separately. This information will often be presented on the face of the income statement although disclosure in the notes is acceptable.

Users (investors, lenders and other users) like to see information about discontinued activities so that they can try to assess the future performance of the enterprise. When assessing the future performance of an enterprise the discontinuing activities should be ignored as these are not pre-sent in the future. If, for example, it can be seen that the discontinuing activities are the poorly performing activities of an enterprise, the enterprise might be a good investment even though overall its performance is not very good.

Also the separate information about discontinued activities will help assess the performance of management. Users will be able to assess if the management is discontinuing unprofitable or profitable business activities. This will help the users decide whether the company is a good or poor investment decision.

Disclosed information about future discontinuances that the company intends to undertake will also be useful to users in assessing the longer-term future potential. Companies must disclose information about discontinued activities as soon as the operations are classified as held for sale, and this could be before the actual discontinuance occurs.

(b) Bedok Ltd

20X9 financial statements

Note to the accounts

On 1 June 20X9 the board of directors announced a plan to dispose of the clothing manufactur-ing segment as this segment is not in line with the core activities of the company. The company is actively seeking a buyer and hopes to have completed the sale by the end of 20Y0. At 31 De-cember 20X9 the carrying amount of the clothing segment assets was £20 million and the liabilities were £4 million. The clothing division had revenue of £65 million, incurred expenses of £50 million, had an operating profit of £14 million and a tax charge of £5 million on the prof-its for the year ended 31 December 20X9. A provision of £1 million has been made in respect of redundancy costs expected when the division is sold.

Page 102: elliottv10

Barry Elliott and Jamie Elliott: Financial Accounting and Reporting (tenth edition) – Instructor’s Manual

102 © Pearson Education Limited 2006

20Y0 financial statements

Income statement for 20Y0 Discontinued activities 20Y0 20X9

£000 £000

Turnover 40,000 65,000

Expenses (32,000) (50,000)

Redundancy costs (1,000)

Provision for redundancy costs 1,000 (1,000)

Operating profit 8,000 14,000

Profit on disposal of division 2,000

Taxation (3,000) (5,000)

Please note that 20X9 has been restated to present the results as discontinuing activities.

Note to the accounts

On 10 May 20Y0 the board signed an agreement to sell the clothing division for £20 million. This plan had been announced on 1 June 20X9. The company decided to dispose of the division because its activities were inconsistent with the core activities of Bedok Ltd. Bedok Ltd recog-nised a provision of £1 million in 20X9 for the costs of redundancy of employees, and this provision was released in 20Y0. Actual redundancy costs of £1 million were paid. The process of selling the company was completed on 1 July 20Y0 and the assets of the division at this date were £23 million and liabilities were £5 million. A pre-tax profit of £2 million was made on the disposal.

Page 103: elliottv10

Barry Elliott and Jamie Elliott: Financial Accounting and Reporting (tenth edition) – Instructor’s Manual

103 © Pearson Education Limited 2006

Chapter 9: Question 5 – Parnell Ltd

(a) Accounting treatment of items 1–5

Item 1 is an exceptional unusual item

• General rule is to include under format heading to which it relates

In this case distribution costs

No adjustment necessary to the income statement but disclose bad debt by way of note.

Item 2, profit/loss on the sale or termination of an operation should be

• Shown separately on face of the income statement after operating profit and before interest

• Analysed under appropriate heading as continuing or discontinued.

Item 3 enables distinction to be made between continued and discontinued operations

• Improves the comparability of current year with previous and next year.

Item 4 would normally be considered a change of accounting policy and requires

• A depreciation charge of £6m for 2003 and

• A prior year adjustment of £12m in respect of 2001/2002 to be charged against retained profits brought forward.

Item 5 is an exceptional unusual item which should be charged

• As an administrative expense in respect of continuing operations

• No adjustment is required to the income statement but the restructuring costs must be dis-closed by way of note.

(b) Redraft of the income statement for 2003

Continuing Discontinued Total £m £m £m

Sales 463 100 563

Cost of sales 280 30 310

183 70 253

Distribution costs 45 45

Administration expense (W1) 94 00 94

Operating profit 44 70 114

Profit on disposal of asset 10 10

Profit on ordinary activities

before tax 44 80 124

Taxation 45

Retained profit 79

Page 104: elliottv10

Barry Elliott and Jamie Elliott: Financial Accounting and Reporting (tenth edition) – Instructor’s Manual

104 © Pearson Education Limited 2006

Reserves

Retained earnings

£m

At beginning of year 101

Prior year adjustment (12)

89

Transfer from income statement 79

At year-end 168

W1 78 per question + 10 being profit on sale of distribution division to be separately disclosed

+ 6 depreciation on offices.

Page 105: elliottv10

Barry Elliott and Jamie Elliott: Financial Accounting and Reporting (tenth edition) – Instructor’s Manual

105 © Pearson Education Limited 2006

Chapter 9: Question 6 – Related Party Scenarios

(a) IAS 24 para. 9 states that a related party is a party that can exercise significant influence over another party. Significant influence would normally be assumed if a party owns at least 20% of voting rights.

Arthur is therefore presumed to be a related party.

However, Arthur would not appear to be able to influence the financial and operating poli-cies because of the disagreement.

Further enquiry would be required to establish that this has actually been the effect of the disagreement.

(b) Brenda appears to fall within the definition of key management personnel.

Brenda is therefore presumed to be a related party.

Supporting evidence: Key management is defined as a person in a senior position having au-thority or responsibility for directing or controlling the major activities and resources of the reporting entity. Brenda’s ability to take 30% of the turnover with her would be prima facie evidence of such authority.

(c) Donald and Emma through their relationship with Carrie would be related parties of Z Ltd.

Both would be presumed to be related parties to Z Ltd.

However, Donald is an employee and there would be no requirement to disclose emolu-ments.

Supporting evidence: Emma is not an employee and amounts paid to Emma by Z Ltd would be disclosed.

The disclosure required by para.17 of IAS 24 includes:

(i) description of relationship

(ii) description of the transaction

(iii) the amounts involved

(iv) any additional information required to understand the transaction

(v) amounts due at balance sheet date.

(d) This requires consideration of the difference between common control and common influ-ence.

Page 106: elliottv10

Barry Elliott and Jamie Elliott: Financial Accounting and Reporting (tenth edition) – Instructor’s Manual

106 © Pearson Education Limited 2006

• Control brings with it the ability to cause the controlled party to subordinate its separate interests, whereas

• the outcome of the exercise of influence is less certain.

Paragraph 11(a) of IAS 24 states that two entities are not related simply because they have a director in common.

Further enquiry is required to consider whether one or both transacting parties, subject to control and influence from the same source or common influence, have subordinated their own separate interests in entering into that transaction.

Page 107: elliottv10

Barry Elliott and Jamie Elliott: Financial Accounting and Reporting (tenth edition) – Instructor’s Manual

107 © Pearson Education Limited 2006

Chapter 9: Question 7 – Maxpool plc

Sale of factory outlet:

• As Bay plc is an investor owning more than 20% of Ching Ltd it is a related party of Ching Ltd.

• Details of the transaction will have to be included in both sets of financial statements for the financial year ending 31 December 20X0.

• Disclosure is required of any elements of the transactions necessary for an understanding of the financial statements.

In this case it means that, as the factory site was sold to a major investor, the financial statements should note that the price was determined by an independent surveyor.

What is position re Maxpool?

• Although both Maxpool plc and Bay plc have an investment in Ching Ltd this does not by itself make these companies related parties. There would appear to be no related party rela-tionship between them and therefore there will be no disclosure in Maxpool plc’s financial statements.

• If Maxpool and Bay fall within IAS 24 para. 9(a) and there is control or influence over Maxpool Group plc they could be related parties.

Evidence of influence would, for example, be presumed if Maxpool plc persuaded Bay to sell the factory at below market value.

What is effect of changes in shareholdings in 20X1?

• Maxpool plc is a related party of Ching Ltd.

• Maxpool plc is presumed to be a related party of Bay plc as Maxpool plc has a holding of more than 20% as per IAS 24 para. 9(b).

• Bay plc is not necessarily a related party of Ching Ltd because there is no presumption that 10% of shareholders have the requisite level of influence.

although Maxpool plc controls Ching and has influence over Bay plc, the relationship between Bay and Ching would not automatically justify their being treated as related parties of each other.

However, one would have to check whether one party has subordinated its interests to the other.

How is the sale of vehicles treated?

• As regards the disclosure of the purchase of the vehicles by Bay plc, as it appears that Bay and Ching are not related parties there is no disclosure required in the financial statements of either company.

• Although Bay plc is not a related party of Ching Ltd it is an associate of Maxpool plc and therefore a related party of Maxpool plc.

Page 108: elliottv10

Barry Elliott and Jamie Elliott: Financial Accounting and Reporting (tenth edition) – Instructor’s Manual

108 © Pearson Education Limited 2006

Maxpool plc will have to disclose details of the transaction between a group member and Bay plc in the group financial statements.

Page 109: elliottv10

Barry Elliott and Jamie Elliott: Financial Accounting and Reporting (tenth edition) – Instructor’s Manual

109 © Pearson Education Limited 2006

C H AP T E R 1 0

Chapter 10: Question 1 – Distribution Rules

(a) A distribution is every description of distribution of a company’s assets to its members whether in cash or otherwise with the exception of:

• an issue of bonus shares

• redemption of a company’s own shares out of capital

• a reduction in share capital by reducing the outstanding liability on unpaid shares or by paying off paid-up share capital.

• a distribution of assets to shareholders on winding up.

(b) The rules for distribution are made to ensure that losses are properly made good before any dividends are paid ensuring that the capital of the company is not eroded. Before 1980, case law had not made this position clear, thus statute was enacted to clarify the position.

(c) Profits available for distribution are:

• accumulated realised profits less accumulated realised losses in so far as they have not already been distributed or capitalised

• capital and revenue profits are taken together provided they are realised

• unrealised profits are not available for distribution.

The additional rule for a public limited company is that the accumulated realised and unrealised profits must exceed the unrealised and realised losses of a company before a distribution can be made. Net unrealised profits are not distributable.

(d) Other constraint on the company’s ability to distribute may be liquidity i.e. availability of cash to pay a dividend.

Page 110: elliottv10

Barry Elliott and Jamie Elliott: Financial Accounting and Reporting (tenth edition) – Instructor’s Manual

110 © Pearson Education Limited 2006

(e) (i) A and B as private companies

A B £000 £000

Profit and loss account is the only distribution profit 90 –

(i) Add back depreciation charged on revalued amount

allowed by companies specifically for distribution 5 10

(ii) Add adjustment to take account of gain now realised

on sale of property – 15

(iii) Take off contingent loss as it is probable. Should

be accrued for and hence in accordance with

general accounting principles be treated as a realised

loss – (13)

Distributable profits for a private limited company 95 12

(ii) A and B as public companies

A B £000 £000

The fixed asset investment is an unrealised loss

take into account (10) (10)

but offset against unrealised profits 40 35

revaluation reserve

therefore no adjustment

to distributable profits

95 12

Page 111: elliottv10

Barry Elliott and Jamie Elliott: Financial Accounting and Reporting (tenth edition) – Instructor’s Manual

111 © Pearson Education Limited 2006

Chapter 10: Question 2 – V.R. Confident Ltd

(a) In a private company: accumulated realised profits less accumulated realised losses by the balance sheet date.

For a public company: as above but net unrealised losses also need to be deducted.

(b) Maximum distributable profits

£000 £000 Profit after tax – as per accounts 314.0

Less: (c) Stock 90.0

(e) Bad debts 60.0

150.0

Tax saving at 33% 49.5 (100.5)

213.5

Add: (b) Depreciation 60.0

(a) Asset revaluation reserve 90.0

363.5

Less: Profit and loss a/c brought forward (288.0)

75.5

Add: General reserve 160.0

Maximum available for distribution 235.5

Less: Preference share dividend 18.0

Maximum distribution for ordinary shareholders 217.5

Assumptions:

• Research and development – unrealised

• Profit and loss a/c brought forward – all realised

• General reserve – all realised

No difference here if the company was a public company as no accumulated net unrealised losses.

Page 112: elliottv10

Barry Elliott and Jamie Elliott: Financial Accounting and Reporting (tenth edition) – Instructor’s Manual

112 © Pearson Education Limited 2006

Chapter 10: Question 3 – Alpha, Beta and Gamma

(a)(i) Private companies

For a private company the maximum distribution that can be made is

• accumulated realised profits of the company (to the extent that they have not previ-ously been distributed or capitalised) less

• its accumulated realised losses (to the extent that they have not been written off in a reduction or reorganisation of capital).

• Such profits and losses may be revenue or capital in origin.

(ii) Public companies

For a public company the same rules apply subject to the additional restriction that

• IF accumulated unrealised losses exceed accumulated unrealised profits

• THEN the net unrealised loss must be deducted from the amount otherwise avail-able for distribution.

Again profits and losses may be revenue or capital in origin.

(b)(i) Alpha plc

Maximum amount available for distribution at 31 March 20X7:

Realised capital profit 1,600

Realised revenue profit brought forward 800

Realised revenue profit for year 400

2,800

Depreciation relating to revaluation surplus treated

as realised (2,200/50) 44

Maximum distribution 2,844

(ii) Beta plc

Maximum amount available for distribution at 31 March 20X7:

Realised revenue profit brought forward 1,000

Realised revenue loss for year (400)

600

Net unrealised capital loss (revaluation deficit) (400)

Maximum distribution 200

Page 113: elliottv10

Barry Elliott and Jamie Elliott: Financial Accounting and Reporting (tenth edition) – Instructor’s Manual

113 © Pearson Education Limited 2006

(iii) Gamma Ltd

Maximum amount available for distribution at 31 March 20X7:

Realised revenue loss brought forward (3,200)

Realised revenue profit for loss 400

(2,800)

No distribution possible – accumulated realised losses exceed accumulated realised profits.

Page 114: elliottv10

Barry Elliott and Jamie Elliott: Financial Accounting and Reporting (tenth edition) – Instructor’s Manual

114 © Pearson Education Limited 2006

C H AP T E R 1 1

Chapter 11: Question 1 – Distribution – identifying unrealised and realised profits

(a) (i) Share premium is the amount by which the value at which a company issues its shares exceeds their nominal value.

(ii) Section 130(1) of the Companies Act 1985 requires that

‘where a company issues shares at a value that exceeds their nominal value a sum equal to the difference between the issue value and the nominal value must be transferred to a share premium account’.

Three circumstances in which this is necessary are as follows:

1. Where the company issues its shares for a cash consideration and the total amount it expects to receive in cash exceeds the par value stated on the share certificate.

2. Where shares were forfeited and re-issued thereafter, and the total consideration re-ceived on both transactions exceeded the nominal value.

3. Where the company issues shares for a consideration other than cash (e.g. in rela-tion to an acquisition of business or merger) and the fair value of the shares issued exceeds their fair value.

(iii) Company law seeks to protect any balance in the share premium account from abuse, because:

• it is part of the capital paid up by shareholders and

• it forms the creditors’ buffer that, according to company law, ought to be protected, in the interest of those whose interests are at peril on account of the privilege of limited li-ability.

Company law seeks to protect the balance in share premium from abuse by specifically stating the purpose for which alone such a balance may be applied as follows:

• pay up fully paid bonus shares

• write off preliminary expenses

• write off any expenses of any issue of shares or debentures

• write off commission paid or discount allowed on any issue of shares or debentures

• provide for any premium payable on redemption of debentures and provide for any premium payable on redemption of its own shares under the circumstances specified in answer to question (b)(i) below.

Page 115: elliottv10

Barry Elliott and Jamie Elliott: Financial Accounting and Reporting (tenth edition) – Instructor’s Manual

115 © Pearson Education Limited 2006

(b) (i) Depreciation

Depreciation on a fixed asset is a realised expense BUT where the fixed asset has been revalued the depreciation on the revalued portion may be treated as a realised profit which is available for distribution.

(ii) Development Development expenditure capitalised in the balance sheet and subsequently charged to the profit and loss account is a realised loss

• in the year the expense appears in the profit and loss account

• NOT when the cost is incurred.

(iii) Associates The share of profits from an associated company to an investing company which is not included in consolidated financial statements is not considered to be realised profits; only the dividends received are considered realised.

(iv) Profit on disposal The profit on disposal of a fixed asset which has been revalued will be the difference between the proceeds and the valuation of the asset.

• However, the excess of the valuation over net book value will not have been treated as real-ised profit in the year of valuation and so in the year of disposal the whole of the profit (proceeds less net book value) will be treated as realised.

(v) Provisions

• The fact that the provision appears in the balance sheet, and not as a note to the balance sheet concerning a contingent liability, clearly indicates that the directors expect to pay damages.

• This decision is probably based on legal advice and, in accordance with SSAP 2 Disclosure of Accounting Policies, has been provided for in the accounts.

• In these circumstances this would be regarded as a realised loss.

(vi) Revaluation surplus

• Any surplus arising from a revaluation of an asset would be regarded as unrealised.

• It would only be realised when the asset was sold. This is in accordance with SSAP 2 pru-dence concept.

(vii) A provision for bad debts A provision would be realised since it is generally accepted accounting practice to make such provisions in accordance with the prudence concept in SSAP 2.

Page 116: elliottv10

Barry Elliott and Jamie Elliott: Financial Accounting and Reporting (tenth edition) – Instructor’s Manual

116 © Pearson Education Limited 2006

Chapter 11: Question 2 – Smith Family Ltd

To: Directors of Smith Family Ltd From: The Auditor Date: August 20X7 Subject: Report on the Proposed Buyback of Ordinary £1 shares held by Mr Otto Smith

Senior

1. Introduction

1.1 Otto Smith Senior wishes to retire from the business and dispose of his holding of 200,000 ordinary £1 shares. These have been valued by independent valuers at £300,000 and the price is not disputed.

1.2 The purpose of this report is to explain:

• whether the company may make the purchase

• the procedure which must be followed

• the rules relating to private companies in respect of the purchase of shares from capital.

1.3 The report will also include illustrative figures and journal entries for the two financing op-tions (internal resources and a combination of internal resources and a new share issue) and will explain the changes to the balance sheet.

(a) 2. Power to purchase own shares

• The Companies Act 1985 permits the purchase whether or not the shares were issued as re-deemable.

• Power to purchase must also be permitted by Smith Family Ltd’s Articles of Association.

(b) 3. Procedure to be followed

• Off market purchase will, therefore, require specific contract approval by special resolution in general meeting.

• Shares bought in from Otto Senior must be cancelled immediately and not reissued. • Permanent capital (share capital and undistributable reserves, e.g. share premium) generally

required to be preserved by Companies Act 1985.

• A transfer must normally be made from distributable profits to undistributable reserves (capital redemption reserve) equivalent to the difference between the proceeds of the new issue (if any) and the nominal value of the shares bought back.

• The premium on redemption (£100,000 in respect of Otto Senior’s shares) must normally be met from distributable reserves (profit and loss)

• although some limited relief may apply if the buyback is partly or wholly financed by a new share issue and Otto Senior’s shares were originally issued at a premium, as indeed they were.

(c) 4. Redemption from capital – private companies

• Smith Family Ltd, as a private company, may be allowed to reduce its permanent capital (share capital and undistributable reserves) where undistributable reserves are insufficient to finance the buyback. The amount of permissible reduction is found by the formula:

Page 117: elliottv10

Barry Elliott and Jamie Elliott: Financial Accounting and Reporting (tenth edition) – Instructor’s Manual

117 © Pearson Education Limited 2006

Redemption cost less sum of the distributable reserves and the proceeds of any new issue. This is called the permissible capital payment (PCP).

• Purchase must be permitted by the company and a special resolution is needed.

• Statutory declaration of solvency is required by directors that, immediately after the buy-back and within 1 year of the buyback, the company will be able to pay its debts. The directors may become personally liable for the debts.

• Report from company auditors must be attached to the declaration stating their agreement.

• Dissenting shareholders who did not vote can apply to the courts to have the purchase from capital set aside.

(d) Option 1: Purchase from internal resources

Balance sheet after buyback: Before After

£000 Net assets £000

400 Cash 100 (400 – 300)

850 Other sundry assets 850

1,250 950

Financed by:

1,000 Ordinary £1 shares 800 (1,000 – 200)

100 Share premium 100

– Capital redemption reserve 50 (200 NV–150 PCP)

150 Profit and loss –

1,250 950

Workings: Redemption cost 300,000

Less: Distributable reserves (150,000)

Permissible capital payment (PCP) 150,000

Transfer to capital redemption reserve (CRR)

Nominal value of shares redeemed 200,000

Less: PCP (150,000)

Transfer to CRR 50,000

Journal entries: Dr Cr Ordinary £1 shares 200,000

Purchase of ordinary £1 shares 200,000

Write out of shares to be bought back

Page 118: elliottv10

Barry Elliott and Jamie Elliott: Financial Accounting and Reporting (tenth edition) – Instructor’s Manual

118 © Pearson Education Limited 2006

Profit and loss account 100,000

Purchase of ordinary £1 shares 100,000

Premium payable on buyback

Purchase of ordinary £1 shares 300,000

Cash account (or bank) 300,000

Payment to buy back shares

Profit and loss account 50,000

Capital redemption reserve 50,000

Transfer from distributable to undistributable reserves in accordance with the require-

ments of the Companies Act 1985. (Nominal value of shares redeemed 200 less permissible capital payment 150.)

Explanation:

• Distributable reserves (£100,000) are lower than the redemption costs (£300,000), therefore, as a private company, Smith Family Ltd will be permitted to reduce its permanent capital.

• Permissible capital payment (the amount of permitted reduction) is the redemption cost (£300,000) less the distributable reserves (£150,000), that is £150,000 and the permanent capital after the buyback has fallen by this amount from £1,100,000 to £950,000.

• The capital redemption reserve is created because the nominal value of the shares redeemed (£200,000) that is, the amount by which the permanent capital of the company would fall af-ter the buyback, is higher than the permissible capital repayment of £150,000.

• Cash falls by £300,000, the cost of redemption.

• The profit and loss balance (distributable reserves) is wiped out by the premium due on re-demption and the statutory transfer to capital redemption reserve.

• Ordinary shares fall to 800,000 after the buyback.

Page 119: elliottv10

Barry Elliott and Jamie Elliott: Financial Accounting and Reporting (tenth edition) – Instructor’s Manual

119 © Pearson Education Limited 2006

Option 2: Partly from internal resources and partly by a new issue of ordi-nary £l share

Balance sheet after buyback: Before After

£000 Net assets £000

400 Cash 200 (400 – 300 + 100)

850 Other sundry assets 850

1,250 1,050

Financed by:

1,000 Ordinary £1 shares 850 (1,000 – 200 + 50)

100 Share premium 110 (100 + 50 – 40)

– Capital redemption reserve 50

150 Profit and loss 40 (150P – 60 (Prem)

– 50 CRR)

1,250 1,050

Workings: PCP £000 £000

Redemption cost 300

Less: Distributable reserves 150

Proceeds of new issue PCP 100 (250)

PCP 50

Transfer to capital redemption reserve:

Nominal value of shares redeemed 200

Less: PCP 50

Proceeds of new issue (to ordinary

shares and share premium) 100 (150)

Transfer to capital redemption reserve 50

Premium on redemption: treatment

Redemption costs partly financed by new issue and Otto Smith Senior’s shares originally issued

at a premium:

Premium on issue 200,000 @ 20p 40,000

Premium on buyback (300 – 200) 100,000

Proceeds of new issue 100,000

Balance on share premium after new issue (100 + 50) 150,000

∴ Write £40,000 of premium due on buyback off against share premium and write balance of

£60,000 off against profit and losses.

Page 120: elliottv10

Barry Elliott and Jamie Elliott: Financial Accounting and Reporting (tenth edition) – Instructor’s Manual

120 © Pearson Education Limited 2006

Journal entries:

Dr Cr Ordinary £1 shares 200,000

Purchase of ordinary £1 shares 200,000

Write out of shares to be bought back

Profit and loss account 60,000

Share premium 40,000

Purchase of ordinary £1 shares 100,000

Premium payable on buyback

Purchase of ordinary £1 shares 300,000

Cash account (or bank) 300,000

Payment to buy back shares

The following two entries may be combined:

Cash account 100,000

Application and allotment account 100,000

Proceeds of new issue

Application and allotment account 100,000

Ordinary £1 shares 50,000

Share premium 50,000

Allotment at a premium of £1

Profit and loss account 50,000

Capital redemption reserve 50,000

Statutory transfer from profit and loss to capital redemption reserve: nominal value of shares bought back (£200,000) exceeds the combined total of the permissible capital payment (£50,000) and the proceeds of the new issue (£100,000).

Explanation:

• Distributable reserves and proceeds of new issue are lower than the redemption costs there-fore, as a private company, Smith Family Ltd will be permitted to reduce its permanent capital on the buyback.

• Permissible capital payment is £50,000 and the permanent capital would have fallen by this amount except that part of the premium due on buyback may be financed through share premium, and therefore permanent capital has fallen by £90,000, the combined effect of the PCP (£50,000) and the write-off against share premium (£40,000).

Page 121: elliottv10

Barry Elliott and Jamie Elliott: Financial Accounting and Reporting (tenth edition) – Instructor’s Manual

121 © Pearson Education Limited 2006

• Capital redemption reserve is created because the nominal value of the shares redeemed (£200,000) exceeds the combined sum of the PCP (£50,000) and the proceeds of the new is-sue (£100,000).

• Cash falls by £200,000, the net difference between the redemption cost and the proceeds of the new issue.

• Share premium account shows a net increase of £10,000, being the difference between the premium on the new issue of £50,000 and the permitted amount of £40,000 of the premium due on the buyback which has been charged against the share premium.

• Profit and loss falls to £40,000, after charging £60,000 premium due on the buyback and transferring £50,000 to capital redemption reserve.

• Ordinary shares fall to £850,000, showing the net effect of the buyback (£200,000) and the new issue (£50,000).

Conclusions and recommendations

Option 2, partly financed by new share issue, is the more favourable because:

• Company is still left with distributable reserves (£40,000 – profit and loss).

• Share premium, which is a restricted reserve in terms of utilisation, may be used to absorb some of the cost (£40,000) of the premium due on the buyback.

Page 122: elliottv10

Barry Elliott and Jamie Elliott: Financial Accounting and Reporting (tenth edition) – Instructor’s Manual

122 © Pearson Education Limited 2006

Chapter 11: Question 3 – Telin plc

(i) Cash and bank

1/10 Balance 5,450,000 28/10 Redemption of

preference shares 8,480,000

4/10 Debentures 2,340,000

12/10 Ord. shares 6,000,000

Share premium 600,000

31/10 P&L a/c 275,000 31/10 Balance c/d 6,185,000

14,665,000 14,665,000

1/11 Balance b/d 6,185,000

10% debentures

4/10 Bank 2,340,000

31/10 Balance c/d 2,400,000 Deb. discount 60,000

1/11 Balance b/d 2,400,000

Discount on debentures

4/10 Debentures 60,000 6/10 Share premium 60,000

Share premium

6/10 Deb. discount 60,000 1/10 Balance 4,000,000

29/10 Premium on

redemption 160,000 12/10 Cash 600,000

31/10 Balance c/d 4,380,000

4,600,000 4,600,000

1/11 Balance b/d 4,380,000

Profit and loss

6/10 Research exp. 1,400,000 1/10 Balance 4,600,000

29/10 Dividends on

pref. shares 80,000 31/10 Cash (profit) 275,000

29/10 Premium on

redemption 240,000

29/10 Capital redemption

reserve 1,400,000

31/10 Balance c/d 1,755,000

4,875,000 4,875,000

Page 123: elliottv10

Barry Elliott and Jamie Elliott: Financial Accounting and Reporting (tenth edition) – Instructor’s Manual

123 © Pearson Education Limited 2006

Product development costs

1/10 Balance 1,400,000 6/10 P&L a/c 1,400,000

Ordinary share capital 1/10 Balance 12,000,000

12/10 Bank 6,000,000

30/10 (Bonus issue) CRR 900,000

31/10 Balance c/f 18,900,000 18,900,000

1/11 Balance c/d 18,900,000

12% preference share capital

29/10 Redemption of

shares 8,000,000 1/10 Balance 8,000,000

Redemption of preference shares

29/10 Cash 8,480,000 29/10 Pref. shares 8,000,000

Premium on red. 400,000

P&L a/c 80,000

8,480,000 8,480,000

Premium on redemption

29/10 Redemption a/c 400,000 29/10 Share premium 160,000

P&L a/c 240,000

Capital redemption reserve

30/10 Ordinary share capital 29/10 P&L a/c 1,400,000

bonus issue 900,000

31/10 Balance c/d 500,000

1,400,000 1,400,000

1/11 Bal. b/d 500,000

(ii) Balance sheet as at 31 October 20X5

Ordinary share capital 18,900,000 Sundry assets 32,170,000

Capital redemption Cash at bank 6,185,000

reserve 500,000

Share premium 4,380,000

Retained profits 1,755,000

Page 124: elliottv10

Barry Elliott and Jamie Elliott: Financial Accounting and Reporting (tenth edition) – Instructor’s Manual

124 © Pearson Education Limited 2006

10% debentures 2,400,000

Creditors 10,420,000

38,355,000 38,355,000

Note: Advantageous course of action for shareholders is not to reduce distributable profits unless there is no other course of action. Therefore, whenever legally possible, reduction has been made from share premium account.

Bonus issue was made from capital redemption reserve, as this is restricted to bonus issues only whereas share premium can be used for some other purposes also.

(iii) Under the Companies Act 1981:

(a) Premium on redemption of shares can be written off against share premium – maximum allowed being premium received on the issue of shares, which are now being redeemed i.e. 2% of £8,000,000 = £160,000 to share premium.

Balance must be written off against profits.

(b) Transfer to capital redemption reserve is the amount by which the aggregate receipts from specific new issue exceeds the nominal value of shares redeemed.

Nominal value of shares redeemed 8,000,000 Less: total receipts from new issue 6,600,000 To capital redemption reserve 1,400,000 (from distributable profits)

Page 125: elliottv10

Barry Elliott and Jamie Elliott: Financial Accounting and Reporting (tenth edition) – Instructor’s Manual

125 © Pearson Education Limited 2006

Chapter 11: Question 4 – Alpha Ltd

(a) Capital reduction and reorganisation account

£000 £000 7¾% notes 50 Ordinary shares 75

Ordinary shares – reissue 15 Ordinary shares 15

Profit and loss account 177 Preference shares 250

Shares in sub. 55 Freehold property 14

Plant 57 ___

354 354

(b) Balance sheet as on 1 July 20X8

£000 £000 Fixed assets

Tangible assets

Freehold property 55

Plant 22

77

Investment

Shares in subsidiary company 45

Loans 40 85

162

Current assets

Inventory 132

Trade receivables 106

Bank 107

345

Payables: Amounts falling due within one year Trade payables 282

Net current assets 63

225

Payables: Amounts falling due after one year 7¾% notes 200

Total assets less liabilities 25

Ordinary share capital 25

Page 126: elliottv10

Barry Elliott and Jamie Elliott: Financial Accounting and Reporting (tenth edition) – Instructor’s Manual

126 © Pearson Education Limited 2006

Ordinary share capital

£000 £000 Capital reduction 75 Balance b/f 75

Capital reduction 15 Bank 25

Balance c/f 25 Reissue 15

115 115

Balance 25

Bank

OSC 25 Balance b/f 58

7¾% notes 150 Shares in sub. 10

Balance c/f 107

175 175

Balance b/f 107

7¾% notes

Balance c/f 200 Bank 150

Capital reduction 50

200 200

Balance b/f 200

Page 127: elliottv10

Barry Elliott and Jamie Elliott: Financial Accounting and Reporting (tenth edition) – Instructor’s Manual

127 © Pearson Education Limited 2006

Chapter 11: Question 5 – Doxin plc

This question is essentially concerned with the issue and redemption of shares by a plc where there is a trading loss impacting on the cash liquidity position. Part (a) requires students to illustrate the effect on key balance sheet components. Part (b) requires a discussion and evaluation of the effects of applying the Companies Act 1985 capital maintenance rules in circumstances where shares are redeemed partly out of distributable profits.

Opening (i) (iia) (iib) (iii) (iv) (v) Closing BS £000 £000 £000 £000 £000 £000 £000 £000

Ord. shares 800 200 5 1,005

Pref. shares 300 (300) -

Capital red.

Reserve 80 80

Share premium 20 (15) (5) -

Reserves 200 (80) (500) (380)

705

Creditors 400 400

Debentures 400 400

1,700 1,505

Bank 200 220 (315) 360 (500) (35)

Other assets 1,500 1,500

Debenture discount 40 40

1,700 1,505

(i) (a) Premium on redemption

• out of profits or

• lowest of:

• premium received on issue of shares to be redeemed (£75,000)

• balance of share premium account including premium on new issue (£20,000)

• total proceeds of the new issue (£220,000).

(ii) (b) Capital redemption reserve:

• excess of nominal value of shares redeemed over total receipt from new issue (£300,000 – 220,000 = 80,000)

Page 128: elliottv10

Barry Elliott and Jamie Elliott: Financial Accounting and Reporting (tenth edition) – Instructor’s Manual

128 © Pearson Education Limited 2006

Comments on Doxin plc

(a) (i) The issue of 200,000 ordinary shares at a premium of 10p each increases the share capital, the share premium and cash balance; note that the issue must be made within specified time limits if it is to be effective in applying the capital maintenance rules which require a transfer to capital redemption reserve.

(ii) On redemption of the preference shares it is necessary to calculate the extent to which the premium on redemption can be charged to the share premium account, and the transfer, if any, to the capital redemption reserve from distributable profits – in this case from the general reserve £200,000.

The full premium on redemption can be charged to the share premium account which was brought into existence by the replacement issue. The limitation imposed by % pre-mium originally received on the shares does not apply.

The preference shares (300,000) disappear from the balance sheets and the share pre-mium account becomes £5,000 with the bank balance reduced by £315,000.

The transfer from general reserve to CRR is always in excess of nominal value re-deemed over the proceeds of other issue (made specifically for redemption).

(iii) The issue of 7% debentures £400,000 valued at 90 results in a long-term liability of £400,000 and a net increase in the bank balance of £360,000 with discount on deben-tures £40,000.

The Companies Act 1985 is silent on treatment of this item apart from the option to write it off against the share premium account. Write-off over the life of the debenture might be the appropriate treatment.

(iv) The use of the share premium balance £5,000 to cover a bonus issue of ordinary shares is reflected by a transfer to the ordinary share capital account as permitted by the Companies Act 1985.

(v) The trade loss £500,000 incurred in the year is recorded as impacting on the bank balance where it creates an overdraft of £35,000.

(b) The interest of creditors is protected by the creation of the CRR £80,000 which is non-distributable and can only be used to issue bonus shares.

However, because of the use of SPA to cover premium on redemption £15,000 the original capi-tal of £1,100,000 is only maintained up to £1,085,000 capital meaning issued share capital plus undistributable reserves.

The effect of this loophole in capital maintenance regulation could be remedied by an additional transfer from distributable profit to CRR – in this case of £15,000.

Page 129: elliottv10

Barry Elliott and Jamie Elliott: Financial Accounting and Reporting (tenth edition) – Instructor’s Manual

129 © Pearson Education Limited 2006

Chapter 11: Question 6

(a) The advantages of purchasing and cancelling own shares are:

It is a method of returning surplus cash that a company is unable to invest profitably within the company

It is a method of overcoming a problem as when shares are acquired from a dissenting share-holder so as to remove the nuisance value.

It is a method of providing cash as a help to a shareholder in liquidating their shareholding as where shares have been issued to employees as part of a profit sharing scheme and the employee wishes to convert to cash or they are acquired from the estate of a deceased shareholder

It is a possible method of improving the share price if the directors consider the current share price are undervalued – on cancellation each remaining share has a greater interest in the net assets.

It is taken as a means of increasing the earnings per share.

(b) The advantages of purchasing and holding shares in treasury

It provides a company with greater flexibility in managing its share capital

It allows a company to optimise its gearing by buy back rather than by increasing or decreasing its debt

It reduces the cost of raising new capital if the shares are reissued later through a Broker rather than through a more expensive placing or rights issue.

It can stimulate an inactive market particularly if existing shareholders have been finding it dif-ficult to sell their shares

It can lead to an increase in the earnings per share

Treasury shares can be used to satisfy the exercise of employee share options and may be ac-quired at the date the option is granted and held in treasury.

Page 130: elliottv10

Barry Elliott and Jamie Elliott: Financial Accounting and Reporting (tenth edition) – Instructor’s Manual

130 © Pearson Education Limited 2006

C H AP T E R 1 2

Chapter 12: Question 1 – Post Balance Sheet Events

(a) IAS 10 requires that financial statements should be prepared on the basis of conditions ex-isting at the balance sheet date, and therefore lays down the following treatment for events after the balance sheet date.

A material event after the balance sheet date requires changes in the amounts to be included in financial statements where:

• it is an adjusting event (i.e. an event which provides additional evidence of condition exist-ing at the balance sheet date); or

• it indicates that application of the going concern concept to the whole or a material part of the company is not appropriate.

A material event after the balance sheet date should be disclosed where:

• it is a non-adjusting event

i.e. an event which concerns conditions which did not exist at the balance sheet date

of such materiality that its non-disclosure would affect the ability of the users of finan-cial statements to reach a proper understanding of the financial position; or

• it is the reversal or maturity after the year end

of a transaction entered into before the year end

the substance of which was primarily to alter the appearance of the company’s balance sheet.

The purpose of the recommended treatment is:

• To ensure that the financial statements show the true position as it existed at the year end.

• All information available to management at the date that the financial statements are final-ised should be taken into account.

• To the extent that such information relates to the financial year dealt with in the financial statements, they should be adjusted.

• To the extent that material matters relate to the subsequent period the user should be in-formed by way of note so that they are not misled as to the current position of the company. However,

If events subsequent to the year end indicate a need to consider whether the enterprise (or a material part of it) is a going concern they are to be treated as adjusting events

because the use of the going concern basis implies a belief that the company will con-tinue in business for the foreseeable future.

Page 131: elliottv10

Barry Elliott and Jamie Elliott: Financial Accounting and Reporting (tenth edition) – Instructor’s Manual

131 © Pearson Education Limited 2006

(b) (i)

• This fraud is an adjusting event, because it has been discovered that the financial statements will not be correct unless they are adjusted for it.

• The £8,000 should therefore be written off in the profit and loss account so that a true and fair view is shown.

(ii)

• As the agreement to purchase this business relates to the period after the year end it is a non-adjusting event.

Knowledge of it in no way affects the position of the company at 31 December 20X6.

Therefore it should be disclosed by way of note.

(iii)

• The rights issue relates entirely to the period after the year end, therefore it is a non-adjusting event.

• It would be wrong to adjust the company’s share capital and cash at 31 December 20X6 because of a subsequent share issue.

• This should be disclosed by way of note.

(iv)

• This is an adjusting event because it provides evidence of conditions existing at the balance sheet date.

• The potential loss of £9,000 should be written off in the income statement and deducted from debtors in the balance sheet.

Page 132: elliottv10

Barry Elliott and Jamie Elliott: Financial Accounting and Reporting (tenth edition) – Instructor’s Manual

132 © Pearson Education Limited 2006

Chapter 12: Question 2 – SEAS Ltd

(i) The closure of the Garratt factory is a discontinuance of a business segment, as it was clearly a material and separately identifiable component of the company’s business opera-tions. Under IFRS 5, disclose under discontinuing heading.

(ii) The fraud is an adjusting event under IAS 10, and, as it has been discovered, the financial statements will not show a true and fair view unless they are adjusted for it. The $30,000 should therefore be written off in the income statement.

(iii) This is an exceptional item, because it is material but arises from the ordinary activities of the business. It should be charged in arriving at the profit on ordinary activities, and should be disclosed separately.

(iv) The agreement to purchase this business relates to the period after the year end and is there-fore a non-adjusting event under IAS 10, as knowledge of it does not affect the company’s position at 31 March 20X8. However, disclose the information by way of note.

(v) This is also an exceptional item and should be treated as in (iii) above. It is not a prior year adjustment because it arises from a change in trading conditions and not from a change in accounting policy or a fundamental error.

(vi) Under IAS 10 this is an adjusting event as it provides evidence of conditions existing at the balance sheet date. The potential loss of £30,000 should be written off in the income statement and deducted from the debtors in the balance sheet.

(vii)The rights issue relates entirely to the period after the year end, therefore it is a non-adjusting event under IAS 10. This should be disclosed by way of note.

Page 133: elliottv10

Barry Elliott and Jamie Elliott: Financial Accounting and Reporting (tenth edition) – Instructor’s Manual

133 © Pearson Education Limited 2006

Chapter 12: Question 3 – World Wide Nuclear Fuels

(a) Explanation

(i) Need for guidance

Difficulties included:

• Definition – the IASB define provisions as a ‘liability of uncertain timing or amount’.

• Treatment of future operating losses – considered these should be accounted for in the fu-ture.

• Provisions differ from liabilities in that provisions are often subject to disclosure require-ments whereas other creditors are not e.g. statutory requirement to disclose – may however be insufficient detail.

• Adequate level of disclosure of movements is important as these do not go through Income Statement once provision is established.

• Unacceptable practice of big bath provisioning used to absorb expenses incurred in later years.

• Management has been able to control the recognition and timing of movements so that user does not have a clear picture of current year’s performance – smoothing profits.

• There has been inconsistency between the accounting for provisions between different com-panies.

(ii) Recognition

IAS 37 applies Framework approach – provisions are an element of the liabilities and not a separate element of the financial statements. Provisions should therefore be recognised only when:

(i) an enterprise has a present legal or constructive obligation and benefits as a result of past events

(ii) it is probably that an outflow of resources embodying economic benefits will be required to settle the obligation

(iii) a reasonable estimate of the amount required to settle the obligation can be made.

IAS 37 takes a balance sheet perspective by concentrating on liability recognition rather than the recognition of an expense.

Criteria include:

• An obligation exists when the entity has no realistic alternative to making a transfer of eco-nomic benefits – may be legally enforceable or constructive.

• Only recognised if existing at balance sheet date.

• Must have arisen from past events.

• Must exist independently from the company’s future actions.

Page 134: elliottv10

Barry Elliott and Jamie Elliott: Financial Accounting and Reporting (tenth edition) – Instructor’s Manual

134 © Pearson Education Limited 2006

• If avoidable by future actions then no provision is recognised.

• No provision should be recognised for future operating losses.

• A constructive obligation for restructuring only exists when the recognition criteria laid out in IAS 37 are satisfied.

• If an enterprise has a contract which is onerous, the present obligation should be recognised and measured as a provision.

(b) Transactions

Although IAS 37 states that no provision should be made for future operation losses, this does not apply if there is an onerous contract. This contract appears to be onerous and so the provi-sion of $135m should remain in the financial statements.

With regard to the provisions for environmental liabilities, the question is whether this is a con-structive obligation. There is no current obligation but it could be argued that there is a ‘constructive obligation’ to provide for the remedial work because the conduct of the company has created a valid expectation that the company will clean up the environment.

We say ‘could be argued’ because there is no clear answer and it may well be determined by the subjective assessment of the directors and auditors as to whether there is a ‘constructive obliga-tion’. The example 2B in IAS 37 would support making a provision.

Page 135: elliottv10

Barry Elliott and Jamie Elliott: Financial Accounting and Reporting (tenth edition) – Instructor’s Manual

135 © Pearson Education Limited 2006

Chapter 12: Question 4

The proposal to include the £62.5m and £40m in sales and cost of sales is incorrect. If it were a genuine disposal it should have been treated as the disposal of a non-current asset with the profit included in the income statement, separately disclosed if material.

The substance of this transaction is, however, a secured loan in that the company is going to have the continued use of the equipment for the whole of its economic life.

There should be a charge of 10% in the Income Statement for the three months from 1 October 20x5. In the balance sheet the asset will continue to be shown as a non-current asset and a year’s depreciation charged. There will also be a liability for the amount of the loan less any capital repayment that has been made.

Page 136: elliottv10

Barry Elliott and Jamie Elliott: Financial Accounting and Reporting (tenth edition) – Instructor’s Manual

136 © Pearson Education Limited 2006

C H AP T E R 1 3

Chapter 13: Question 1 – DDB AG

Issue of deep discount bond

Charges to income statement and carrying value in BS shown in tabular form

(i) (ii) (iii) Cash Finance charge

Carrying value flows to IS in BS £000 £000 £000 At 1 Apr 1 (2,500–125–100) (2,275) –2,275.000

At 31 Mar 1 (10% of 2,500) (2,250 (12.5% × 2,275) 284.375 –2,309.375

At 31 Mar 2 (10% of 2,500) (2,250 (12.5% × 2,309.375) 288.671 –2,348.046

At 31 Mar 3 (10% of 2,500) (2,250 (12.5% × 2,348.046) 293.506 –2,391.552

At 31 Mar 4 (10% of 2,500) (2,250 (12.5% × 2,391.552) 298.944 –2,440.496

At 31 Mar 5 (2,500 +10% of 2500) (2,750 (12.5% × 2,440.496) 305.062 –

Adj.* 4.442

Net cash flow 1,475 1,475.000

*Adjustment necessitated by rounding of implicit rate to 12.5%

Workings

Implicit rate has been determined by interpolation via formula

t = n Σ = t = 1

The initial cost of 2,275,000 is deducted to arrive at the net present value.

Using 13%

t = n Σ = t = 1

= 221,239 + 195,787 + 173,263 + 153,330 + 1,492,590 – 2,275,000 = –38,791

then using 12%

t = n Σ =

At (1+r)t

250 + 250 + 250 + 250 + 2,750 – 2,275,000 1.131 1.132 1.133 1.134 1.135

223,214 + 199,298 + 177,945 + 158,880 + 1,560,424 – 2,275,000 = 44,761

Page 137: elliottv10

Barry Elliott and Jamie Elliott: Financial Accounting and Reporting (tenth edition) – Instructor’s Manual

137 © Pearson Education Limited 2006

[( ])t = 1

Implicit rate = 12% + 44,761 (44,761 + 38,791) × 1% = 12.536 say = 12.5%

Page 138: elliottv10

Barry Elliott and Jamie Elliott: Financial Accounting and Reporting (tenth edition) – Instructor’s Manual

138 © Pearson Education Limited 2006

)(

Chapter 13: Question 2 – RPS plc

Redemption of preference shares

The treatment of the finance cost of preference shares follows the pattern of loan debt. IAS 32 requires that redeemable preference shares are presented and treated as debt instruments because they are in substance debt.

Calculation of finance costs and outstanding principal sum (i) (ii) (iii) Cash Finance

Balance flows charge £000 £000 £000

At 1 Oct 1 (100 – 50) (950) – 950

At 30 Sept 1 (Div 5% 1,000) 50 (6.2% × 950) 58.9 958.9

At 30 Sept 2 (Div 5% 1,000) 50 (6.2% × 958.9) 59.5 968.4

At 30 Sept 3 (Div 5% 1,000) 50 (6.2% × 968.4) 60.0 978.4

At 30 Sept 4 (Div 5% 1,000) 50 (6.2% × 978.4) 60.7 989.1

At 30 Sept 5 (1000+div of 5% 1,000)

1050 (6.2% × 989.1) 61.3

Adj.* (0.4)

Net cash flow 300 300.00

*Adjustment caused by rounding in determining implicit rate of 6.2%, namely

t = n Σ = t = 1

For interest, using 6%

t = n Σ = t = 1

= 47.2 + 44.5 + 41.98 + 39.60 + 784.6 – 950 = 7.88

then 7% =

t = n Σ = t = 1

= 46.7 + 43.7 + 40.8 + 38.1 + 748.6 – 950 = 32.1

–1 = 0

50 + 50 + 50 + 50 + 1050 – 950 1.061 1.062 1.063 1.064 1.065

50 + 50 + 50 + 50 + 1050 – 950 1.071 1.072 1.073 1.074 1.075

At (1+r)t

Page 139: elliottv10

Barry Elliott and Jamie Elliott: Financial Accounting and Reporting (tenth edition) – Instructor’s Manual

139 © Pearson Education Limited 2006

Interpolation gives rate of 6% + 7.88 × 1% = 6.2% 39.98

Treatment of total finance costs through the life-span of the capital instrument

IAS 32 stipulates that the finance costs of redeemable preference shares are to be shown in the income statement but separately after interest.

Income Statement for year ended 30 September (extracts) Years 1 2 3 4 5

£000 £000 £000 £000 £000 Interest

Finance cost on redeemable

preference shares 58.9 59.5 60.0 60.7 60.9

The balance sheet extracts reveal the impact of the IAS regarding liabilities as follows:

Balance Sheet as at 30 September (extracts)

Years 1 2 3 4 5 £000 £000 £000 £000 £000 Long-term liabilities:

Redeemable preference shares 958.9 968.4 978.4 989.1 –

Page 140: elliottv10

Barry Elliott and Jamie Elliott: Financial Accounting and Reporting (tenth edition) – Instructor’s Manual

140 © Pearson Education Limited 2006

Chapter 13: Question 3 – Little Raven

(a) The considerations involved in deciding how to account for the is-sue:

• the issue is made at a substantial discount

• the coupon rate is significantly below market rates

• adopting substance over form, the discount is effectively rolled-up interest and should be accounted for over the period of the borrowing

• the balance sheet should report the obligation to redeem at par and the income statement should report the true cost of the borrowing.

If the borrowing was accumulated for:

(i) As per the question:

DR Cash 4,000

CR Debt 4,000

and each year, DR Income statement 300

CR Cash 300

neither the obligation to repay nor the true cost of the borrowing would be fairly reported.

(ii) Taking advantage of the legal point (available in some countries) that permits discount on issue to be debited to share premium account, the debt could be reported as follows:

DR Cash 4,000

DR Share premium a/c 1,000

CR Debt 5,000

and, each year DR Income statement 300

CR Cash 300

in which case the amount of debt would be fairly reported but not the true cost of the debt.

(iii) Alternatively, DR Cash 4,000

DR Unamortised discount 1,000

CR Debt 5,000

And, each year, DR Income statement 300

CR Cash 300

DR Income statement X

CR Unamortised discount

with amortisation of discount on an appropriate basis over period of debenture.

Page 141: elliottv10

Barry Elliott and Jamie Elliott: Financial Accounting and Reporting (tenth edition) – Instructor’s Manual

141 © Pearson Education Limited 2006

At each year-end the debt would be reported as £5,000 less unamortised discount. Such ac-counting achieves the objective of reporting the actual amount repayable and the true cost of the debt but is not the approach adopted by IAS 32.

(iv) Under IAS 32 the approach would be:

• on issue date DR Cash X

CR Debt X

with the net proceeds of issue.

• determine finance costs as total amounts repayable (interest plus redemption) less net pro-ceeds of issue

• allocate finance costs to each period at a constant rate on the carrying amount of the debt by

DR Income statement X

CR Debt X

DR Debt X

CR Cash X

with amounts paid in each period

(b) Carrying Finance Carrying

Period amount at cost Payments amount at y/e beginning (11.476%) end

£000 £000 £000 £000 30.9.X2 4,000 459 (300) 4,159

30.9.X3 4,159 477 (300) 4,336

30.9.X4 4,336 498 (300) 4,534

30.9.X5 4,534 520 (300) 4,754

30.9.X6 4,754 546 (300 + 5,000) –

2,500

(300 x 5 = 1,500 + 5,000 = 6,500 – 4,000 = 2,500)

Revised income statements for the year ended 30 September 20X5 20X4 (restated)

Turnover 6,700 6,300

Cost of sales (3,025) (2,900)

Gross profit 3,675 3,400

Overheads (600) (550)

Interest payable – debenture (520) (498)

– other (75) (50)

Page 142: elliottv10

Barry Elliott and Jamie Elliott: Financial Accounting and Reporting (tenth edition) – Instructor’s Manual

142 © Pearson Education Limited 2006

Profit for the financial year 2,480 2,302

Retained profit brought forward,

as previously stated 4,300 1,800

Prior year’s adjustment (336)

[159 + 177]

[159 + 177 + 208] (544)

Retained profit brought forward

restated 3,756 1,464

Retained profit, carried forward 6,236 3,766

Page 143: elliottv10

Barry Elliott and Jamie Elliott: Financial Accounting and Reporting (tenth edition) – Instructor’s Manual

143 © Pearson Education Limited 2006

C H AP T E R 1 4

Chapter 14: Question 1 – Kathryn

(a) The original IAS 19 approach to defined benefit pension schemes needed to be reviewed for a number of reasons as follows:

Misleading balance sheet

The original approach to the valuation of the asset or liability for pensions in the balance sheet was potentially misleading to users of the accounts, and did not follow the statement of principles. The income statement charge was ‘smoothed’ out across all the years of ser-vice of the employees. The concept that was being followed in doing this was the accruals concept. Problems for presentation rose when surplus or deficits arose on the pension scheme. For example suppose a company had:

Normal contributions £5m per annum Surplus £10m Average remaining working life of staff 5 years

The actuary recommended a two-year contribution holiday.

The annual income statement account charge would be £3m (the surplus of £10m has been spread over the remaining working lives of 5 years), but in the balance sheet after one year would be a liability of £3m. This liability would grow to £6m in the second year, and would only return to nil by the end of year 5. This balance sheet liability could be understood by users of the financial statements to mean that the company owed the pension fund money. In fact if anything the company had overpaid into the pension scheme.

This approach to pension accounting does not meet the Framework Document in two ways. Firstly the framework prioritises the balance sheet over the income statement whereas the original IAS 19 made the income statement the key statement; and secondly the balance sheet asset or liability does not meet the definition of the item. In the above example for in-stance the liability that is created is not an obligation to transfer economic benefits as a result of past transactions or events.

Internationally inconsistent

The original version of IAS 19 was out of line with the approaches in US GAAP. The re-vised version of the standard is nearer to the requirements to US GAAP as it follows similar valuation principles for assets and obligations, although variations still exist in recognition of gains and losses.

Valuation of pension fund assets and liabilities

The original IAS 19 did not use valuation principles for assets and liabilities that were inter-nationally consistent or the most realistic methods available. Assets were valued at actuarial

Page 144: elliottv10

Barry Elliott and Jamie Elliott: Financial Accounting and Reporting (tenth edition) – Instructor’s Manual

144 © Pearson Education Limited 2006

value as opposed to a market value and liabilities of the pension fund were discounted at the expected rate of return on assets, not a realistic discount rate for liabilities.

The amended standard has addressed this by requiring pension scheme assets to be meas-ured at market values and liabilities to be valued using the ‘projected unit credit method’ discounted at an appropriate corporate bond rate.

(b) Income statement

Operating cost

£000 £000 Pension cost – current service cost (W2) (600)

Financing cost

Expected return on assets (W1) 1,155 Interest cost (W2) (1,020) Net return 135

[Under IAS 19 it is not necessary to include the income statement income or expense as op-erating and finance costs, it would all be acceptable under operating costs, however this split is appropriate given the nature of the income and expense items.]

Balance sheet

Pension liability Present value of obligations (10,900) Market value of assets 10,700 (200)

Statement of movement in equity

Actuarial gain on the obligations (W2) 120 Actuarial loss of the assets (W1) (855) Loss in equity (735)

WORKINGS

W1 Assets of the pension fund £000

Market value of assets as at 1 May 2000 10,500 Expected return on assets – 11% 1,155 Contributions 700 Benefits paid (800) Actuarial gains (losses) – bal. fig. (855) Market value of assets as at 30 April 2001 10,700

Page 145: elliottv10

Barry Elliott and Jamie Elliott: Financial Accounting and Reporting (tenth edition) – Instructor’s Manual

145 © Pearson Education Limited 2006

W2 Obligations of the pension fund

Present value of the obligations as at 1 May 2000 10,200 Interest cost – 10% 1,020 Current service cost 600 Benefits paid (800) Actuarial (gains) losses – bal. fig. (120) Present value of obligations as at 30 April 2001 10,900

Page 146: elliottv10

Barry Elliott and Jamie Elliott: Financial Accounting and Reporting (tenth edition) – Instructor’s Manual

146 © Pearson Education Limited 2006

Chapter 14: Question 2 – Donna

In 2004 the IASB amended IAS 19 Employee Benefits and introduced an extra treatment for actuarial gains and losses. Under the previous version of the standard actuarial gains and losses had to be recognised in the income statement, but only if they exceeded the greater of 10% of the present value of the obligations or 10% of the assets.

In December 2004 an amendment to the standard was issued however that gave an extra treatment. If a company chooses immediate recognition of all gains and losses they can recognise the gains and losses outside the income statement, which would mean in equity. The impact of these approaches is shown below:

Workings

Change in the obligation

2002 2003 2004 Present value of obligation, 1 January 3,500 3,600 3,500 Interest cost 210 180 140 Current service cost 150 160 170 Benefits paid (140) (150) (130) Actuarial (gain)/loss on obligation (120) (290) (480) (balancing figure) Present value of obligation, 31 December 3,600 3,500 3,200

Change in the assets

Fair value of plan assets, 1 January 3,200 3,400 3,600 Expected return on plan assets 320 306 288 Contributions 120 120 130 Benefits paid (140) (150) (130) Actuarial gain/(loss) on plan assets (balancing figure) (100) (76) (288) Fair value of plan assets, 31 December 3,400 3,600 3,600

10% corridor

The limits of the ‘10% corridor’ need to be calculated in order to establish whether actuarial gains or losses exceed the corridor limit and therefore need recognising in the income statement. Actuarial gains and losses are recognised in the income statement if they exceed the 10% corridor, and they are recognised by being amortised over the remaining service lives of employees.

Page 147: elliottv10

Barry Elliott and Jamie Elliott: Financial Accounting and Reporting (tenth edition) – Instructor’s Manual

147 © Pearson Education Limited 2006

The limits of the 10% corridor are set (at 1 January each year) at the greater of:

(a) 10% of the present value of the obligation before deducting plan assets; and

(b) 10% of the fair value of plan assets. 2002 2003 2004

Limit of ‘10% corridor’ (at 1 January) 320 340 350

Actuarial gains and losses unrecognised

(1 January) – 20 234

Actuarial gain (loss) recognised over 10 years – – –

Cumulative unrecognised gains (losses) – 20 234

(1 January)

Gains (losses) on the obligation 120 290 480

Gains (losses) on the assets (100) (76) (288)

Cumulative gains (losses) before amortisation 20 234 426

Amortisation in the period – – –

Cumulative unrecognised gains (losses) 20 234 426

(31 December)

There would be some recognition of actuarial gains in 2005 as the unrecognised gains and losses in 2004 exceed the 10% corridor as measured at 31 December 2004.

Accounts presentation

The final step is to work out the balance sheet and income statement position for the company.

Income statement

2002 2003 2004

Current service cost 150 160 170

Interest cost 210 180 140

Expected return on plan assets (320) (306) (288)

Income statement charge 40 34 22

Page 148: elliottv10

Barry Elliott and Jamie Elliott: Financial Accounting and Reporting (tenth edition) – Instructor’s Manual

148 © Pearson Education Limited 2006

Balance sheet Present value of obligation, 31 December 3,600 3,500 3,200

Fair value of assets, 31 December (3,400) (3,600) (3,600)

Unrecognised actuarial gains (losses) 20 234 426

Liability in balance sheet 220 134 26

Equity recognition approach

The working will still remain the same, but the recognition of actuarial gains and losses changes.

Income statement

2002 2003 2004 Current service cost 150 160 170

Interest cost 210 180 140

Expected return on plan assets (320) (306) (288)

Income statement charge 40 34 22

Balance sheet

Present value of obligation, 31 December 3,600 3,500 3,200

Fair value of assets, 31 December (3,400) (3,600) (3,600)

Liability (Assets) in balance sheet 200 (100) (400)

Statement of movement in equity

Actuarial gains recognised in the period 20 234 426

Page 149: elliottv10

Barry Elliott and Jamie Elliott: Financial Accounting and Reporting (tenth edition) – Instructor’s Manual

149 © Pearson Education Limited 2006

C H AP T E R 1 5

Chapter 15: Question 1 – Notes to Assist in Answering

The Government deliberately sets up special provisions to reduce taxes in order to encourage certain conduct. Examples might be allowance for payments in to pension funds or for capital investment. The running of a company’s affairs to take maximum benefit of items such as these is tax planning. When a company alters its behaviour solely for tax purposes, with no commercial reason, with the intention of saving tax by using the tax system in a way not intended by Parliament, this is called tax avoidance. Tax evasion is when a company illegally hides income from the tax authorities. Tax planning is to be encouraged, and it is for an accountant to point out the opportunities to use it. Tax evasion is illegal, and an accountant is under an obligation to prevent it happening. Tax avoidance is legal; an accountant is under an obligation to ensure that steps are not taken which are illegal and that details are reported accurately to the authorities. In all dealings relating to the tax authorities, an accountant must act honestly and do nothing that might mislead the authori-ties, but he must do all that he can to assist his employer within these criteria.

Page 150: elliottv10

Barry Elliott and Jamie Elliott: Financial Accounting and Reporting (tenth edition) – Instructor’s Manual

150 © Pearson Education Limited 2006

Chapter 15: Question 2

(a) Accounts Tax Difference (Depreciation) (capital allowances) (timing) 31/3/20X2 Cost 25,000.00 25,000.00 Depn/allce 2,812.50 4,687.50 1,875.00 22,187.50 20,312.50 1,875.00 31/3/20X3 Depn/allce 3,750.00 5,078.13 1,328.13 18,437.50 15,234.38 3,203.13 31/3/20X4 Depn/allce 3,750.00 3,808.59 58.59 14,687.50 11,425.78 3,261.72 31/3/20X5 Depn/allce 3,750.00 2,856.45 (893.55) 10,937.50 8,569.34 2,368.16 31/3/20X6 Depn/allce 3,750.00 2,142.33 (1,607.67) 7,187.50 6,427.00 760.50

Tax Calculated By Deferral Method Deferred tax Deferred charge in year tax provision 31/3/20X2 1,875.00 20% 375.00 375.00 Balance at 31/3/20X2

31/3/20X3 1,328.13 30% 398.44 773.44 Balance at 31/3/20X3

31/3/20X4 58.59 20% 11.72 785.16 Balance at 31/3/20X4

31/3/20X5 (893.55) 19% (169.77) 615.38 Balance at 31/3/20X5

31/3/20X6 (1,607.67) 19% (302.57) 310.12 Balance at 31/3/20X6

Tax Calculated By Liability Method Difference As at As at As at As at As at As at (timing) 31/3/X2 31/3/X3 31/3/X4 31/3/X5 31/3/X6 Tax rate 20% 30% 20% 19% 19%

31/3/X2 1,875.00

31/3/X3 3203.13

31/3/X4 3261.72

31/3/X5 2368.16

31/3/X6 760.50

--------- -------- --------- -------- ----------

375.00 960.94 652.34 449.95 144.69

(b) Under the liability method the focus is on the balance sheet (the objective being to compute the deferred tax liabilities), whereas the deferral method places the focus on the Profit and Loss Account (the objective being to show the annual effect that has arisen in the year of account).

Page 151: elliottv10

Barry Elliott and Jamie Elliott: Financial Accounting and Reporting (tenth edition) – Instructor’s Manual

151 © Pearson Education Limited 2006

Chapter 15: Question 3 – Notes to Assist in Answering

The law has been amended to allow the Inland Revenue to accept accounts drawn up in accor-dance with IFRS, so that two different standards will be acceptable for some years. Therefore the legislation will have to provide for different treatment of specific items under UK GAAP and IFRS. The Finance Act 2004 included legislation which ensured that companies that adopted IFRS to draw up their accounts would receive broadly equivalent tax treatment to companies that con-tinue to use UK GAAP. The clear intention of these provisions is to defer the major tax effects of most transitional adjustments until the tax impact becomes clearer. It remains to be seen whether the taxation effects of any significant changes in profit resulting from the change from UK GAAP to IFRS will be deferred until UK GAAP becomes truly aligned with IFRS. The move towards IFRS is leading to a detailed study of accounting theory and principles, so that the accounting treatment may eventually become the benchmark standard for taxation pur-poses, although this will take several years to reach fruition (if it proves to be attainable).

Page 152: elliottv10

Barry Elliott and Jamie Elliott: Financial Accounting and Reporting (tenth edition) – Instructor’s Manual

152 © Pearson Education Limited 2006

Chapter 15: Question 4 – Notes to Assist in Answering

For Discounting:

• Most transactions take place at fair value.

• Rational buyers and sellers will ensure that this fair value reflects the time value of money and the risk associated with the future expected cash flows, which means that market prices generally will reflect such factors.

• To be consistent, these factors need also to be reflected in the other measures that can be used to determine the carrying amount of assets (in other words, value in use and net realis-able value) and the carrying amount of any liabilities measured by reference to expected future cash flows.

• It follows that, when basing carrying amounts on future cash flows, those cash flows will need to be discounted.

Against Discounting:

• The reliable determination of deferred tax assets and liabilities on a discounted basis re-quires detailed scheduling of the timing of the reversal of each temporary difference. In many cases such scheduling is impracticable or highly complex.

• Therefore, it is inappropriate to require discounting of deferred tax assets and liabilities.

• To permit, but not to require, discounting would result in deferred tax assets and liabilities which would not be comparable between enterprises.

• Discounting would result in deferred tax assets and liabilities which would not be compara-ble between enterprises unless there was a set methodology using standard prescribed discount rates.

• In some cases where capital expenditure is uneven, an unexpected effect of discounting could be to turn an eventual liability into an initial asset.

• Discounting is not generally used in financial accounting, so its use for deferred taxation would be an exception to general accounting principles.

Page 153: elliottv10

Barry Elliott and Jamie Elliott: Financial Accounting and Reporting (tenth edition) – Instructor’s Manual

153 © Pearson Education Limited 2006

C H AP T E R 1 6

Chapter 16: Question 1 – Universal Entrepreneurs plc

(a) The principles outlined in IAS 16 are that:

• A non-current asset is assessed at the year end to ensure that it has not been impaired.

• Fair charge is made to the income statement each year for the benefit of accruing to that ac-counting period for use of the asset concerned.

• In no way does the IAS address the notion of showing on the balance sheet under the head-ing of ‘Non-current assets’ either the value of the assets to the enterprise or the value at which they might be sold.

• It was this factor that caused property investment companies to feel that they were disadvan-taged by the requirements of IAS 16 to depreciate buildings when these formed the major proportion of their asset structure.

They argued strongly that the assets were not USED in the business but were held, like any other investment, for their income-producing value and potential capital growth.

As a result of these representations the IASC developed IAS 40.

(b) (i) Depreciate on the basis of the rate of extraction of growth over the 10 year period in reviewing annually.

(ii) The cost of the building (£4,000,000) should be depreciated over its useful life. It is not an investment property and the period of the lease granted is irrelevant.

(iii) 20% per annum straight-line. (iv) Depreciate on the basis of actual flying hours. (v) Treat as an investment property, revaluing annually but providing depreciation as it

is a base of less than 20 years.

(c) • The revalued amount of the buildings should be depreciated over the remainder of their

useful lives, taking account of the amounts of depreciation already provided. • Unless the value of the land is being consumed in some way (e.g. by mining) this should

not be depreciated over the remaining period of the bases, again having account of the amounts of amortisation already provided.

• When the valuer is instructed in respect of the freehold properties it must be made clear that interests of land need to be distinguished from those in the buildings thereon.

Page 154: elliottv10

Barry Elliott and Jamie Elliott: Financial Accounting and Reporting (tenth edition) – Instructor’s Manual

154 © Pearson Education Limited 2006

Chapter 16: Question 2 – Mercury

(a) Identifying the method

The method of depreciation is the diminishing balance method. The following calculations show that the rate applied at 20%. 20X6 charge = 20% of £80,000 = 16,000 20X7 charge = 20% of £64,000 = 12,800 Cumulative provision = 28,800

(b) How the accumulated depreciation in line (B) was calculated

B/d from (a) above:

20X6 20% of £80,000 = 16,000 20X7 20% of £64,000 = 12,800

20X8 Balance b/f £28,800 Less: 1st disposal 20X6:£15,000 x 20% = £3,000 20X7: £12,000 x 20% = £2,400 (£5,400)

Less: 2nd disposal 20X6: £30,000 x 20% = £6,000

20X7: £24,000 x 20% = £4,800 (£10,800) (16,200) 12,600

20% of (£80,000 – £45,000 disposed of) – £12,600 for accumulated depreciation 4,480 20% of £50,000 replacement for 2nd disposal = 10,000 Depreciation on other asset 1,000 Total given in question 28,080

(c) How the figures for 20X9 are calculated

20X9 Property, plant and equipment Acquired Acquired Acquired Total 20X6 20X7 20X8 20X9 (balancing figure)

£ £ £ £ Cost 35,000 50,000 5,000 90,000 Depreciation to date 17,080 10,000 1,000 28,080 17,920 40,000 4,000 61,920 Charge for 20X9 3,584 8,000 800 12,384 14,336 32,000 3,200 49,536

Page 155: elliottv10

Barry Elliott and Jamie Elliott: Financial Accounting and Reporting (tenth edition) – Instructor’s Manual

155 © Pearson Education Limited 2006

(d) Calculation of profit/(loss) on disposal

Plant disposal I £

Cost 15,000 Less: Depreciation (5,400)

Cash (8,000) Loss 1,600

Plant disposal II

Cost 30,000 Less: Depreciation (10,800)

Cash (21,000) Profit 1,800

Page 156: elliottv10

Barry Elliott and Jamie Elliott: Financial Accounting and Reporting (tenth edition) – Instructor’s Manual

156 © Pearson Education Limited 2006

Chapter 16: Question 3 – Amy

(a) The figures should be the total cost of making the non-current asset usable, excluding all costs of actually using it. Therefore,

11,000 + 100 + 200 + 400 = £11,700

The additional component is cost of machine as it enhances the revenue-earning capacity of the asset.

The replacement parts are cost of using machine – hence the difference in treatment be-tween the two. Maintenance is obviously a cost of usage.

(b) Depreciation spreads the cost (or value) of an item over its useful life, in appropriate pro-portion to the benefit (usefulness).

It is necessary in accordance with the matching convention – allocating expense against corresponding benefit, as part of the profit calculation.

(c) The straight-line method charges a constant percentage of the cost (or value) each year.

The diminishing balance method charges a constant percentage of the net book value (cost less accumulated depreciation brought forward).

Thus the straight-line method has a constant charge but the diminishing balance method has a charge reducing each year of the asset life.

The two methods therefore make different assumptions about the usefulness, the trend or pattern of benefit, of the fixed asset concerned.

(d) Objectivity implies lack of bias. It removes the need for, and the possibility of, subjectivity, of personal opinion. For an accounting figure to be objective, it must be expected that all accountants would arrive at the same figure.

Clearly the figure stated on an invoice has a high degree of objectivity. However, the cal-culation of depreciation is based on estimates of future life and future usefulness and is therefore highly subjective.

(e) This practice can claim the advantage of greater prudence, as the expense is always the higher of the two possibilities. However, it seems to lack consistency. Perhaps more impor-tantly, it obviously fails to attempt to follow the matching convention. It makes no attempt to make the trend of expenses consistent with the trend of benefit or usefulness.

If the profit figure, or profit trend, is regarded as important, then it seems an unsatisfactory practice.

Page 157: elliottv10

Barry Elliott and Jamie Elliott: Financial Accounting and Reporting (tenth edition) – Instructor’s Manual

157 © Pearson Education Limited 2006

Chapter 16: Question 4 – Small Machine Parts Ltd

(a) Year 1 Year 2 Year 3 Year 4 Year 5 £ £ £ £ £

Opening balance 20,000.00 17,085.57 13,733.98 9,879.65 5,447.17 Interest at 15% 3,000.00 2,562.84 2,060.10 1,481.95 817.08

23,000.00 19,648.41 15,794.08 11,361.60 6,264.25 Depreciation 5,914.43 5,914.43 5,914.43 5,914.43 5,914.43

17,085.57 13,733.98 9,879.65 5,447.17 349.82

The income from secondary assets is calculated at 15% of the depreciation charge less the no-tional interest.

Year 1 Year 2 Year 3 Year 4 Year 5 £ £ £ £ £

Depreciation 5,914.43 5,914.43 5,914.43 5,914.43 5,914.43 Interest 3,000.00 2,562.84 2,060.10 1,481.95 817.08

2,914.43 3,351.59 3,854.33 4,432.48 5,097.35 15% 437.16 502.74 578.15 664.87 764.60

437.16 437.16 437.16 437.16 502.74 502.74 502.74 578.15 578.14 664.87 437.16 939.90 1,518.05 2,182.92

Income statement entries Cash flow

Year 1 Year 2 Year 3 Year 4 Year 5 £ £ £ £ £

Operating CF 25,000.00 25,000.00 25,000.00 25,000.00 25,000.00 Depreciation 5,914.43 5,914.43 5,914.43 5,914.43 5,914.43 Operating profit 19,085.57 19,085.57 19,085.57 19,085.57 19,085.57 Income from secondary assets 437.16 939.90 1,518.05 2,182.92

19,085.57 19,522.73 20,025.47 20,603.62 21,268.49 Interest 3,000.00 2,562.84 2,060.10 1,481.85 817.08 Net profit 22,085.57 22,085.57 22,085.57 22,085.57 22,085.57

(b) The annuity method is recommended because it attempts to show the effect of the loss of interest suffered as a result of investing the funds in non-current assets within the organisation. It does this by charging notional interest in addition to the depreciation charge with a reduction for the estimated secondary income on the difference between the depreciation charge and the notional interest. The method suggested of charging the annual average cost is frequently met in practice but is less accurate in that it fails to take account of the opportunity cost of the interest foregone.

Page 158: elliottv10

Barry Elliott and Jamie Elliott: Financial Accounting and Reporting (tenth edition) – Instructor’s Manual

158 © Pearson Education Limited 2006

Chapter 16: Question 5 – Calculation of Depreciation

(a)

Year 1

Straight-line (SF800,000 – SF104,000)/4 = SF174,000

Reducing balance 40% of SF696,000 = SF278,400

(b) Comment to include:

• Directors responsible under IAS 16 for selecting an appropriate method.

• Little guidance given as to how to exercise the choice but the following matters may be relevant:

risk of technological change

incidence of repairs

extent to which the asset characteristics favour a particular method, e.g. a lease would be amortised evenly over its life.

Page 159: elliottv10

Barry Elliott and Jamie Elliott: Financial Accounting and Reporting (tenth edition) – Instructor’s Manual

159 © Pearson Education Limited 2006

Chapter 16: Question 6 – AB

(a) IAS 36 (i) Indicators (assuming significant in all cases)

• Market value lower than book value

• Lower expected cash flows affecting the value in use

• Rates of return have increased adversely affecting the recoverable amount

• Adverse change in the environment e.g. technological, economic or legal or in physical state of asset e.g. obsolescence or damage

• Adverse change in the use to which asset is put e.g. reorganisation programme

• Evidence that the economic performance of the asset will be worse than expected

• The asset has suffered considerable physical change, or obsolescence or physical damage

• Cost of construction over-run making asset less profitable (ii) Recognition and measurement

IAS 36 Impairment of Assets says that if indicated under the above, then undertake a review to establish the extent of any impairment.

Criteria in HCA model

• An asset should not be valued at an amount greater than its cost or recoverable amount

• The recoverable amount being the higher of net selling price and value in use (net present value of future cash flows).

Criteria in Revaluation model

• Compare the carrying value of the asset with its net selling price or value in use.

• If the net selling price OR value in use exceeds the carrying value, no write-down is necessary

• If the recoverable amount is lower than the carrying value, the asset is impaired and the carrying amount of the asset should be reduced to its recoverable amount.

Recognition

• Any recognition is an impairment loss to be recognised as an expense immediately in the income statement.

What if it is not possible to estimate the recoverable value of an individual asset?

This can occur if the asset does not generate independent cash flows and in such a case, the re-coverable amount of the asset’s cash generating unit should be calculated together with value in use on the same basis.

Page 160: elliottv10

Barry Elliott and Jamie Elliott: Financial Accounting and Reporting (tenth edition) – Instructor’s Manual

160 © Pearson Education Limited 2006

An impairment loss is only recognised where its recoverable amount is less than the carrying amounts of the items in that unit.

Allocation of impaired amount where the HCA model is being followed

Any specific impairment of assets should be dealt with initially then allocate first to goodwill, then to intangible assets which have no active market, then to assets whose Net Selling Price is less than their carrying value and finally to other assets on a pro rata basis.

Allocation of impaired amount where the Replacement model is being followed

An impairment loss relating to a revalued asset is treated as a revaluation decrease and therefore charged to revaluation account. Where the impairment loss is greater than the carrying amount of the asset, a liability should only be recognised where it is required by other International Standards. After recognition of an impairment loss, the depreciation charge should be adjusted to allocate the revised carrying amount (less residual value) systematically over its remaining life. An enterprise should review the balance sheet to assess whether a recognised impairment loss still exists or has decreased. Any reversal of an impairment loss should be recognised in the income statement.

(b) AB (i) Impairment of machinery

Indicators are the inventory losses and the taxi business problems.

Procedure: Compare the carrying value ($290,000) with its recoverable amount which has to be calcu-lated. The calculation is to determine the higher of an asset’s net selling price ($120,000) and its value in use. The value in use is $100,000 discounted at 10% for three years, i.e. $248,600 approx.

Thus the recoverable amount would be deemed to be $248,600. AB would therefore write down the asset from $290,000 (carrying value) to $248,600 (its value in use) and recognise the loss of $41,400 in the income statement.

(ii) Impairment of the car taxi business treated as a CGI

Impairment losses should be recognised if the recoverable amount of the cash generating unit is less than the carrying value of the items of that unit.

At 1 February 20X1

1.1.X1 Impairment 1.2.X1 loss $000 $000 $000 Goodwill 40 (15) 25 Intangible assets 30 30 Vehicles 120 (30) 90 Sundry net assets 40 40 230 (45) 185

An impairment loss of $30,000 is recognised first for the specific asset (i.e. the stolen vehi-cles) and the balance ($15,000) is attributed to goodwill.

Page 161: elliottv10

Barry Elliott and Jamie Elliott: Financial Accounting and Reporting (tenth edition) – Instructor’s Manual

161 © Pearson Education Limited 2006

At 1 March 20X1 1.2.X1 Impairment 1.3.X1 $000 $000 $000 Goodwill 25 (25) Intangible assets 30 (5) 25 Vehicles 90 90 Sundry net assets 40 40 185 (30) 155

Note the tricky point – that is that the net selling price of the sundry net assets has not fallen. It is therefore not permissible to reduce the sundry net assets.

Page 162: elliottv10

Barry Elliott and Jamie Elliott: Financial Accounting and Reporting (tenth edition) – Instructor’s Manual

162 © Pearson Education Limited 2006

C H AP T E R 1 7

Chapter 17: Question 1 – Grabbit plc

(a) PV of future lease payments = £350,000 (cost of asset) The interest rate to apply to the six payments of £92,500 to equate to a PV of £350,000 is 15%. This is calculated as follows:

80,435 + 69,943 + 60,820 + 52,887 + 45,989 + 39,990 = 350,064

Balance sheet

Assets £

Cost of leased asset 350,000

Depreciation for year 43,750

306,250

Liabilities

PV of future lease payments

80,435 + 69,943 + 60,820 + 52,887 + 45,989 = 310,074, say 310,000

Income statement Interest on leasing obligation

(£350,000 × 15%) 52,500

Depreciation on leased asset 43,750

Liability at year end 96,250

(NB 350,000 Cost

52,500 Interest

402,500

(92,500) Payment

310,000 Bal. c/f)

(b) The obligations under a long-term lease are in substance no different from those under a loan but prior to the introduction of the leasing standard they did not appear on the balance sheet. This made the balance sheet unreliable as one could not be confident that there were not undisclosed liabilities such as leases. Further, where a business had essentially the con-trol of an asset for a substantial period of time they had most if not all of the benefits and risks associated with ownership. It was considered a deficiency that these asset rights were not reflected in the balance sheets, even if they were different in nature to outright owner-ship. Therefore the leasing standard attempted to capture the assets and liabilities which occurred when longer-term ownership like contracts were entered into.

Page 163: elliottv10

Barry Elliott and Jamie Elliott: Financial Accounting and Reporting (tenth edition) – Instructor’s Manual

163 © Pearson Education Limited 2006

(c) The advantage would be that the obligation would no longer involve the lessee in a long-term commitment although they would probably have long-term access. This would take the relationship out of the financial lease category and put it in as an operating lease. As a con-sequence, under existing standards it would not have to be capitalised but rather would be disclosed under operating leases. The ratios would be better but only if influential investors did not adjust their figures for the long-term obligations that would probably arise.

Page 164: elliottv10

Barry Elliott and Jamie Elliott: Financial Accounting and Reporting (tenth edition) – Instructor’s Manual

164 © Pearson Education Limited 2006

Chapter 17: Question 2

(a)

• The ‘title’ to the goods acquired on a hire purchase agreement or lease finance remains le-gally vested in the lessor.

• Yet in commercial substance, at the point of getting custody of the asset the lessee acquires substantially all the risks and rewards of owning the asset.

• Hence accountants have preferred to overlook the legal form and focus more on the com-mercial substance. For example, they have accounted for assets acquired on HP terms or on finance lease terms as if the title passes to the lessee at the date of transfer of the custody of the asset.

• IAS 17, while endorsing the accounting practice, changes the conceptual basis for this ac-counting practice. It emphasises that what the lessor capitalises, at the point of acquiring custody of the leased assets, is not the asset itself (which admittedly he does not own yet), but his own right to use that asset.

The amount at which he capitalises this right is the present value of the minimum lease pay-ments he commits himself to.

(b) In accordance with IAS 17:

• A finance lease is a lease that transfers to the lessee substantially all the risks and rewards of owning an asset.

• All other leases are operating leases.

• The Standard sets out a presumption that a lease transfers substantially all the risks and re-wards of ownership to the lessee IF ‘at the inception of a lease the risks and rewards of ownership are transferred to the lessee’ and if ‘at the inception of a lease the present value of the minimum lease payments, including any initial payment, amounts substantially to all of the fair value of the leased asset’.

Other criteria such as transference of legal title and the right to use the asset for all its life are also considered.

(c) (1) Smarty plc

Machine acquired under a finance lease

1.8.X7 Obligation 15,000

Provision for depreciation on machine

31.3.X8 Depreciation 2,000

31.3.X9 Depreciation 3,000

31.3.X0 Depreciation 3,000

31.3.X1 Depreciation 3,000

Page 165: elliottv10

Barry Elliott and Jamie Elliott: Financial Accounting and Reporting (tenth edition) – Instructor’s Manual

165 © Pearson Education Limited 2006

Finance charges account

31.3.X8 Obligation under 31.3.X8 Income statement 1,200

fin. lease 1,200

31.3.X9 Obligation under 31.3.X9 Income statement 1,229

fin. lease 1,229

31.3.Y0 Obligation under 31.3.Y0 Income statement 543

fin. lease 543

31.3.Y1 Obligation under 31.3.Y1 Income statement 28

fin. lease 28

Obligation under finance lease on machinery account

Cash 4,000 1.8.X7 Machinery a/c 15,000

31.3.X8 Balance c/d 12,200 31.3.X8 Finance charges a/c 1,200

16,200 16,200

Cash 6,000 1.4.X8 Balance b/d 12,200

31.3.X9 Balance c/d 7,429 31.3.X9 Finance charges a/c 1,229

13,429 13,429

Cash 6,000 1.4.X9 Balance b/d 7,429

31.3.Y0 Balance c/d 1,972 31.3.Y0 Finance charges a/c 543

7,972 7,972

Cash 2,000 1.4.Y1 Balance b/d 1,972

31.3.Y1 Finance charges a/c 28

2,000 2,000

(2) Income statement entries for y/e 31 March 20X8 extracts:

£ £

Finance charge 1,200

Depreciation 2,000

Balance sheet as at 31 March 20X8 extracts:

Non-current asset 15,000

Depreciation (2,000) 13,000

Liability 12,200

Page 166: elliottv10

Barry Elliott and Jamie Elliott: Financial Accounting and Reporting (tenth edition) – Instructor’s Manual

166 © Pearson Education Limited 2006

Chapter 17: Question 3 – The Mission Company Ltd

(a) Income statement

20X6 20X7

£ £

Item Y 20,000 20,000

Item Z 30,000 30,000

As an operating lease each annual payment is simply rent.

(b)

Workings (Item Y) £

Total lease price: 10 × £20,000 = 200,000

Purchase price = 160,000

Finance charge = 40,000

Period of lease: 10 years

Sum of the digits = 10 + 9 + 8 + 7 + 6 + 5 + 4 + 3 + 2 + 1 = 55

Therefore Charge to 20X6 = 4/55 × £40,000

Charge to 20X7 = 3/55 × £40,000

Finance charge to income statement 20X6 20X7

£ £

Item Y 2,909 2,182

Working (Item Z)

Total lease price: 10 × £30,000 = £300,000

Purchase price = £234,000

£ 66,000

Sum of the digits = 55

Therefore Charge to 20X6 = 6/55 × £66,000

Charge to 20X7 = 5/55 × £66,000

Finance charge to income statement

20X6 20X7

£ £

Item Z 7,200 6,000

Page 167: elliottv10

Barry Elliott and Jamie Elliott: Financial Accounting and Reporting (tenth edition) – Instructor’s Manual

167 © Pearson Education Limited 2006

Depreciation charge to income statement

20X6 20X7

Item Y 16,000 16,000

Item Z 19,500 19,500

(c) Balance sheet 20X7

Non-current assets Cost Accumulated Dep’n NBV

£ £ £

Item Y 160,000 128,000 32,000

Item Z 234,000 117,000 117,000

Current liabilities £

Item Y 2[20,000 40,000]

55− × 18,545

Item Z 25,200

Non-current liabilities £

Item Y 19,273

Item Z 82,800

Note: Minimum lease commitments at 31 December 20X7 in relation to finance leases were:

Item Y Item Z

£ £

20X8 20,000 30,000

20X9 20,000 30,000

20Y0 30,000

20Y1 30,000

40,000 120,000

Less: Finance charge allocated to future periods 2,182 12,000

37,818 108,000

Workings: to liabilities Item Y

Year B/forward Cost Repayment Interest C/forward

£ £ £ £ £

20X0 160,000 20,000 7,273 147,273

20X1 147,273 20,000 6,545 133,818

20X2 133,818 20,000 5,818 119,636

20X3 119,636 20,000 5,091 104,727

Page 168: elliottv10

Barry Elliott and Jamie Elliott: Financial Accounting and Reporting (tenth edition) – Instructor’s Manual

168 © Pearson Education Limited 2006

20X4 104,727 20,000 4,364 89,091

20X5 89,091 20,000 3,636 72,727

20X6 72,727 20,000 2,909 55,636

20X7 55,636 20,000 2,182 37,818

20X8 37,818 20,000 1,455 19,273

20X9 19,273 20,000 727 -

£40,000

Therefore current liability = 20,000 – 1,455 = £18,545

non-current liability = 37,818 – 18,545 = £19,273

Item Z Year B/forward Cost Repayment Interest C/forward

20X2 234,000 30,000 12,000 216,000

20X3 216,000 30,000 10,800 196,800

20X4 196,800 30,000 9,600 176,400

20X5 176,400 30,000 8,400 154,800

20X6 154,800 30,000 7,200 132,000

20X7 132,000 30,000 6,000 108,000

20X8 108,000 30,000 4,800 82,800

20X9 82,800 30,000 3,600 56,400

20Y0 56,400 30,000 2,400 28,800

20Y1 28,800 30,000 1,200 –

£66,000

Therefore current liability = 30,000 – 4,800 = £25,200

non-current liability = 108,000 – 25,200 = £82,800

Page 169: elliottv10

Barry Elliott and Jamie Elliott: Financial Accounting and Reporting (tenth edition) – Instructor’s Manual

169 © Pearson Education Limited 2006

Chapter 17: Question 4 – X Ltd

(a) Workings to provide figures for income statement and balance sheet entries

Period Liability Rental Liability Finance Liability

at start payment during charge at end

of period period (4.3535%) of period

1.1.X4 100,000 12,000 88,000 3,831 91,831

1.7.X4 91,831 12,000 79,831 3,475 83,306

1.1.X5 83,306 12,000 71,306 3,104 74,410

1.7.X5 74,410 12,000 62,410 2,717 65,127

1.1.X6 65,127 12,000 53,127 2,313 55,440

1.7.X6 55,440 12,000 43,440 1,891 45,331

1.1.X7 45,331 12,000 33,331 1,451 34,782

1.7.X7 34,782 12,000 22,782 992 23,774

1.1.X8 23,774 12,000 11,774 513 12,287

1.7.X8 12,287 12,000 287 12 299

120,000

Depreciation is over useful economic life, using the historical cost of the leased asset

100,0008

= £12,500 per year

Income statement

Finance Depreciation Total

Charge

20X4 (3,831 + 3,475) 7,306 12,500 19,806

20X5 5,821 12,500 18,321

20X6 4,204 12,500 16,704

20X7 (1,451 + 992) 2,443 12,500 14,943

The balance sheet is as follows: Assets

20X4 20X5 20X6 20X7

Equipment

under finance

lease 100,000 100,000 100,000 100,000

Depreciation 12,500 25,000 32,500 50,000

87,500 75,000 62,500 50,000

Page 170: elliottv10

Barry Elliott and Jamie Elliott: Financial Accounting and Reporting (tenth edition) – Instructor’s Manual

170 © Pearson Education Limited 2006

Liabilities Within 2–5 years 65,127 45,331 23,774 299 Within 1 year *18,179 19,796 21,557 23,475

83,306 65,127 45,331 23,774

* The £18,179 = £24,000 – interest (£3,104 + £2,717)

Page 171: elliottv10

Barry Elliott and Jamie Elliott: Financial Accounting and Reporting (tenth edition) – Instructor’s Manual

171 © Pearson Education Limited 2006

Chapter 17: Question 5 – Bridge Finance plc

Workings for the lease receivable account

$

1 Jan × 5 Cost 37,200

Direct expense 708

31 Dec × 5 Balance 37,908

10% interest 3,791

Cash –10,000

1 Jan × 6 Balance 31,699

31 Dec × 6 Interest 3,170

Cash –10,000

1 Jan × 7 Balance 24,869

31 Dec × 7 Interest 2,487

Cash –10,000

Balance 17,356

As this is a finance lease the accounts in the lessor’s books would be;

Income Statement

Interest revenue $2,487

Balance sheet

Asset

Lease receivable $17,356 Current 8,265

Non-current 9,091

This can be reconciled as follows:

Gross cashflows receivable $20,000

Less interest 2,644

Net amount 17,356

Page 172: elliottv10

Barry Elliott and Jamie Elliott: Financial Accounting and Reporting (tenth edition) – Instructor’s Manual

172 © Pearson Education Limited 2006

C H AP T E R 1 8

Chapter 18: Question 1 – Environmental Engineering plc

(i) Referring to IAS 38 criteria in (ii) below, only (c) might qualify for de-ferral as development expenditure.

(a) is applied research.

(b) is development cost (£1.2m) that has not yet been incorporated into a specific, separate viable project. However, the line between categories is often indistinct in practice, e.g. between development and production costs.

Looked at in general, all three relate to a specific project to which it appears expenditure can be separately allocated. However, the outcome is not reasonably certain as to either technical feasi-bility or commercial viability. We have no idea or projections of sales volume or price/revenue in total and whether it will exceed costs. It is assumed that a plc would have the necessary re-sources to complete the project but there is no evidence of this. Item (c) would not stand out from (a) and (b) and it is recommended that all be written off as expenses. It could be capitalised later when evidence is produced as to criteria for proceeds.

(ii) IAS 38 criteria (para. 45):

(a) Technical feasibility

(b) Intention to complete and use or sell

(c) Ability to use or sell

(d) Asset will generate possible future income – demonstrate existence

(e) of a market, availability of technical, financial and other resources to complete the de-velopment or to use or sell.

(iii)

• Amortisation should begin with the commencement of production.

• Any write-off should be over the period the product is expected to be sold.

This implies that the amortisation costs can be included in stocks being produced for sale.

• Deferred development expenditure should be reviewed at the end of each accounting period and,

to the extent that it is not considered recoverable, it should be written off.

Page 173: elliottv10

Barry Elliott and Jamie Elliott: Financial Accounting and Reporting (tenth edition) – Instructor’s Manual

173 © Pearson Education Limited 2006

Chapter 18: Question 2 – Italin NV

IAS 38: Pure and applied research, always written off in period incurred; development expendi-ture may be carried forward in certain circumstances.

Income Statement for the year ended 30 September (extract)

20X1 20X2 20X3 20X4 20X5 20X6 20X7 Research expenditure 200

Development cost – 50 50 50 50 50 50

Depreciation 300 300 300 300 300 300 300

Balance Sheet as at 30 September (extract)

20X1 20X2 20X3 20X4 20X5 20X6 20X7 Intangible fixed assets 300 250 200 150 100 50 –

Tangible fixed assets 2,200 1,900 1,600 1,300 1,000 700 –

Projects must be reviewed each year.

Treatment of fixed assets used in R&D as for any assets.

Disclosure

• Accounting policy

• Consistency and application of IAS 38

amounts written off in the period

pure and applied research is written off

development expenditure is capitalised and written off over six years

• Movement on development costs capitalised

• Fixed assets used are depreciated in the normal way over their useful life of 7 years.

Page 174: elliottv10

Barry Elliott and Jamie Elliott: Financial Accounting and Reporting (tenth edition) – Instructor’s Manual

174 © Pearson Education Limited 2006

Chapter 18: Question 3 – Oxlag plc

(i)

Research and Development Costs Account

£000 £000 Capital costs b/f at Capitalised costs c/f

start of year (project C) 200 Project C 500

Costs incurred in year:

Project A 25 Costs written off to

Project C 265 Profit & loss account:

Project D 78 Project A 35

Project D 98 133

Depreciation:

Laboratory:

Project C 20

Equipment:

Project A 10

Project C 15

Project D 20

633 633

Capitalised costs b/f Project C 500

(consists of 200 b/f + 265 costs incurred + 20 laboratory depreciation and 15 equipment depre-ciation)

Fixed Assets: Specialized Laboratory Account

£000 £000 Cost b/f at start of year 500 Depreciation b/f at start of

year 25

Depreciation c/f at end of year 45 Depreciation charge for year20

Cost c/f at end of year 500

545 545

Cost b/f at start of year 500 Depreciation b/f at start of year 45

Page 175: elliottv10

Barry Elliott and Jamie Elliott: Financial Accounting and Reporting (tenth edition) – Instructor’s Manual

175 © Pearson Education Limited 2006

Fixed Assets: Specialized Equipment Cost

£000 £000

Cost b/f at start of year: Depreciation b/f at start of year

Project C 75 Project C 15

Project D 50 Project D 10

Additions: Depreciation provided in year:

Project A 50 Project A 10

Project D 50 Project C 15

Project D 20

Depreciation c/f at end of year 70 Cost c/f at end of year 225

295 295

Cost b/f at start of year 225 Depreciation b/f at start of year 70

Market Research Costs Account

£000 £000

Costs b/f at start of year 250 Costs c/f at end of year 325

Costs in year 75

325 325

Costs b/f at start of year 325

Assumption is that this is a contract that will continue in future years.

(ii) Amount to be charged as research costs charged in the income statement for the year ended 31 January 20X2

Fees Fees

Per T a/c Project A: Costs 25

Dep’n 10 35

Project D: Costs 78

Dep’n 20 98

133

(iii) Basis of amortisation:

• Any reasonably systematic basis of amortisation per IAS 38.

• Amount spent and written off reconciled with opening and closing balances in the bal-ance sheets.

• Most likely basis here will be expected sales of the new drug with amortisation being calculated as the proportion of total sales sold during each year.

Page 176: elliottv10

Barry Elliott and Jamie Elliott: Financial Accounting and Reporting (tenth edition) – Instructor’s Manual

176 © Pearson Education Limited 2006

Disclosure

• Accounting policy stating basis of capitalisation and basis of write-off

(iv) Balance sheet amounts

Fixed assets £000 Intangible assets:

Deferred development expenditure

(recovery assured by projected future sales) 500

Tangible assets:

Land and buildings: Specialised laboratory 455

Plant and machinery: Specialised laboratory equipment 155

Current assets Inventories:

Long-term work-in-progress 325

(v) Disclosures about new improved drug sales

Identify as non-adjusting post balance sheet event which requires disclosure if material in ac-cordance with IAS 10, having arisen between the end of year 31.1.20X2 and the date of signing the accounts on 14.7.20X2. This does appear to nbe material, therefore the accounts will need to disclose:

• date of new drug going on sale

• success of new drug

• expectation that the sales of the new drug will significantly increase following year’s profits.

Page 177: elliottv10

Barry Elliott and Jamie Elliott: Financial Accounting and Reporting (tenth edition) – Instructor’s Manual

177 © Pearson Education Limited 2006

Chapter 18: Question 4 – Goodwill and Intangible Assets

(a) IFRS 3 defines goodwill as ‘any excess of the cost of the acquisition over the acquirer’s interest in the fair value of the identifiable assets, liabilities and contingent liabilities ac-quired at the date of the exchange transaction’ (i.e. the date the assets were acquired).

The cost of the acquisition is the cash or cash equivalent paid or the fair value, at the date of exchange, of the other purchase consideration given in exchange for control over the net assets of the other enterprise plus any costs directly attributable to the acquisition.

Fair value is defined as the amount for which an asset could be exchanged or a liability set-tled between knowledgeable, willing parties in an arm’s length transaction.

The fair value of the assets or liabilities at the date of purchase can include a provision for reorganisation costs, provided a plan has been developed within three months of the date of acquisition. The costs included in the provision can only include costs relating to the ac-quired business for employee redundancy, closing facilities, eliminating product lines and terminating onerous contracts.

(b) IFRS 3 requires an impairment test to be carried out. Unless goodwill is impaired continue to carry at cost.

(a) Total net assets per balance sheet 58,234

(i) No adjustment as no readily ascertainable

market and no information to verify

directors’ estimate (but see (vi) below)

(ii) Valuation 23,000

Other tangible fixed assets 18,000

Total fair value 41,000

Value in balance sheet 38,300

Revaluation 2,700

(iii) Inventories at net realisable value 20,000

(less than replacement cost)

21,600

Loss (1,600)

(iv) Bank loan

Loan at 31.5.09: 12,100×1.13 16,105

Discounted to 31.5.06 at 7% 13,146

Value in balance sheet 12,100

Increase in liability (1,046)

(v) No adjustment required

(as the reorganisation was decided before

acquisition and future losses cannot be

Page 178: elliottv10

Barry Elliott and Jamie Elliott: Financial Accounting and Reporting (tenth edition) – Instructor’s Manual

178 © Pearson Education Limited 2006

included in the provision)

(vi) Impairment of brand names (6,020)

Fair value of Yukon at acquisition 52,268

Goodwill:

Cost of acquisition 2.5 × 2.25 × 10,000 × 80% 45,000

Fair value of net assets acquired 80% × 52,268 41,814

Goodwill 3,186

Valuation of goodwill 1,000

Impairment 2,186

Page 179: elliottv10

Barry Elliott and Jamie Elliott: Financial Accounting and Reporting (tenth edition) – Instructor’s Manual

179 © Pearson Education Limited 2006

Chapter 18: Question 5 – The Brands Debate

Many of the arguments for including brands in the balance sheet are given in Section 18.12 of Chapter 18. Including brands in the balance sheet increases shareholders’ funds and thus re-duces gearing. A reduction in gearing reduces investors’ and banks’ perception of the risk of the company and it is likely to increase the company’s ability to borrow funds. Also, including brands in the balance sheet shows investors and management the value of the company’s brands, thus providing more information to those users of accounts and enabling them to make more rational decisions.

A brand which has been purchased by a company can be included in the balance sheet. Under IAS 38 Intangible Assets, the brand is included at cost and amortised over its useful life. The ‘allowed alternative treatment’ enables the brand to be revalued and reductions in the brand’s valuation below its original cost are charged to the income statement.

IAS 38 says that internally generated goodwill should not be recognised as an asset (para. 36). However, the cost of developing a brand could be taken as development expenditure and this cost subsequently capitalised in the balance sheet and amortised in the income statement. So, it is possible to capitalise internally generated brands. However, the cost of developing a success-ful brand is likely to be considerably less than its market value (if the brand was purchased, it would be shown in the balance sheet at its market value).

So, the treatment of purchased and internally generated brands is different, and in most situa-tions internally generated brands will not be included in the balance sheet (whereas purchased brands would be included).

It would be possible to include an internally generated brand in the balance sheet at its current market value, provided it was initially included as development expenditure and the ‘allowed alternative treatment’ (of IAS 38) of including the brand at its fair value was included for the balance sheet valuation. However, IAS 38 does require the fair value to be determined by refer-ence to an active market.

On the subject of separability of brands, when a business is acquired, it is likely to be difficult to distinguish between brands and other goodwill. The total amount of goodwill, being the differ-ence between the purchase consideration and the fair value of assets acquired, can be determined. However, dividing this total goodwill between brands and other goodwill will be difficult and is likely to be subjective. Also, in acquiring a company, a number of brand names may be acquired, and it is likely to be very difficult (and subjective) to say how much each of the brands is worth.

On purchased vs home-grown brands, the different accounting treatment has been discussed above. In most situations, purchased brands will be included as an asset in the balance sheet, whereas home-grown brands will not. As both purchased and home-grown brands have value, this different accounting treatment is not consistent. However, this different accounting treat-ment arises because accountants are prepared to include an item in assets when its purchase price is known (as with purchased brands) but are reluctant to include it as an asset when it has been internally generated (and its market value is not certain). For investors, ideally they would like the value of both purchased and home-grown brands to be included in the balance sheet, but this creates the risk that directors may artificially inflate the value of home-grown brands and thus mislead investors.

Page 180: elliottv10

Barry Elliott and Jamie Elliott: Financial Accounting and Reporting (tenth edition) – Instructor’s Manual

180 © Pearson Education Limited 2006

As with land and buildings, some companies argue that brands have an infinite life. Current ex-penditure on advertising and marketing the product (e.g. a Mars Bar) maintain the value of the brand, so no amortisation of the brand’s value is justified. IAS 38 says that intangible assets should be amortised over their life, which should not normally exceed 20 years. One can see that some brands have a life of significantly less than 20 years. For instance, a 1 GHz micro-processor has a life of, at most, only a few years as it is superseded by faster processors. Although the brand of ‘Intel’ may have a relatively long life, the company must continue to de-velop its products (i.e. make the microprocessors faster) in order to keep its brand alive.

However, other brands, like the Mars Bar, have a life of significantly more than 20 years – the Mars Bar existed more than 60 years ago. For most brand names, a life of 20 years is a realistic maximum (many brands have a life of less than 20 years), but some brands may have a signifi-cantly longer life. However, although a brand may have had a life of more than 20 years, there is no certainty that it will continue to exist for another 20 years. Many computer companies which were successful 20 years ago no longer exist (e.g. Commodore, Sinclair). How many of today’s well-known brands will no longer exist in 10 or 20 years’ time?

All buildings eventually fall down or are demolished, and all brands will eventually die. So, we would argue that the cost of brands should be amortised in the income statement. It is wrong not to amortise the cost of brands, as eventually they will be worthless.

There is a further argument that even if the brand continues to be reported in the balance sheet at the existing value, the reality is that expenditure has been currently incurred which effectively replaces the original brand value. This means that there has been a substitution of a new brand for the old rather than a maintenance of the old brand.

Page 181: elliottv10

Barry Elliott and Jamie Elliott: Financial Accounting and Reporting (tenth edition) – Instructor’s Manual

181 © Pearson Education Limited 2006

C H AP T E R 1 9

Chapter 19: Question 1 – Sunhats Ltd

1. The principle usually followed is to include in the stock valuation only those expenses which relate to the bringing of the product to its present condition and location. In practice, this often entails:

• including factory (or production) expenses

• excluding selling, finance and administration expenses.

2. Sunhats Ltd factory expenses to be included in inventory valuation: Wages of storemen and foremen Salary of production manager Rent and rates, repairs and depreciation; proportion relating to factory and stores would be included e.g. electric power.

3. Expenses to be excluded from inventory valuation:

• Salaries of sales manager and salespeople, advertising and carriage outwards. These ex-penses are excluded as they relate to selling and distributing the goods, not to the production of them.

• Bad debts and bank interest*: these finance charges are excluded as they relate to the business as a whole and not merely to production.

• Salaries of personnel officer*, buyer*, accountant* and company secretary*, and direc-tors’ fees*: these administration expenses are excluded, as they similarly relate to the business as a whole.

• Development expenditure: this is excluded as it is clearly not relevant to the cost of ex-isting stock.

*The items marked with an asterisk are marginal. It can be argued that part of these ex-penses relate to production and should therefore be regarded as factory overheads.

4. It is important to ensure that the overhead expenses included in the inventory valuation are:

• appropriate in the circumstances of the business, and

• included on a consistent basis from year to year.

Page 182: elliottv10

Barry Elliott and Jamie Elliott: Financial Accounting and Reporting (tenth edition) – Instructor’s Manual

182 © Pearson Education Limited 2006

Chapter 19: Question 2 – Inventory Valuation Methods

(i) Receipts Issues Balance Date Quantity Rate £ Quantity Rate £ Quantity Rate £

FIFO 1/7 100 10 1,000 100 10 1,000 10/7 80 10 800 20 10 200 12/7 100 9.8 980 100 9.8 980 14/7 20 10 200 20 9.8 196 80 9.8 784 15/7 50 9.6 480 50 9.6 480 20/7 100 9.4 940 100 9.4 940 30/7 20 9.8 196 80 9.4 752 50 9.6 480 20 9.4 188 Cost of goods sold 2,648

LIFO 1/7 100 10 1,000 100 10 1,000

10/7 80 10 800 20 10 200

12/7 100 9.8 980 100 9.8 980

14/7 100 9.8 980 20 10 200

15/7 50 9.6 480 50 9.8 480

20/7 100 9.4 940 100 9.4 940

30/7 90 9.4 846 20 10 200

50 9.6 480

10 9.4 94

Cost of goods sold 2,626

Weighted average 1/7 100 10 1,000 100 10 1,000

10/7 80 10 800 20 10 200

12/7 100 9.8 980 100 9.8 980

14/7 100 9.83 983 120 9.83 1,180

20 9.83 196.7

15/7 50 9.6 480 50 9.6 480

20/7 100 9.4 940 100 9.4 940

30/7 90 9.5 855 170 9.5 1,615

80 9.5 760

Cost of goods sold 2,638

Page 183: elliottv10

Barry Elliott and Jamie Elliott: Financial Accounting and Reporting (tenth edition) – Instructor’s Manual

183 © Pearson Education Limited 2006

(ii) Advantages and disadvantages

FIFO

• The movement of some stock follows this pattern in reality e.g. perishables.

• However, the charge to cost of sales will still represent out-of-date prices.

• This means that a distribution policy based on profits calculated using this method will re-duce the operating capital base.

• The balance sheet value will value stock at approaching current values.

LIFO

• The movement of stock does not follow this pattern and detailed records will be required to track costs.

• The charge to cost of sales will represent prices prevalent at date of sale.

• This means that a distribution policy based on profits calculated using this method will tend to maintain the operating capital base.

• However, the balance sheet value will value stock at out-of-date values.

Average cost

• This is a common compromise between the two methods.

• The advantage is that the average represents a compromise between the FIFO and LIFO methods.

• However, there is a disadvantage in that the average cost has to be recalculated after each purchase.

(iii) Effect of a physical shortage of inventory

FIFO Closing inventory

75 @ 9.4 705

Cost of sales increased by

5 @ 9.4 47

LIFO Closing inventory

15 @ 10.0 150

50 @ 9.6 480

10 @ 9.4 9.4

724

Page 184: elliottv10

Barry Elliott and Jamie Elliott: Financial Accounting and Reporting (tenth edition) – Instructor’s Manual

184 © Pearson Education Limited 2006

Cost of sales increased by

5 @ 10 50

Weighted average Closing inventory

75 @ 9.5 712.5

Cost of sales increased by

5 @ 9.5 47.5

Page 185: elliottv10

Barry Elliott and Jamie Elliott: Financial Accounting and Reporting (tenth edition) – Instructor’s Manual

185 © Pearson Education Limited 2006

Chapter 19: Question 3 – Alpha Ltd

Principles

The basis on which the stocks are valued in this solution is the one which is most commonly used by companies, i.e. the lower of cost and net realisable value. The term ‘cost’ includes those overheads which have been incurred in bringing the stocks to their existing condition, namely manufacturing overheads. Selling and distribution expenses have been excluded from cost as it is assumed that these are not incurred until the units are sold.

Valuation details

Raw materials: 100 tons ×××× cost £140 per ton = £14,000

The net realisable value is assumed to be greater than this amount as the finished units (which incorporate the steel) sell at a profit, as follows

£ Selling price 500

Less: Selling and distribution expenses 60

Net realisable value 440

Manufacturing costs (see workings below) 350

Profit per unit 90

The current replacement price has not been taken, as it is not within the basis of valuation stated above. However, as the replacement price has fallen this is a suitable time to con-sider whether the client should be advised to amend the basis of stock valuation to ‘the lower of cost, replacement price and net realisable value’, which is more conservative. On this basis the stock would be valued at £130 per ton.

Finished units: 100 ×××× cost £350 = £35,000

The cost comprises: Per unit £

Materials 50

Labour 150

Manufacturing overheads – 100% of labour 150

£350

Net realisable value is greater than the cost:

Selling price 500

Less: Selling and distribution expenses 60

Net realisable value £440

Page 186: elliottv10

Barry Elliott and Jamie Elliott: Financial Accounting and Reporting (tenth edition) – Instructor’s Manual

186 © Pearson Education Limited 2006

Damaged, finished units: 10 ×××× £240 = £2,400

These units have been valued at cost less the amount of the loss that will be incurred when the units have been rectified, as follows:

Per unit Valuation £ £

Cost of finished units 350 350

Cost to rectify 200

Total cost 550

Less: Net realisable value 440

Loss £110 110

Amount per unit included in the balance sheet £240

Semi-finished units: 40 ×××× cost £250 = £10,000

The cost comprises: Per unit £

Materials 50

Labour 100

Manufacturing overheads – 100% of labour 100

Total cost per unit so far £250

An estimate should be made of the cost required to finish the work. If the total estimated cost exceeds the net realisable value, then the excess must be provided for by deducting it from the £250 cost; this is similar in principle to the treatment of the damaged units. For example:

Per unit

£

Total cost per unit so far (as above) 250

Estimated costs to complete 220

Estimated total costs to completion 470

Less: Net realisable value 440

Estimated loss on completion 30

Valuation:

Total cost per unit so far 250

Less: Estimated loss on completion 30

220

Page 187: elliottv10

Barry Elliott and Jamie Elliott: Financial Accounting and Reporting (tenth edition) – Instructor’s Manual

187 © Pearson Education Limited 2006

Chapter 19: Question 4 – Beta Ltd

1. As the raw materials will realise more than cost, they have obviously been valued at the standard cost, namely £30,000.

2. A review of the price variance account shows that, in total, the actual cost of materials has consistently been well above the standard costs.

3. Consequently, the £30,000 standard cost of raw materials in stock is significantly below the actual cost; and, unless the stock figure is adjusted to the actual cost, this year’s profit will be understated. (Moreover, the understatement of stocks this year will result in next year’s profits being artificially inflated.)

4. Therefore, the figure to be included in the balance sheet should not be the standard cost but a figure which is reasonably close to actual cost. This could be done in one of the following ways:

(a) Value each item at the actual cost paid for it, by referring to the purchase invoices con-cerned. However, this may be too laborious, in which case method (b) or (c) should be considered.

(b) If the company has revised the standard costs for use in the following year, then it may be suitable to use these revised costs for valuing the stocks in the balance sheet. (Presumably the revised standards are based on the cost applicable around the year-end.)

(c) If methods (a) and (b) are impracticable, a rough and ready method may be used, as follows: £

Balance on raw materials control account 30,000

This is equal to the goods purchased in October,

November and December, when the price

variances totalled 2,700

Value of raw materials at year-end 32,700

Care is needed in using this method, as the price variances may have arisen over a narrow range of materials, in which case the calculations of the adjustment needed should embrace only those materials.

Conclusion

Standard costs are used mainly as a tool of management control; their use in the valuation of stocks for accounts purposes is merely identical. Standard costs should not be used for stock valuation unless they are reasonably close to actual costs.

Page 188: elliottv10

Barry Elliott and Jamie Elliott: Financial Accounting and Reporting (tenth edition) – Instructor’s Manual

188 © Pearson Education Limited 2006

Chapter 19: Question 5 – Bottom

Bottom income statement using FIFO inventory valuation

$000 Revenue 75,000

Cost of sales (37,600)

Gross profit 37,400

Other operating expenses (9,000)

Profit from operations 28,400

Investment income

Finance cost (4,000)

Profit before tax 24,400

Income tax expense (7,000)

Net profit for the period 17,400

The change from LIFO to FIFO would be a change of accounting policy. Under IAS 8 (revised) the effects of such a change should be applied retrospectively and comparative figures restated, with the opening balance of retained profits adjusted.

Working – cost of sales $000 As originally stated 38,000

Increase to opening Inventory 00,500

Increase to closing Inventory (900)

As restated 37,600

Page 189: elliottv10

Barry Elliott and Jamie Elliott: Financial Accounting and Reporting (tenth edition) – Instructor’s Manual

189 © Pearson Education Limited 2006

C H AP T E R 2 0

Chapter 20: Question 1 – MACTAR

(£m) M1 M6 M62 Costs to date 2.1 0.3 2.3

Future costs 0.3 1.1 0.8

2.4 1.4 3.1

Contract sum 3.0 2.0 2.75*

Expected profit/(loss) 0.6 0.6 (0.35)

* assumes a 10% increase in contract price is allowed and negotiated.

Recognised profit

Value of work certified 1.8 0.1 1.3

% of work certified to total 60% 5% 47%

So Recognised profit 0.36 Note 1 Note 2

Note 1

Contract M6 is probably at too early a stage of completion to recognise any profit.

Note 2

The anticipated loss on contract M62 must be recognised in full on the grounds of prudence.

£m M1 M6 M62

Recorded as revenue 1.8 0.1 1.3

C of S balance 1.44 0.1 1.65

Profit 0.36 nil (0.35)

Balance sheet work-in-progress

Costs to date 2.1 0.3 2.3

Plus recognised profits 0.36 –

Less: recognised losses (0.35)

Less: progress billings (1.75) (0.1) (1.0)

Closing balance 0.71 0.2 0.95

Receivables

Progress billings 1.75 0.1 1.0

Less: Payment received (1.5) nil (0.75)

Closing balance 0.25 0.1 0.25

Page 190: elliottv10

Barry Elliott and Jamie Elliott: Financial Accounting and Reporting (tenth edition) – Instructor’s Manual

190 © Pearson Education Limited 2006

Thus the overall profit on these three contracts is £0.01m (£10,000) for the year. This is of course a prudent view as one of the projects (M6) has only just started, one project is set for a cost over-run (M62) and one contract is very nearly complete (M1).

Page 191: elliottv10

Barry Elliott and Jamie Elliott: Financial Accounting and Reporting (tenth edition) – Instructor’s Manual

191 © Pearson Education Limited 2006

Chapter 20: Question 2 – Lytax Ltd

Contract No. 1 2 3 4 5

£000 £000 £000 £000 £000

Contract price 1,100 950 1,400 1,300 1,200

Costs incurred to date 664 535 810 640 1,070

Estimated further cost

to complete 106 75 680 800 165

Estimated cost of post-

completion work 30 10 45 20 5

800 620 1,535 1,460 1,240

Estimated profit/(loss)

On contracts 300 330 (135) (160) (40)

Recognised profit/(loss)

580/800 × 300 218

470/620 × 330 250

(135) (160) (40)

Notes 1 Recognised profit on the profitable contracts is taken as the proportion that costs taken to

revenue bear to total anticipated costs. Other sensible proportions would be acceptable. 2 Losses on unprofitable contracts are recognised in full.

The balance sheet work-in-progress balance will show: 1 2 3 4 5 £000 £000 £000 £000 £000 Costs incurred to date 664 535 810 640 1,070

Recognised profits less 218 250 (135) (160) (40)

foreseeable losses

Progress billings:

Received (615) (680) (615) (385) (722)

Awaited (60) (40) (25) (200) (34)

Retained (75) (80) (60) (65) (84)

Closing balance 132 (15) (25) (170) 190

The positive balances on contracts 1 and 5, totalling £322,000, will be presented as an asset. The negative balances on contracts 2, 3 and 4, totalling £210,000, will be presented as a liability. The difference between total progress billings and total receipts will be shown as a receivable.

Page 192: elliottv10

Barry Elliott and Jamie Elliott: Financial Accounting and Reporting (tenth edition) – Instructor’s Manual

192 © Pearson Education Limited 2006

Chapter 20: Question 3 – Beavers Ltd

(a) Contract Account

£ £ Wages 91,000

Materials 36,000

Other costs 18,000

HO costs (see Note) 6,000 Progress billings 150,000

Plant (12/15 × £8,000) 6,400 Materials c/f 3,000

Recognised profit 40,500 WIP c/f 44,900

197,900 197,900

Note

It is assumed that the Head Office costs are associated with the provision of contract-related services that cannot be directly allocated to a specific contract. In these circumstances IAS 11 allows their inclusion in ‘contract costs’. Any apportionment of general Head Office costs would not be permitted under IAS 11.

(b) Balance sheet extract as at 30 June 20X7

Current assets

Inventory – materials 3,000

Long-term contract balance 44,900

(c) Calculation of profit

Contract price 240,000

Costs to date

[157,400 – 3,000 material on site] 154,400

Estimated further costs to complete

[10,000 + 12,000 + 8,000 + 1,600 plant] 31,600

186,000

Estimated profit 54,000

Recognised profit (say)

£180,000/240,000 × 54,000 = £40,500

Amount recoverable on long-term contract (180,000 – 150,000) £30,000

Page 193: elliottv10

Barry Elliott and Jamie Elliott: Financial Accounting and Reporting (tenth edition) – Instructor’s Manual

193 © Pearson Education Limited 2006

Chapter 20: Question 4 – Newbild SA

(a) Contract Account

€ € Materials – Stores 13,407 Progress billing 134,800

Materials – Site 73,078 (see Note)

Wages 39,894

Site expenses 4,815

Administration 3,742

Plant – Depreciation 5,160 Materials Bal c/f 5,467

Subtotal 140,096 WIP Bal c/f 22,928

Recognised profit 23,099

163,195 163,195

Note

The progress billing is the amount received from the customer grossed up by the 15% retention.

Calculation of profit Contract price 780,000

Costs to date [140,096 – 5,467] 134,629

Estimated further costs to complete 490,000

Guarantee work [2.5% of 780,000] 19,500

644,129

Estimated profit 135,871

Recognised profit

The work certified as complete has a value of €134,800 ($114,580/0.85). Therefore the contract is around 17% complete (€134,800/$780,000 is 17.28%). Therefore rec-ognised profit could be 17% × €135,871 = €23,099.

(b) Income statement

Revenue 134,800

Cost of sales 111,701

Profit 23,099

Balance sheet extract Current assets

Inventory – materials 5,467

Long-term contract balance 22,928

Receivables – amount recoverable on long-term contract 20,220

Page 194: elliottv10

Barry Elliott and Jamie Elliott: Financial Accounting and Reporting (tenth edition) – Instructor’s Manual

194 © Pearson Education Limited 2006

Chapter 20: Question 5 – Good Progress SpA

Note that the contract is 25% complete at the end of 20X0 and 40% complete by the end of 20X1. Therefore 15% of the contract was completed in 20X1.

Income statement entries: 20X1 20X0

€ € Revenue 150,000 250,000

Cost of goods sold

112,500 187,500

Profit [15%/25%] 37,500 62,500

Balance sheet entries 20X1 20X0

Current assets € €

Long-term contracts balance

[117,000 – 112,500] 4,500 [265,000 – 262,500] 2,500

Page 195: elliottv10

Barry Elliott and Jamie Elliott: Financial Accounting and Reporting (tenth edition) – Instructor’s Manual

195 © Pearson Education Limited 2006

Chapter 20: Question 6

Solution (a) Water Away

The following extract from the Balfour Beatty 2003 Annual Report deals with such a situation:

Revenue on Aberdeen Environmental Services Limited (AES) is related to the volume and quality of the wastewater processed by the plant. AES therefore takes demand risk and has a fixed asset, which is depreciated over the life of the concession. The revenue is recognised as turnover as it is earned.

Solution (b) Learn A head

The following extract from the Balfour Beatty 2003 Annual Report deals with such a similar situation:

Hospitals and Schools

Balfour Beatty's hospitals and schools concessions receive income based on the avail-ability of the asset, rather than their actual usage. The costs of constructing the asset are therefore accounted for as amounts recoverable on contracts during the course of con-struction and reclassified to contract debtors when construction is complete.

The contract debtor gives rise to an interest income calculation based on an appropriate rate of return for the asset concerned. The income is split into two elements: that relat-ing to the contract debtor, and that relating to the provision of other services such as cleaning and catering. The element of revenue relating to the contract debtor is split be-tween principal repayments, reducing the amount owed to the concession, and interest income, which is credited to the profit and loss account as it is earned. The revenue re-lating to services is recognised as turnover as it is earned, reflecting the continuing provision of services to the concession.

Page 196: elliottv10

Barry Elliott and Jamie Elliott: Financial Accounting and Reporting (tenth edition) – Instructor’s Manual

196 © Pearson Education Limited 2006

C H AP T E R 2 1

Chapter 21: Question 1 – Parent Ltd

(a) Parent Ltd balance sheet as at 1 January 20X1

Ordinary Shares of 1 each 40,500

Retained earnings 4,500

45,000

Investment in Daughter Ltd 10,800

Cash[20,000 – 10,800] 9,200

Other net assets 25,000

45,000

Note: The investment is shown as its fair value of 10,800 and the cash has been reduced by consideration.

Consolidated balance sheet as at January 20X1

Parent Daughter Add Eliminate CBS (Dr)/Cr Ordinary shares 40,500 9,000 49,500 (9,000) 40,500

Retained earnings 4,500 1,800 6,300 (1,800) 4,500

55,800 10,800 55,800 45,000

Investment in

Daughter 10,800 10,800 10,800

Cash 9,200 2,000 11,200 11,200

Other net assets 25,000 8,800 33,800 33,800

55,800 10,800 55,800 Q 45.000

Note: Because the cash paid exactly equalled the value of the net assets acquired, there was no difference on consolidation i.e. no positive or negative goodwill.

(b) Parent Ltd balance sheet as at 1 January 20X7

Ordinary Shares of 1 each (40,500+(10,800/2) 45,900

Share premium 5,400

Retained earnings 4,500

55,800

Page 197: elliottv10

Barry Elliott and Jamie Elliott: Financial Accounting and Reporting (tenth edition) – Instructor’s Manual

197 © Pearson Education Limited 2006

Investment in Daughter Ltd 10,800

Cash 20,000

Other net assets 25,000

55,800

Note: The investment is shown as its fair value of 10,800 and the shares are issued at their fair value of 5,400 par value and 5,400 premium.

Consolidated balance sheet as at January 20X7

Parent Daughter Add Eliminate CBS (Dr)/Cr

Ordinary shares 45,900 9,000 54,900 (9,000) 40,500

Share premium 5,400 5,400 5,400

Retained earnings 4,500 1,800 6,300 (1,800) 4,500

55,800 10,800 66,600 55,800

Investment in

Daughter 10,800 10,800 10,800

Cash 20,000 20,000 20,000

Other net assets 25,000 10,800 35,800 ___ 35,800

55,800 10,800 66,800 Q 45.000

Note: Because the value of the shares issued exactly equalled the value of the net assets ac-quired, there was no difference on consolidation i.e. no positive or negative goodwill.

Page 198: elliottv10

Barry Elliott and Jamie Elliott: Financial Accounting and Reporting (tenth edition) – Instructor’s Manual

198 © Pearson Education Limited 2006

Chapter 21: Question 2 – Parent Ltd

(a) Parent Ltd balance sheet as at 1 January 20X7

Ordinary Shares of 1 each 40,500

Retained earnings 4,500

45,000

Investment in Daughter Ltd 16,200

Cash [20,000 – 16,200] 3,800

Other net assets 25,000

45,000

Note: The investment is shown as its fair value of 16,200 and the cash has been reduced by consideration.

Consolidated balance sheet as at January 20X7

Parent Daughter Add Eliminate CBS (Dr)/Cr

Ordinary shares 40,500 9,000 49,500 (9,000) 40,500

Retained earnings 4,500 1,800 6,300 (1,800) 4,500

55,800 10,800 55,800 45,000

Investment in

Daughter 16,200 16,200 10,800 5,400

Cash 3,800 2,000 5,800 5,800

Other net assets 25,000 8,800 33,800 ___ 33,800

55,800 10,800 55,800 Q 45.000

Note: Because the cash paid exceeded the value of the net assets acquired, there was a difference on consolidation of 5,400 which appears in the consolidated balance sheet as an asset goodwill – this will be reviewed for possible impairment.

(b) Parent Ltd balance sheet as at 1 January 20X7

Ordinary Shares of 1 each (40,500+(16,200/3) 45,900

Share premium 10,800

Retained earnings 4,500 61,200

Investment in Daughter Ltd 16,200

Cash 20,000

Other net assets 25,000

61,200

Page 199: elliottv10

Barry Elliott and Jamie Elliott: Financial Accounting and Reporting (tenth edition) – Instructor’s Manual

199 © Pearson Education Limited 2006

Note: The investment is shown as its fair value of 16,200 and the shares are issued at their fair value of 5,400 par value and 10,800 premium.

Consolidated balance sheet as at January 20X7

Parent Daughter Add Eliminate CBS

(Dr)/Cr Ordinary shares 45,900 9,000 54,900 (9,000) 45,900

Share premium 10,800 10,800 10,800

Retained earnings 4,500 1,800 6,300 (1,800) 4,500

61,200 10,800 72,000 61,200

Investment in Daughter 16,200 16,200 10,800 5,400

Cash 20,000 20,000 20,000

Other net assets 25,000 10,800 35,800 ___ 35,800

55,800 10,800 72,000 Q 61,200

Note: Because the value of the shares issued exceeded the value of the net assets acquired, there was a difference on consolidation.

Page 200: elliottv10

Barry Elliott and Jamie Elliott: Financial Accounting and Reporting (tenth edition) – Instructor’s Manual

200 © Pearson Education Limited 2006

Chapter 21: Question 3 – Parent Ltd

(a) Parent Ltd balance sheet as at 1 January 20X7

Ordinary Shares of £ 1 each 40,500

Retained earnings 4,500

45,000

Investment in Daughter Ltd 16,200

Cash [20,000 – 16,200] 3,800

Other net assets 25,000

45,000

Note: The investment is shown as its fair value of 16,200 and the cash has been reduced by consideration.

Consolidated balance sheet as at January 20X7

Parent Daughter Add Eliminate CBS (Dr)/Cr

Ordinary shares 40,500 9,000 49,500 (9,000) 40,500

Retained earnings 4,500 1,800 6,300 (1,800) 4,500

55,800 10,800 55,800 45,000

Investment in

Daughter 16,200 16,200 10,800

Revaluation increase (1,200) 4,200

Cash 3,800 2,000 5,800 5,800

Other net assets 25,000 8,800 33,800 1,200 35,000

55,000 10,800 55,800 Q 45.000

Note:

1. The net assets in the CBS will be increased by 1,200.

2. The fair value of the shares issued (16,200) exceeded the fair value of the net assets ac-quired (12,000). This difference on consolidation will be reported as goodwill and reviewed for impairment.

(b) Parent Ltd balance sheet as at 1 January 20X7

Ordinary Shares of 1 each (40,500+(16.200/3) 45,900

Share premium 10,800

Retained earnings 4,500

61,200

Investment in Daughter Ltd 16,200

Page 201: elliottv10

Barry Elliott and Jamie Elliott: Financial Accounting and Reporting (tenth edition) – Instructor’s Manual

201 © Pearson Education Limited 2006

Cash 20,000

Other net assets 25,000

61,200

Note: The investment is shown as its fair value of 16,200 and the shares are issued at their fair value of 5,400 par value and 10,800 premium.

Consolidated balance sheet as at January 20X7

Parent Daughter Add Eliminate CBS (Dr)/Cr

Ordinary shares 45,900 9,000 54,900 (9,000) 45,900

Share premium 10,800 10,800 10,800

Retained earnings 4,500 1,800 6,300 (1,800) 4,500

61,200 10,800 72,000 61,200 Investment in

Daughter 16,200 16,200 10,800

Revaluation increase (1,200) 4,200

Cash 20,000 20,000 20,000

Other net assets 25,000 10,800 35,800 1,200 37,000

55,800 10,800 72,000 Q 61,200

Note:

1. The net assets in the CBS will be increased by 1,200.

2. The fair value of the shares issued (16,200) exceeded the fair value of the net assets ac-quired (12,000). This difference on consolidation will be reported as goodwill and reviewed for impairment.

Page 202: elliottv10

Barry Elliott and Jamie Elliott: Financial Accounting and Reporting (tenth edition) – Instructor’s Manual

202 © Pearson Education Limited 2006

Chapter 21: Question 4 – Parent Ltd

Parent Ltd balance sheet as at 1 January 20X7

Ordinary Shares of £ 1 each 40,500

Retained earnings 4,500

45,000

Investment in Daughter Ltd 6,000

Cash [20,000 – 6,000] 14,000

Other net assets 25,000

45,000

Note: The investment is shown as its fair value of 6,000 and the cash has been reduced by con-sideration.

Consolidated balance sheet as at January 20X7

Parent Daughter Add Eliminate CBS (Dr)/Cr

Ordinary shares 40,500 9,000 49,500 (9,000) 40,500

Retained earnings 4,500 1,800 6,300 (1,800) 4,500

55,800 10,800 55,800 45,000

Investment in Daughter 6,000 6,000 10,800 (4,800)

Cash 14,000 2,000 16,000 16,000

Other net assets 25,000 8,800 33,800 ___ 33,800

55,800 10,800 55,800 Q 45.000

Note: Because the cash paid was less than the value of the net assets acquired, there was a credit difference on consolidation i.e. negative goodwill which will be credited to the retained earnings.

Page 203: elliottv10

Barry Elliott and Jamie Elliott: Financial Accounting and Reporting (tenth edition) – Instructor’s Manual

203 © Pearson Education Limited 2006

Chapter 21: Question 5 – Parent Ltd

Parent Ltd balance sheet as at 1 January 20X7

Ordinary Shares of £ 1 each 40,500

Retained earnings 4,500

45,000

Investment in Daughter Ltd 9,000

Cash [20,000 – 9,000] 11,000

Other net assets 25,000

45,000

Note: The investment is shown as its fair value of 9,000 and the cash has been reduced by con-sideration.

Consolidated balance sheet as at January 20X7

Parent Daughter Add Eliminate CBS (Dr)/Cr

Ordinary shares 40,500 9,000 49,500 (6,750) a

(2,250) b 40,500

Retained earnings 4,500 1,800 6,300 (1,350) a

(450) b 4,500

Minority interest 2,250 b

450 b 2,700 45,000 10,800 55,800 47,700

Investment in

Daughter 9,000 9,000 6,750 a

1,350 a 900

Cash 11,000 2,000 13,000 13,000

Other net assets 25,000 8,800 33,800 ___ 33,800

45,000 10,800 55,800 Q 47,700

Note: Because the cash paid was more than the value of the net assets acquired, there was a debit difference on consolidation of 900.

(a) represents the elimination of the shares and reserves of the company acquired against the invest-ment in the company acquired.

(b) Represents the transfer to the minority shareholders their 25% interest in the net assets of 10,8700 in Daughter Ltd.

Page 204: elliottv10

Barry Elliott and Jamie Elliott: Financial Accounting and Reporting (tenth edition) – Instructor’s Manual

204 © Pearson Education Limited 2006

Chapter 21: Question 6

(a) Balance sheet as at 30 June 20X2

Property 43,400

Plant and equipment 12,320 Current assets

Inventory 51,324

Trade receivables 22,829

Cash 63,500

137,713

Current liabilities

Trade payables 63,700

Income tax 6,440

70,140

Net current assets 67,573

123.293 Share capital 56,000

Retained earnings 67,293

123.293

Reserves at date of acquisition

Investment 151,200

Less shares 50,400 Goodwill 100,800

85,680

Reserves 15.120 Step 1: Calculate the % ownership Proposed dividend in CBS 11,760

Proposed dividend in Parent 11,200

Minority dividend 560 10%

Dividend receivable by Parent 5,040 90%

Step 2: Calculate the retained earnings balance Consolidated balance 79,884

Less Parent 35,280

44,604

Page 205: elliottv10

Barry Elliott and Jamie Elliott: Financial Accounting and Reporting (tenth edition) – Instructor’s Manual

205 © Pearson Education Limited 2006

Add Profit on stock (4,200 – 3,360) 840

45,444

Add Minority interest (10% of 50,493 or 1/9 of 45,444)) 5,049

Add Pre-acquisition

Parent (90% o £16,800) 15,120

Minority (10% of 16,800) Subsidiary retained earnings 1,680

67,293

Step 3: Reconcile the minority interest Shares 5,600 Retained earnings post-acquisition 5,049

Retained earnings pre-acquisition 1,680

12,329

Worksheet

Non-current assets

Group Parent Subsidiary Adjustment

Property 127,400 84,000 43,400 43,400

Plant 62,720 50,400 12,320 12,320

Current assets

Inventory 121,604 71,120 50,484 840 51,324

Receivables 70,429 51,800 18,629 4,200 22,829

Cash 24,360 24,360 39,200 63,560

Current liabilities

Payables 140,420 80,920 59,500 4,200 63,700

Income tax 27,160 20,720 6,440 6,440

Page 206: elliottv10

Barry Elliott and Jamie Elliott: Financial Accounting and Reporting (tenth edition) – Instructor’s Manual

206 © Pearson Education Limited 2006

Chapter 21: Question 7 – Rouge plc

Rouge plc Balance Sheet as at 1 January 20X0

ASSETS £ Million

Non-current assets

Property, plant and equipment [100 + 60] 160

Goodwill [132 – 100] 32

Current assets [80 + 70] 150

342

Common £10 shares 200

Retained earnings 52

Share capital and reserves 252

Current liabilities 90

342

Page 207: elliottv10

Barry Elliott and Jamie Elliott: Financial Accounting and Reporting (tenth edition) – Instructor’s Manual

207 © Pearson Education Limited 2006

Chapter 21: Question 8 – Ham plc

(a)

Ham plc Balance sheet as at January 20X0

ASSETS £000

Non-current assets Property, plant and equipment [250 + 100] 350

Goodwill [90 – 110] (20)

Current assets [100 + 70] 170

500

Common £5 shares 200

Retained earnings 160

Share capital and reserves 360

Current liabilities 140

500

(b) See discussion in chapter.

Following recent changes in the treatment of goodwill, negative goodwill will be taken to in-come statement immediately.

Page 208: elliottv10

Barry Elliott and Jamie Elliott: Financial Accounting and Reporting (tenth edition) – Instructor’s Manual

208 © Pearson Education Limited 2006

Chapter 21: Question 9 – Berlin plc

Berlin plc Balance sheet as at 1 January 20X0

(a) (b) Cash acquisition Share exchange ASSETS £000 £000 Non-current assets Property, plant and equipment 250 250

Investment in Hanover 100 100

Current assets 50 150

400 500

Common £5 shares 200 250

Additional paid-in capital – 50

Retained earnings 80 80

Share capital and reserves 280 380

Current liabilities 120 120

400 500

Page 209: elliottv10

Barry Elliott and Jamie Elliott: Financial Accounting and Reporting (tenth edition) – Instructor’s Manual

209 © Pearson Education Limited 2006

Chapter 21: Question 10 – Bleu plc

Bleu plc Balance sheet as at January 20X0

ASSETS £ Million

Non-current assets Property, plant and equipment [150 + 120] 270

Goodwill [210 – (80% × 180) ] 66

Current assets [108 + 105] 213

549

Common £10 shares 300

Retained earnings 78

Share capital and reserves 378

Minority interest [20% × 180] 36

Current liabilities [90 + 45] 135

549

Page 210: elliottv10

Barry Elliott and Jamie Elliott: Financial Accounting and Reporting (tenth edition) – Instructor’s Manual

210 © Pearson Education Limited 2006

Chapter 21: Question 11 – Base plc

Base plc Balance sheet as at January 20X0

ASSETS £000

Non-current assets Property, plant and equipment [250 + 120] 370

Goodwill [90 – (60% × 110) + (40% × 20)] 12

Current assets [100 + 70] 170

Total assets 552

Common £5 shares 200

Retained earnings 160

Share capital and reserves 360

Minority interest [(40% × 110) + (40% × 20)] 52

Current liabilities [80 + 60] 140

Total equity and liabilities 552

Page 211: elliottv10

Barry Elliott and Jamie Elliott: Financial Accounting and Reporting (tenth edition) – Instructor’s Manual

211 © Pearson Education Limited 2006

C H AP T E R 2 2

Chapter 22: Question 1 – Sweden

Sweden Balance Sheet as at 31 December 20X1

ASSETS Krm

Non-current assets Property, plant and equipment [264 + 120] 384

Goodwill [200 – (110 + 10 + 70) – 2] 8

Current assets [160 + 140] 300

Total assets 692

Common Kr10 shares 400

Revaluation reserve 20

Retained earnings [104 +10 – 2] 112

Share capital and reserves 532

Current liabilities [100 + 60] 160

Total equity and liabilities 692

Page 212: elliottv10

Barry Elliott and Jamie Elliott: Financial Accounting and Reporting (tenth edition) – Instructor’s Manual

212 © Pearson Education Limited 2006

Chapter 22: Question 2 – Summer plc

Summer plc Balance Sheet as at 31 December 20X1

ASSETS £000 Non-current assets Property, plant and equipment [200 + 200] 400

Goodwill [141 – 60% (20 + 35 + 160) – 1] 11

Current assets [100 + 140] 240

651

Common £5 shares 175

Additional paid-in capital 25

Retained earnings [161 + 60% (40 – 35) – 1] 163

Share capital and reserves 363

Minority interest [40% × 220] 88

Current liabilities [80 + 120] 200

651

Page 213: elliottv10

Barry Elliott and Jamie Elliott: Financial Accounting and Reporting (tenth edition) – Instructor’s Manual

213 © Pearson Education Limited 2006

Chapter 22: Question 3 – Gold plc

Gold plc Balance Sheet as at 31 December 20X1

ASSETS £

Non-current assets Fixed assets, including land [82,300 + 108,550 + 3,000] 193,850

Goodwill (Note 1) 1,240

Current assets Inventories [23,200 +10,000 – 300] 32,900

Other current assets [5,000 + 7,500] 12,500

Total assets 240,490

Common £10 shares 55,000

Preferred shares 10,000

Additional paid-in capital 5,000

Retained earnings

[(75,000 + 75% (21,200 – 16,000)) – 300 – 310)] 78,290

Share capital and reserves 148,290

Minority interest (Note 2) 26,950

Non-current liabilities [12,500 +14,000] 26,500

Current liabilities Bond interest payable [625 + 700] 1,325

Other current liabilities [18,550 + 18,875] 37,425 38,750

240,490

Note 1: Goodwill £ £

Investment in Silver 46,000

Acquired 75% × 24,000 18,000

30% × 20,000 6,000

20% × 17,500 3,500

27,500

75% × 3,600 2,700

75% × 3,000 2,250

75% × 16,000 12,000 44,450

Goodwill 1,550

Page 214: elliottv10

Barry Elliott and Jamie Elliott: Financial Accounting and Reporting (tenth edition) – Instructor’s Manual

214 © Pearson Education Limited 2006

Impairment @ 20% = £310

Goodwill at 31.12.20×1 = £1,550 – 310 = £1,240

Note 2: Minority interest £ 25% × 24,000 6,000

70% × 20,000 14,000

25% × 3,600 900

25% × 3,000 750

25% × 21,200 5,300

26,950

Page 215: elliottv10

Barry Elliott and Jamie Elliott: Financial Accounting and Reporting (tenth edition) – Instructor’s Manual

215 © Pearson Education Limited 2006

C H AP T E R 2 3

Chapter 23: Question 1 – Bill plc

Bill SA

Consolidated Income Statement for the year ended 31 December 20X1

£ Sales [300,000 + 180,000 – 12,000] 468,000

Cost of sales [90,000 + 90,000 – 12,000 + 2,000] 170,000

Gross profit 298,000

Expenses [88,623 + 60,000] 148,623

Impairment of goodwill 3,000

Profit before taxation 146,377

Taxation [21,006 + 9,000] 30,006

Profit after taxation 116,371

Minority interest [(20% (21,000 – 4,500)) + (90% x 4,500)] 7,350

Profit attributable to the group 109,021

Dividend paid 60,000

Retained profit for the year 49,021

Retained profit brought forward

[104,004 + 80% (81,000 – 45,000)] 132,804

Retained profit carried forward 181,825

Page 216: elliottv10

Barry Elliott and Jamie Elliott: Financial Accounting and Reporting (tenth edition) – Instructor’s Manual

216 © Pearson Education Limited 2006

Chapter 23: Question 2 – Morn Ltd

Morn Ltd Consolidated Income Statement for the year ended 31 December 20X1

£ Gross profit [360,000 + 180,000)] 540,000

Expenses [120,000 + 110,000] 230,000

Profit before taxation 310,000

Taxation [69,000 + 18,000] 87,000

Profit after taxation 223,000

Minority interest [10% × 52,000] 5,200

Profit after taxation belonging to the group 217,800

Dividends paid 120,000

Retained profit for the year 97,800

Page 217: elliottv10

Barry Elliott and Jamie Elliott: Financial Accounting and Reporting (tenth edition) – Instructor’s Manual

217 © Pearson Education Limited 2006

Chapter 23: Question 3 – River plc

River A/S

Consolidated Income Statement for the year ended 31 December 20X1 £ Sales [100,000 + [(9/12 × 60,000)] 145,000

Cost of sales [30,000 +[(9/12 × 30,000]) 52,500

Gross profit 92,500

Expenses [20,541 + (9/12 × 15,000)] 31,791

Interest payable on 5% bonds [9/12 × (5,000 – 500)] 3,375

Impairment of goodwill 4,000

Profit before taxation 53,334

Taxation [7,002 + (9/12 × 3,000)] 9,252

Profit after taxation 44,082

Minority Interest [10% × 7,000 × 9/12] 525

Profit after taxation belonging to the group 43,557

Dividends paid 20,000

23,557

Profit and loss account balance

brought forward from previous years

34,668

58,225

Page 218: elliottv10

Barry Elliott and Jamie Elliott: Financial Accounting and Reporting (tenth edition) – Instructor’s Manual

218 © Pearson Education Limited 2006

Chapter 23: Question 4 – Mars plc

Balance Sheet as at 31 December 20X2

Assets £ £ Non-current assets [330,000 + 157,500] 487,500

Goodwill 37,100 Current assets Inventories [(225,000 + 67,500) – 3,000] 289,500

Trade receivables [180,000 + 90,000] 270,000

Bank [36,000 + 18,000] 54,000 613,500

Total assets 1,138,100

Equity and liabilities

Capital and reserves Issued capital 196,000

General reserve [245,000 + 10,800] 255,800

Retained earnings [222,000 + 44,000] 266,000

717,800

Minority interest 51,300

Current liabilities Trade payables [283,500 + 40,500] 324,000

Taxation [31,500 + 13,500] 45,000

369,000

1,138,100

W1: Cancel inter-company balances

• Current accounts of £22,500

• Dividends receivable in Mars of £9,000 cancels with £9,000 of the dividends payable in Jupiter, leaving £2,250 payable to the minority interest.

Consolidated Income Statement for the year ending 31 December 20X2

£ Sales [1,440,000 + 270,000 – 18,000] 1,692,000

Cost of sales [1,045,000 + 135,000 – 18,000 + 3,000] 1,165,000

Gross profit 527,000

Expenses [123,500 + 90,000] 213,500

Profit before tax 313,500

Taxation [31,500 + 13,500] 45,000

Page 219: elliottv10

Barry Elliott and Jamie Elliott: Financial Accounting and Reporting (tenth edition) – Instructor’s Manual

219 © Pearson Education Limited 2006

Profit after tax 268,500

Minority interest [20% × 31,500] 6,300

262,200

Dividend paid 180,000

Retained profit 82,200

Accumulated profit brought forward

[(156,000 + 80% (114,750 – 80,000)) 183,800

266,000

W2: Goodwill

£ £ Investment in Jupiter 187,500

£1 Ordinary shares [80% × 90,000] 72,000

Accumulated profits [80% × 80,000] 64,000

General Reserve [80% × 18,000] 14,400

150,400

Goodwill 37,100

W3: Unrealised profit on inter-company sales

50/150 × 18,000 = 6,000. Only half the stock is unsold at the year end so 6,000/2 is the provi-sion required against the closing stock figure.

W4: The income statement of Jupiter

£ £ Balance at 31/12/20X2 as per the balance sheet 135,000

Pre-acquisition profit held by Mars 64,000

Minority interest [20% × 135,000] 27,000 91,000

44,000

W5: The income statement of Mars

£ Balance at 31/12/20X2 as per the balance sheet 225,000

Less: Provision for unrealised profit 3,000

222,000

Page 220: elliottv10

Barry Elliott and Jamie Elliott: Financial Accounting and Reporting (tenth edition) – Instructor’s Manual

220 © Pearson Education Limited 2006

W6: The minority interest

£ 20% × 90,000 Shares 18,000

20% × 31,500 General reserve 6,300

20% × 135,000 27,000

51,300

W7: Jupiter general reserve

£ Balance at 31/12/20X2 as per balance sheet 31,500

Less:

Mars share of pre-acquisition 80% × 18,000 = (14,400)

Minority interest 20% × 31,500 (6,300)

10,800

Page 221: elliottv10

Barry Elliott and Jamie Elliott: Financial Accounting and Reporting (tenth edition) – Instructor’s Manual

221 © Pearson Education Limited 2006

Chapter 23: Question 5 – Red Ltd

Consolidated Balance Sheet as at 31 December X2

Assets $ $ Non-current assets Tangible [120,000 + 70,000] 190,000

Intangible – Goodwill (W1) 51,500

Current assets

Inventories [100,000 + 30,000 – 1,500] (W2) 128,500

Trade receivables [80,000 + 40,000] 120,000

Bank [16,000 + 8,000] 24,000 272,500

Total assets 514,000

Equity and liabilities Capital and reserves Issued capital 176,000

General reserve [20,000 + (75% × (14,000 – 8,000))] 24,500

Retained earnings [(100,000 + (75% × (60,000 – 30,000)

– 1,500))] 121,375

321,875

Minority interest 25% × (114,000 – 1,500) 28,125

Current liabilities

Creditors [125,996 + 18,000] 143,996

Taxation [14,004 + 6,000] 20,004 164,000

514,000

Income Statement for the year ending 31 December X2

£ Sales [200,000 + 120,000 –12,000] 308,000

Cost of sales [60,000 + 60,000 –12,000 + 1,500 109,500

Gross profit 198,500

Expenses [59,082 + 40,000] (99,082)

Profit before tax 99,418

Taxation [14,004 + 6,000] 20,004

Profit after tax 79,414

Minority interest [25% × (14,000 – 1,500] 3,125

76,289

Dividend paid 40,000

Page 222: elliottv10

Barry Elliott and Jamie Elliott: Financial Accounting and Reporting (tenth edition) – Instructor’s Manual

222 © Pearson Education Limited 2006

Retained profit 36,289

Accumulated profit brought forward (W3) 85,086

121,375

W1: Goodwill

$

Investment in Pink 110,000

$1 Common shares [75% × 40,000] 30,000

Accumulated profits [75% × 30,000] 22,500

General Reserve [75% × 8,000] 6,000

58,500

Goodwill 51,500

W2: Unrealised profit on inter-company sales

Mark-up = 9,000 × 1/3 = $3,000 Only half the stock is unsold at the year end so $3,000/2 is the provision required against the closing stock figure = $1,500 W3: Red 69,336

Pink [75% × (51,000 – 30,000)] 15,750

85,086

Page 223: elliottv10

Barry Elliott and Jamie Elliott: Financial Accounting and Reporting (tenth edition) – Instructor’s Manual

223 © Pearson Education Limited 2006

Chapter 23: Question 6 – Try plc

Try Hard CPL £000 £000 £000 Profit before tax 80 56 136.00

Tax 42 28.60 70.60

38 27.40 65.40

Minority interest [25%] 6.85 6.85

38 20.55 58.55

Dividends 20 . 20.00

Carried forward 18 20.55 38.55

Page 224: elliottv10

Barry Elliott and Jamie Elliott: Financial Accounting and Reporting (tenth edition) – Instructor’s Manual

224 © Pearson Education Limited 2006

Chapter 23: Question 7 – Mother plc

(a) Consolidated income statement for the year ended 31 December 20x6 Mother Daughter CPL

000 000 000

Profit before tax 300 100 400

Less Intra-company dividend (20) (20)

Income tax (120) (30) (150)

Profit after tax 160 70 230

Intra-group dividend 20 (20)

180 50 230

Transfer to general reserve (30) (20) (50)

Dividends (50) (50)

Retained 100 30 130

Balance b/f at 1.1.20x6 50 10 60

150 40 190

Consolidated balance sheet as at 31 December 20x6

Mother Daughter CBS

000 000 000

Non-current assets 300 150 450

Investment in Daughter plc 200

Net current assets 280 110 390

780 260 840

Share capital 500 200 500

General reserve 130 20 150

Retained earnings 150 40 190

780 260 840

Page 225: elliottv10

Barry Elliott and Jamie Elliott: Financial Accounting and Reporting (tenth edition) – Instructor’s Manual

225 © Pearson Education Limited 2006

(b) The consolidated balance sheet:

• The parent company has 100% control of the subsidiary.

• The subsidiary was acquired at the date of incorporation when the net assets were equal to share capital with both stated at £200,000, there was therefore no goodwill.

• The CBS is formed by the aggregation of the assets and the reserves. Only the share capital of the parent company is included.

The consolidated income statement:

• Items are aggregated except for dividend.

• Intra-group dividends are cancelled.

• Only the dividend of the parent company is shown.

• Reserves are aggregated because they are post-acquisition

Page 226: elliottv10

Barry Elliott and Jamie Elliott: Financial Accounting and Reporting (tenth edition) – Instructor’s Manual

226 © Pearson Education Limited 2006

Chapter 23: Question 8 – Mother plc

(a) Consolidated income statement for the year ended 31 December 20x6 Mother Daughter CPL

000 000 000

Profit before tax 300 100 400

Less Intra-company dividend (16) (16)

Income tax (120) (30) (150)

Profit after tax 164 70 234

Less minority interest 20% of 70 (14) (14)

164 56 220

Intra-group dividend 16 (16)

180 50 220

Transfer to general reserve (30) 80% of 20 (16) (46)

Dividends (50) (50)

Retained 100 24 124

Balance b/f at 1.1.20x6 50 80% of 10 8 58

150 80% of 40 32 182

Consolidated balance sheet as at 31 December 20x6

Mother Daughter CBS

000 000 000

Non-current assets 300 150 450

Goodwill (200,000-160,000) 40

Investment in Daughter plc 200

Net current assets 280 110 390

780 260 880

Share capital 500 200 500

General reserve 130 20 146

Retained earnings 150 40 182

Minority interest 20% of 260,000 52

780 260 880

Page 227: elliottv10

Barry Elliott and Jamie Elliott: Financial Accounting and Reporting (tenth edition) – Instructor’s Manual

227 © Pearson Education Limited 2006

(b) The consolidated balance sheet:

• The parent company holds 80% of the shares • There is a 20% minority interest

• In CPL it is 20% of post tax profit

• In the CBS it is 20% of the closing net assets of 260,000

• The intragroup dividend is 80% of the Daughter plc dividend i.e. 80% of 20,000

• The retained earnings is then 80% of the closing balance i.e. 80% of 40,000.

• Goodwill has arisen because Mother paid more than the fair value of the net assets i.e. 200,000 for net assets of 160,000.

• The CBS is formed by the aggregation of the assets and the reserves. Only the share capital of the parent company is included.

The consolidated income statement:

• Items are aggregated except for dividend.

• Intra-group dividends are cancelled.

• Only the dividend of the parent company is shown.

• Reserves are aggregated because they are post-acquisition

Page 228: elliottv10

Barry Elliott and Jamie Elliott: Financial Accounting and Reporting (tenth edition) – Instructor’s Manual

228 © Pearson Education Limited 2006

Chapter 23: Question 9 – Mother plc

(a) Consolidated income statement for the year ended 31 December 20X6

Mother Daughter CPL 000 000 000

Retained 150 30 130

Balance b/f at 1.1.20X6 50 5 55

150 (Note 1) 35 185

Consolidated balance sheet as at 31 December 20X6

000

Non-current assets 450

Net current assets 390

840

Share capital 500

Negative goodwill

[Cost £200,000 - net assets £220,000] 20 (Note 2)

General reserve 135 [only includes post-acquisition]

Retained earnings 185

840

(b)

Note 1. The group share of Daughter's retained profit is 100% of the post-acquisition profits i.e. 100% of 40,000 – 5,000 that existed at 1.1.20X2.

Note 2. The price paid is less than the fair value of the net assets. This can be attributed either to a bargain purchase perhaps because the vendor needs to achieve a quick sale or to expectation of future losses whereby the purchase price has been reduced to take account of future costs, such as reorganisation costs, or losses that do not represent identifiable liabilities at the balance sheet date.

Page 229: elliottv10

Barry Elliott and Jamie Elliott: Financial Accounting and Reporting (tenth edition) – Instructor’s Manual

229 © Pearson Education Limited 2006

Chapter 23: Question 10 – Mother plc

(a) Consolidated income statement for the year ended 31 December 20X6

Mother Daughter CPL £000 £000 £000

Retained 100 24 124

Bal b/f at 1.1.20X6 50 [80% × 5] 4 54

150 (Note 1) 28 178

Note 1. The group share of Daughter's retained profit is 80% of the post-acquisition profits i.e.

80% of 40,000 – 5,000 that existed at 1.1.20X2.

Consolidated balance sheet as at 31 December 20X6

000

Goodwill

[200,000 – 80% of 220,000] 24

Non-current assets 450

Net current assets 390

864

Share capital 500

General reserve [130,000 + 80% of 5,000] 134

Income statement 178

Minority interest [20% of 260,000] 52

864

(b) In this case a minority interest is recorded representing the minority's 20% interest in the net assets at the balance sheet date which are under the control of the majority shareholder.

Page 230: elliottv10

Barry Elliott and Jamie Elliott: Financial Accounting and Reporting (tenth edition) – Instructor’s Manual

230 © Pearson Education Limited 2006

Chapter 23: Question 11 – Tailor Ltd

Plant [207,900–184,900] 23,000

Depreciation [44,000–32,000] 12,000 11,000

Inventory [70,000 – 63,000 + 300 unrealised profit] 7,300

Receivables [25,000–23,000] 2,000 9,300

20,300

Less:

Payables [58,000 – 56,000] 2,000

Overdraft [6,000–0] 6,000 8,000

12.300

Capital 7,000

Reserves 5,300

12,300

Reconcile the reserves: (1) Cost of control Cash for investment - per balance sheet 9,100

Capital reserve - per Note 2 in question 800

Less: 9,900

Shares (6,300)

Reserves as the difference 3.600

Reserves of 3,600 is a 90% interest (i.e. 6300/7000 × 100)

Therefore 100% interest was 4,000

(2) Calculate distributable profits

Minority interest per consolidated balance sheet 1,200

Less Shares (700)

Post-dividend reserve 500

Dividend (1/9 of 900 in parent balance sheet) 100

Pre dividend reserve 600

Less Pre-acquisition (1/9 of 3,600 from Working 1 above) (400)

200

10% is £200, therefore 100% is £2,000

Page 231: elliottv10

Barry Elliott and Jamie Elliott: Financial Accounting and Reporting (tenth edition) – Instructor’s Manual

231 © Pearson Education Limited 2006

(3) Calculate the reserve of subsidiary

Profits were from (2) above 2,000

Add profit on stock (inter-group) 300

2,300

Less dividend (1,000)

Retained 1,300

Balance b/f 4,000

Balance c/f 5.300 (4) Treatment of reserves on consolidation

Balance 5,300

Less:

Inventory adjustment 300

Minority interest 500

Pre-acquisition 3,600

Post-acquisition 900

5.300

Page 232: elliottv10

Barry Elliott and Jamie Elliott: Financial Accounting and Reporting (tenth edition) – Instructor’s Manual

232 © Pearson Education Limited 2006

Chapter 23: Question 12 – H and S Ltd

(a) % of S Ltd owned by H Ltd = 75% Derived from the minority interest figures

Income statement 170/680 × 100 = 25% Balance sheet 555/2,220 × 100 = 25%

(b) Inter-company sales = 500 The amount eliminated in the Income Statement Parent 4,000 + subsidiary 2,200 - group 5,700

(c) Inventory unrealised profit = 45 Parent 410 + subsidiary 420 - group 785

(d) Inter-company receivables and payables offset/eliminated Receivables: Parent 535 + subsidiary 220 - group 595 = 160 Payables: Parent (300) + subsidiary (260) - group 355 = 225

(e) S Ltd – retained earnings on acquisition = 960 Comprising: Cost 1,700

Less Share capital 570 Goodwill 410 980

Reserves attributable to 75% 720 Total reserves 720/75 × 100 960

Page 233: elliottv10

Barry Elliott and Jamie Elliott: Financial Accounting and Reporting (tenth edition) – Instructor’s Manual

233 © Pearson Education Limited 2006

C H AP T E R 2 4

Chapter 24: Question 1 – Swish

(a) Income Statement for the year ended 31 December 20X3

£ Sales [300,000 + 160,000 – 16,000] 444,000

Cost of Sales [90,000 + 80,000 – 16,000 + 3,200] 157,200

286,800

Expenses [95,000 + 50,000] 145,000

Group profit before taxation 141,800

Share of Associated Company

profits [25% × 30,000] 7,500

Profit before taxation 149,300

Taxation – Group [30,000 + 7,000] 37,000

– Associate [25% × 8,000] 2,000 39,000

110,300

Minority Interest (10% × 23,000) 2,300

108,000

Dividends paid 40,000

Retained profit for the year 68,000

Retained earnings brought forward

[94,000 + ( 90% × 47,000) + (25% × 6,000)] 137,800

205,800

(b) Consolidated Balance Sheet as at 31 December 20X3

Cost Dep’n Net Non-current assets £ £ £ Intangible: Goodwill in subsidiary 17,600

Tangible [120,000 + 110,000] 500,000 270,000 230,000

Investment in Associate [18,000 + 28,000] 46,000

293,600

Current Assets Inventories [120,000 + (60,000 – 3,200)] 176,800

Trade receivables [130,000 + 70,000] 200,000

Page 234: elliottv10

Barry Elliott and Jamie Elliott: Financial Accounting and Reporting (tenth edition) – Instructor’s Manual

234 © Pearson Education Limited 2006

Current account – Handle (Note 1) 3,000

Bank [24,000 + 7,000] 31,000

410,800

Current liabilities Trade payables [132,000 + 25,000] 157,000

Taxation payable 37,000

194,000 216,800

510,400

Financed by £1 Common shares 250,000

General reserve [30,000 + 3,600 + (25% × 4,000] (Note 2) 34,600

Retained earnings

[150,000 – 3,200 + 54,000 + (25% × 20,000) (Note 2) 205,800

490,400

Minority interest 20,000

510,400

Notes 1. The inter-company current account balance with the associated company has not been can-

celled because the associated company is not a member of the group.

2. The group’s share of the retained earnings and general reserve is calculated on the post-acquisition accumulated profits and general reserve of Handle, i.e.

General reserve per Handle balance sheet is 12,000

Pre-acquisition (see question) 8,000

Post-acquisition 4,000

Retained earnings per Handle balance sheet 50,000

Pre-acquisition (see question) 30,000

Post-acquisition 20,000

3. Goodwill in Broom

Cost of investment 140,000

Less 90% of share capital 60,000

General reserve 16,000

Retained earnings 60,000

136,000

90% 122,400

Goodwill 17,600

Page 235: elliottv10

Barry Elliott and Jamie Elliott: Financial Accounting and Reporting (tenth edition) – Instructor’s Manual

235 © Pearson Education Limited 2006

4. Unrealised profit in Inventory

Swish sold Broom 16,000

Remaining unsold at year end 12,800

Profit element 25% 3,200

Page 236: elliottv10

Barry Elliott and Jamie Elliott: Financial Accounting and Reporting (tenth edition) – Instructor’s Manual

236 © Pearson Education Limited 2006

Chapter 24: Question 2 – Stop Group

(a) Consolidated income statement for year to 31 December 20X1

£ Sales 375,000

Cost of sales 125,000

250,000

Expenses 149,750

100,250

Share of profits of Associate [43,750 × 30% × 0.75] 9,844

110,094

Taxation – Group 25,000

– Associate [11,250 × 30% × 0.75] 2,531

27,531

82,563

Dividends paid 50,000

Retained profit for the year 32,563

Balance brought forward 112,500

145,063

(b) Consolidated Balance Sheet at 31 December 20X1

£ Fixed assets at cost 375,000

Depreciation 125,000

250,000

Goodwill 7,000

Investment in Start Ltd [204,375 × 30%] 61,313

Current assets

Inventory 160,000

Trade receivables 165,000

Current a/c Start Ltd 15,000

Bank 12,500

352,500

Page 237: elliottv10

Barry Elliott and Jamie Elliott: Financial Accounting and Reporting (tenth edition) – Instructor’s Manual

237 © Pearson Education Limited 2006

Current liabilities

Trade payables 187,500

Taxation 25,000

212,500

140,000

458,313

Financed by

Ordinary shares of £1 281,000

General reserves [31,500 + (2,500 × 30%)] 32,250

Retained earnings 145,063

458,313

Working 1 Investment in Start [63,250 – 750] (see Note) 62,500

Shares acquired 18,750

Retained earnings [102,500 × 30%] 30,750

General reserve [20,000 × 30%] 6,000

55,500

Goodwill 7,000

Note: The £750 which has correctly been credited to Stop’s investment in Start represents the share of the dividend receivable out of the profits of Start before it became an associate i.e. £10,000 × 30% × 3/12 = 750. This amount is not a profit made by the group and must not there-fore be in the group profit and loss. In effect this adjustment restores the net assets purchased on acquisition to their full amount.

Page 238: elliottv10

Barry Elliott and Jamie Elliott: Financial Accounting and Reporting (tenth edition) – Instructor’s Manual

238 © Pearson Education Limited 2006

Chapter 24: Question 3 – Ant Co

(a) Consolidated Income Statement for the year ended 31 December 20X9

$ Sales [225,000 + 120,000 – 12,000] 333,000

Cost of sales [67,500 + 60,000

– 12,000 + 2,700] 118,200

214,800

Expenses [70,500 + 37,500] 108,000

Group profit before taxation 106,800

Share of Associated Company

profits [25% × 22,500] 5,625

Profit before taxation 112,425

Taxation – Group [22,500 + 5,250] 27,750

– Associate [25% × 6,000] 1,500 29,250

83,175

Minority interest [20% × 17,250] 3,450

79,725

Dividends paid 30,000

Retained profit for the year 49,725

Retained earnings brought forward

[70,500 + (80% × 35,250*) + (25% × 4,500*) 99,825

149,550

* Post-acquisition profits brought forward

(b) Consolidated Balance Sheet as at 31 December 20X9

Cost Dep’n Net $ $ $ Non-current assets Intangible: Goodwill in subsidiary 8,400

Tangible: [90,000 + 82,500] 375,000 202,500 172,500

Investment in Associate [21,000 + 13,500] 34,500

215,400

Current assets Inventories [105,000 + 45,000 – 2,700] 147,300

Page 239: elliottv10

Barry Elliott and Jamie Elliott: Financial Accounting and Reporting (tenth edition) – Instructor’s Manual

239 © Pearson Education Limited 2006

Trade receivables [98,250 + 52,500] 150,750

Current account – Nit (Note 1) 2,250

Bank [17,750 + 5,250] 22,500

322,800

Current liabilities Trade payables [99,000 + 18,750] 117,750

Taxation payable 27,750

145,500 177,300

392,700

Financed by $1 common shares 187,500

General reserve [22,500 + 2,400 + (25% × 3,000)] (Note 2) 25,650

Retained earnings 149,550

362,700

Minority interest (20% × 150,000) 30,000

392,700

Notes 1. The inter-company current account balance with the Associated company has not been can-

celled because the Associated company is not a member of the group.

2. The group’s share of the Associated retained earnings and General Reserve is calculated on the post-acquisition retained earnings and General reserve of Nit, i.e.

General reserve per Nit balance sheet is 9,000

Pre-acquisition (see question) 6,000

Post-acquisition 3,000

Retained earnings per Nit balance sheet 37,500

Pre-acquisition (see question) 22,500

Post-acquisition 15,000

Page 240: elliottv10

Barry Elliott and Jamie Elliott: Financial Accounting and Reporting (tenth edition) – Instructor’s Manual

240 © Pearson Education Limited 2006

Chapter 24: Question 4 – Twist plc

(a) Consolidated Income Statement for year to 31 December 20X3

£ Sales 450,000

Cost of sales 150,000

300,000

Expenses 171,000

129,000

Share of profits of Associate [74,000 × 30% × 0.25] 5,550

134,550

Taxation – Group 30,000

– Associate [14,000 × 30% × 0.25] 1,050

31,050

103,500

Minority interest 10,000

Attributable to the shareholders of the parent 93,500

Dividends paid 60,000

33,500

Retained earnings brought forward 136,000

169,500

(b) Consolidated Balance Sheet at 31 December 20X3

£ Property, plant and equipment 450,000

Depreciation 150,000

300,000

Investment in Turn [(244,000 × 30%) + 3,300] (W1) 76,500

Current assets

Inventories 180,000

Trade receivables 207,000

Current a/c Turn 18,000

Bank 18,000

423,000

Page 241: elliottv10

Barry Elliott and Jamie Elliott: Financial Accounting and Reporting (tenth edition) – Instructor’s Manual

241 © Pearson Education Limited 2006

Current liabilities

Trade payables 225,000

Taxation 30,000

255,000

168,000

544,500

Financed by

Common shares of £1 each 200,000

General reserve 37,000

Retained earnings [168,000 + (20,000 × 25% × 30%)] 169,500

406,500

Minority interest 138,000

544,500

Working 1

Investment in Turn 75,000

Shares acquired 21,000

Retained earnings [137,000 × 30%] 41,100

General reserve [32,000 × 30%] 9,600

71,700

Goodwill 3,300

Page 242: elliottv10

Barry Elliott and Jamie Elliott: Financial Accounting and Reporting (tenth edition) – Instructor’s Manual

242 © Pearson Education Limited 2006

C H AP T E R 2 5

Chapter 25: Question 1 – Fry Ltd

The profit or loss on foreign exchange in these cases will be as follows:

American Dollar Name of account: Texas Inc Alamos Inc Chicago Inc bank bank account Payable Receivable Creditor Loan creditor

Foreign currency $40,000 $60,000 $100,000 $90,000 $90,000

at exchange rate on 2.60 2.60 2.40 2.40 2.40

date of initial transaction

£15,385 £23,077 £41,667 £37,500 £37,500

Foreign currency $40,000 $30,000 $80,000

at exchange rate on 2.40 2.40 2.30

date of settlement £16,667 £12,500 £34,783

Foreign currency $30,000 $20,000 $90,000 $90,000

at exchange rate on 2.10 2.10 2.10 2.10

date of balance sheet 000 000 £14,286 £9,524 £42,847 £42,857

Profit/(loss)

on foreign exchange (£1,282) £3,709 (£2,640) (£5,357) £5,357

Page 243: elliottv10

Barry Elliott and Jamie Elliott: Financial Accounting and Reporting (tenth edition) – Instructor’s Manual

243 © Pearson Education Limited 2006

Chapter 25: Question 2 – Walpole Ltd

(a) Translate the financial statements of Paris SA into sterling

Income statement

Paris SA Exchange rate Translated €000 £000 Sales 200,000

Purchases (90,000)

Other expenses (7,000)

Interest (3,000)

Taxation (15,000)

85,000 3.0 28,333 Note 1

Opening inventories (22,000) 2.5 (8,800) Note 2

Closing inventories 12,000 4.5 2,667 Depreciation (30,000) 3.5 (8,571) Note 3

Dividend paid (10,000) 5.0 (2,000) Note 4

Net profit carried forward 35,000 11,629

Notes

1 Sales, purchases and expenses have been translated at an average which is an approxima-tion of the rate when they were originally recorded. This requirement of the IAS has been rather more loosely interpreted in the case of interest and taxation, which have been trans-lated at the rates or approximate rates when they originally accrued. Under the wording of the IAS it might be more strictly correct if the taxation and interest were translated at the date they were first recorded in the books.

2 Translated at the actual date of acquiring the inventories.

3 The rate is that applicable to the date of revaluation rather than that at the date of acquiring the fixed assets.

4 The closing rate has been taken as the actual rate in this case.

Balance sheet

Non-current assets 150,000 3.5 42,857 Note 3 Current assets

Inventories 12,000 4.5 2,667 Note 5

Receivables 40,000 5.0 8,000 Note 6

Cash 11,000 5.0 2,200

Page 244: elliottv10

Barry Elliott and Jamie Elliott: Financial Accounting and Reporting (tenth edition) – Instructor’s Manual

244 © Pearson Education Limited 2006

Current liabilities Payables (18,000) 5.0 (3,600)

Walpole Ltd (12,000) 5.0 (2,400)

Taxation (15,000) 5.0 (3,000)

Non-current assets – bonds (10,000) 5.0 (2,000)

Total assets less liabilities 158,000 44,724

Equity Share capital 60,000 2.0 30,000 Note 7

Additional paid-in capital 20,000 2.0 10,000 Note 8

Accumulated

Profit [66,000 – 35,000] 31,000 Balancing figure 30,229 Note 9

111,000 70,229 Note 10

Revaluation reserve 12,000 3.5 3,428 Note 11

Profit for the year 35,000 see above 11,629 Note 12

Loss on exchange (40,562) Note 13

Total equity 158,000 44,724

Notes

5 As inventories are non-monetary assets they are translated at actual.

6 The receivables and all the following assets in the balance sheet are monetary items and therefore retranslated at the closing rate.

7 Share capital issued should be translated at the date of acquiring the subsidiary or at the date of issue if later. At this stage of the process we are attempting to find the figure of profit or loss on exchange differences up to the beginning of the current year so that we can isolate the profit or loss on exchange in the current year. One way to do this is to split the share capital and reserves into the amount arising this year and the balance at the end of the previ-ous year. In this question the balance at the end of the previous year can be calculated as €111,000 (see Note 10). The rest (profit for the year 35,000 and revaluation reserve 12,000) arose in the present year. If we can then translate the 111,000 into pounds sterling at the end of the previous year, we can eventually find this year’s profit or loss on exchange. In practice we could obtain the sterling equivalent of the €111,000 from the workings for the previous year’s consolidated accounts.

8 The additional paid-in capital (share premium) is translated at the same rate as the shares to which it relates.

9 The accumulated profit at the end of the previous year can be taken as the balancing figure after translating the €111,000. The accumulated profit will have been translated at many dif-ferent rates over the years. In practice the translated figure would be available from the previous years’ consolidated accounts.

10 The 111,000 represents a mixture of monetary and non-monetary assets. The amount can be translated by applying the exchange rates used in the balance sheet in the previous year. This is calculated as follows:

Page 245: elliottv10

Barry Elliott and Jamie Elliott: Financial Accounting and Reporting (tenth edition) – Instructor’s Manual

245 © Pearson Education Limited 2006

Non-current assets 150,000

Depreciation 30,000

180,000

Revaluation (12,000)

168,000 2.0 84,000

Opening inventory 22,000 2.5 8,800

190,000 92,800

Balance = Monetary

Liabilities at 31/12/20X1 (79,000) 3.5 (22,571)

Net assets at 31/12/20X1 111,000 70,229

11 The revaluation reserve is translated at the rate when the revaluation took place in the cur-rent year.

12 The income statement was translated to give the profit figure in sterling.

13 The loss on exchange differences is the balancing figure but can be found directly, as shown below.

Calculating the exchange difference for the year

(i) The monetary net liabilities at the beginning of the year of €79,000 (see Note 10 above) have been retranslated into sterling at 31 December 20X2 and a gain of £6,771 has been made since translation at the end of the previous year. £ Opening net monetary liabilities: €79,000 @ opening rate 3.5 = 22,571

Opening net monetary liabilities: €79,000 @ closing rate 5.0 = 15,800

Gain 6,771

(ii) Any profit made during the year initially goes into monetary net assets. The translation of these at the year-end rate will give a profit in this case of the difference between the actual or average rate (as an approximation of the actual rate) used for translating the income statement items and the year-end rate used for translating monetary items in the balance sheet. £

Translated – at average rate 85,000 @ 3.0 = 28,333

– at closing rate 85,000 @ 5.0 = 17,000

Loss 11,333

(iii) The fixed assets were acquired when the rate of exchange was 2 euros to the £ and this was the rate applied to them at 31 December 20X1. On 1 January 20X2 the fixed assets were revalued when the rate was 3.5 euros to the £ and this was therefore the rate applied to these fixed assets at 31 December 20X2. This produced a loss of £36,000 being (£168,000/2.0) – (£168,000/3.5).

Total loss on changing exchange rates (11,333 + 36,000 – 6,771) = £(40,562)

Page 246: elliottv10

Barry Elliott and Jamie Elliott: Financial Accounting and Reporting (tenth edition) – Instructor’s Manual

246 © Pearson Education Limited 2006

(b) Prepare consolidated accounts – Walpole

Balance Sheet as at 31 December 20X2

ASSETS £000

Non-current assets

Goodwill (W1) 550

Other [94,950 + 42,857] 137,807

138,357

Current assets (W2) Inventories [60,000 + 2,667] 62,667

Receivables [59,600 + 8,000] 67,600

Bank [11,000 + 2,200] 13,200

143,467

Total assets 281,824

Equity and liabilities

Common share capital 80,000

Additional paid-in capital 6,000

Revaluation reserve [10,000 + 3,085] (W3) 13,085

Retained earnings [67,000 – 3,334] (W4) 63,666

162,751

Minority interest (W5) 4,473

Non-current liabilities (W2)

Bonds [40,000 + 2,000] 42,000

Current liabilities Payables [45,000 + 3,600] 48,600

Taxation [21,000 + 3,000] 24,000

114,600

Total equity and liabilities 281,824

W1: Goodwill £000 £000

Investment in Paris 41,050 Common share in Paris [90% × 30,000] 27,000 Retained earnings [90% × 5,000*] 4,500 Additional paid-in capital [90% × 10,000*] 9,000 40,500 Goodwill 550

* Pre-acquisition retained earnings translated at the exchange rate on the day of acquisition (£1

= €2)

Page 247: elliottv10

Barry Elliott and Jamie Elliott: Financial Accounting and Reporting (tenth edition) – Instructor’s Manual

247 © Pearson Education Limited 2006

W2: Cancel inter-company balances

Current accounts of 2,400

W3: Paris SA revaluation reserve £000 Balance at 31/12/20X2 as per balance sheet 3,428

Minority interest 10% × 3,428 343

Consolidated balance sheet 3,085

W4: Retained earnings of Paris £000

Balance at 31/12/20X2 as per the balance sheet

[11,629 + 30,229 – 40,562 loss on exchange] 1,296

Pre-acquisition profit 4,500

Minority interest [10% × 1,296] 130 4,630 (3,334)

W5: The minority interest £000

Common shares [10% × 30,000] 3,000

Retained earnings [10% × 1,296] 130

Additional paid-in capital [10% × (9,800 +10,000)] 1,000

Revaluation reserve [10% × 3,429] 343

4,473

The consolidated income statement for the year ended 31/12/20X2 includes the subsidiary fig-ures using the exchange rates as shown in the question.

£ Sales [317,200 + (200,000/3)] 383,867

Cost of sales [170,000 + (8,800 + 90,000/3 – 2,667)] (206,133)

Gross profit 177,734

Depreciation [30,000 + 8,571] (38,571)

Expenses [15,000 + 7,000/3] (17,334)

Loss on foreign exchange (40,562)

Interest [6,000 + 3000/3] (7,000)

Profit before tax 74,267

Taxation [21,000 + 15,000/3] 26,000

Profit after tax 48,267

Minority interest [10% × (11,629 + Div 2,000 – 40,562)] 2,693

Net profit for the year 50,960

Page 248: elliottv10

Barry Elliott and Jamie Elliott: Financial Accounting and Reporting (tenth edition) – Instructor’s Manual

248 © Pearson Education Limited 2006

Dividend paid (20,000)

Retained profit 30,960

Retained earnings brought forward (W6) 32,706

63,666

Group statement of changes in equity for the year ended 31/12/20X2

This statement will appear as follows: £ Retained earnings brought forward (W6) 32,706

Net profit for the year 50,960

Dividend paid (20,000)

Accumulated profit carried forward 63,666

W6: Group retained earnings brought forward at 1/1/20X2

£ Walpole [67,000 – this year 57,000] 10,000

Paris [90% × (30,229 – Pre-acquisition 5,000 (W1))] (22,706)

32,706

Page 249: elliottv10

Barry Elliott and Jamie Elliott: Financial Accounting and Reporting (tenth edition) – Instructor’s Manual

249 © Pearson Education Limited 2006

Chapter 25: Question 3 – Paris SA

Walpole Question 3

(a) Income statement

Paris SA Exchange rate Translated

As in solution 2 35,000 See sol to Qn 2 11,629

Balance sheet

Net assets 158,000 5.0 31,600

Share capital 60,000 2.0 30,000

Additional paid in capital 20,000 2.0 10,000

Retained earnings (66,000–

35,000)

31,000

Balancing figure

(8,286)

111,000 3.5 31,714

Revaluation reserve 12,000 3.5 3,428

Profit for the year 35,000 See above 11,629

Loss on exchange (15,171)

Total equity 158,000 31,600

Proof of the loss on translation

£ (i) Opening net assets €111,000 @ opening rate 3.5 = 31,714

Closing net assets €111,000 @ closing rate 5.0 = 22,200

Loss (9,514)

(ii) Income statement €85,000 @ average rate 3.0 = 28,333

@ closing rate 5.0 = 17,000

Loss (11,333)

(iii) Opening inventories €22,000 @ 2.5 = 8,800

@ 5.0 = 4,400

Profit 4,400

(iv) Closing inventories €12,000 @ 4.5 = 2,667

@ 5.0 = 2,400

Loss (267)

Page 250: elliottv10

Barry Elliott and Jamie Elliott: Financial Accounting and Reporting (tenth edition) – Instructor’s Manual

250 © Pearson Education Limited 2006

(v) Depreciation €30,000 @ 3.5 = 8,571

@ 5.0 = 6,000

Profit 2,571

(vi) Revaluation Reserve €12,000 @ 3.5 = 3,428

@ 5.0 = 2,400

Loss (1,028)

Total loss [ (9,514) + (11,333) +(267) + (1,028) – 2,571 – 4,400] = (15,171)

Add loss on retranslation of goodwill 550 (@2) – 220 (@5) = (330)

(b) Prepare consolidated accounts – Walpole

Balance sheet as at 31 December 20x2

Non-current assets £ £ Goodwill (W1) 220

Other (94,950 + 150,000/5) 124,950

125,170

Current assets (W2)

Inventories (60,000 + 12,000/5) 62,400

Receivables (59,600 + 8,000) 67,600

Bank (11,000 + 2,200) 13,200

143,200

Total assets 268,370

Equity and liabilities

Common share capital 80,000

Additional paid in capital 6,000

Revaluation reserve (10,000 + 3,085) (W3) 13,085

Retained earnings (67,000 – 15,475) (W4) 51,525

150,610

Minority interest (W5) 3,160

Non-current liabilities (W2)

Bonds (40,000+2,000) 42,000

Current liabilities

Payables (45,000 + 3,600) 48,600

Taxation (21,000 + 3,000) 24,000

114,600

Total equity and liabilities 268,370

Page 251: elliottv10

Barry Elliott and Jamie Elliott: Financial Accounting and Reporting (tenth edition) – Instructor’s Manual

251 © Pearson Education Limited 2006

W1 Goodwill

£ £ Investment in Paris 41,050

Common shares in Paris (90% of 30,000) 27,000

Retained earnings (90% of 5,000) 4,500

Additional paid in capital (90% of 10,000* 9,000 40,500

Goodwill 550

• Pre-acquisition retained earnings translated at the exchange rate on the day of acquisition (£1 = €2) Restated at the rate on 31/12/20X2 as goodwill is treated as the asset of the sub-sidiary. Restated at 5.0 = £330.

W2 Cancel inter-company balances Current accounts of 2,400

W3 Paris SA revaluation reserve

£000 Balance at 31.12.20x2 per balance sheet 3,428

Minority interest 10% of 3,428 343

Consolidated balance sheet 3,085 W4 Retained earnings of Paris

£000 £000 Balance at 31.12.20x2 per the balance sheet

(11,629 – 8,286 – 15,171) (11,828)

Pre-acquisition profit 4,500

Minority interest 10% of (11,829) (1,183) (3317)

(15,145)

Loss on restatement of goodwill (330)

(15,475) W5 The minority interest

£000 Common shares (10% of 30,000) 3,000

Retained earnings (W4) (1,183)

Additional paid in capital (10% of 10,000) 1,000

Revaluation reserve (10% of 3,428) 343

3,160 The consolidated income statement for the year ended 31.12.20x2 includes the subsidiary fig-ures using the exchange rates as shown in the question

Page 252: elliottv10

Barry Elliott and Jamie Elliott: Financial Accounting and Reporting (tenth edition) – Instructor’s Manual

252 © Pearson Education Limited 2006

£ £ Sales (317,200 + (200,000/3) 383,867

Cost of sales [170,000 + (8,800 + 0,000/3 – 2,667)] (206,133)

Gross profit 177,734

Depreciation (30,000 + 8,571) (38,571)

Expenses (15,000 + 7,000/3) (17,333)

Interest (6,000 + 3,000/3) (7,000)

Profit before tax 114,830

Taxation (21,000 + 15,000/3) 26,000

Profit after tax 88,830

Minority interest [10% of (11,629 + Div 2,000)] (1,363)

Net profit for the year 87,467

Dividend paid (20,000)

Retained profit 67,467

Group statement of changes in equity for the year ended 31.12.20x2 £ Retained earnings brought forward (W6) (1,957)

Net profit for the year 87,467

Dividend paid (20,000)

65,510

Loss on translation (90% of 15,171) (13,655)

Loss on restatement of goodwill (330)

Retained earnings carried forward 51,525 W6 Group retained earnings brought forward at 1.1.20x2

£ £ Walpole (67,000 – this year 57,000) 10,000

Paris [90% of ( (8,286) – pre-acquisition 5,000 (W1)] (11,957)

(1,957)

Note that in this example the loss on translation has been taken to retained earnings. In practice such gains or losses might be taken to a separate foreign currency reserve.

Page 253: elliottv10

Barry Elliott and Jamie Elliott: Financial Accounting and Reporting (tenth edition) – Instructor’s Manual

253 © Pearson Education Limited 2006

C H AP T E R 2 6

Chapter 26: Question 1 – Alpha plc

(i)

Step 1 Theoretical ex-rights calculation

The shareholders get an element of bonus at the same time as the company receives additional capital. The bonus element may be quantified by the calculation of a theoretical ex-rights price, which is compared with the last market price prior to the issue; the difference is a bonus. The theoretical ex-rights price is calculated as follows: £ 4 shares at fair value of £1 each prior to rights issue = 4.00

1 share at discounted rights issue price of 80p each = 0.80

∴5 shares at fair value after issue (i.e. ex-rights) = 4.80

The theoretical ex-rights price is £4.80/5shares = 0.96

The bonus element is fair value £1 less 96p = 0.04

Step 2 The time-weighted average number of shares is calculated for the current year

No. of shares Shares to date of rights issue

Shares × Increase by bonus fraction × Time adjustment

2,000,000 × 9/12 = 1,500,000

Bonus : ((2,000,000 × 100/96) – 2,000,000) × 9/12 = 62,500

Shares from date of issue

2,500,000 × × 3/12 = 625,000

Weighted average number of shares 2,187,500

Step 3 Calculate BEPS for current year

BEPS for 20X1 is then calculated as £5,000,000/2,187,500 shares = £2.29

Page 254: elliottv10

Barry Elliott and Jamie Elliott: Financial Accounting and Reporting (tenth edition) – Instructor’s Manual

254 © Pearson Education Limited 2006

(ii)

Step 4 Adjusting the previous year’s BEPS for the bonus element of a rights is-sue

This bonus element will affect the comparison with the previous year’s BEPS which will need to be reduced to ensure comparability. The approach is to reduce the prior year by multiplying it by:

Theoretical ex-rights fair value per share = £0.96 Fair value per share immediately before the exercise of rights £1.00

Assuming that the earnings for 20X0 and 20X1 were £4.5m and £5m respectively, the 20X0 BEPS figures will be reported as follows:

As reported in the 20X0 accounts £4.5m/2m = £2.25 As restated in the 20X1 accounts (£4.5m/2m) × (0.96/1.00) = £2.16

The same effect is achieved by increasing the number of shares in the denominator by 100/96 for 20X0:

Earnings/(Number of shares × Bonus fraction ) £4,500,000 / (2,000,000 × (100/96) = £2.16

Page 255: elliottv10

Barry Elliott and Jamie Elliott: Financial Accounting and Reporting (tenth edition) – Instructor’s Manual

255 © Pearson Education Limited 2006

Chapter 26: Question 2 – Beta Ltd

Beta Ltd weighted average number of shares

Time Bonus Bonus element apportion adjustment in rights issue 1 January – 31 March

1,000,000 × 3/12 × 3/2 × 7/6 = 437,500

1 April – 30 April

1,500,000 × 1/12 × 3/2 × 7/6 = 218,750

1 May – 31 August

2,250,000 × 4/12 × × 7/6 = 875,000

1 September – 31 October

3,250,000 × 2/12 × × 7/6 = 631,944

1 November – 31 December

4,333,333 × 2/12 × = 722,222

Weighted average number of shares 2,885,416

Note: bonus element in rights issue calculated as follows:

3 shares at $5.60 = 16.80

1 share at $2.40 = 2.40

Fair value of 4 shares 19.20

Theoretical ex-rights price $4.80

Fair value $5.60

Bonus factor = 5.6/4.8 = 7/6

Page 256: elliottv10

Barry Elliott and Jamie Elliott: Financial Accounting and Reporting (tenth edition) – Instructor’s Manual

256 © Pearson Education Limited 2006

Chapter 26: Question 3 – Nottingham Industries plc

(a) EPS complying with IAS 33 definition of earnings:

Earnings for EPS calculation is ‘profit of the period after tax, minority interests and extraordi-nary items and after preference dividends’.

Basic EPS calculation: £000

Equity earnings:

Profit after tax and extraordinary items 580

Preference dividend [10% of £1,000,000] (100)

480

Weighted average number of ordinary shares (25p)

Actual no. Weight Bonus Weighted time factor average

1.4.X5 in issue 16,000,000 3/12 6/5 4,800,000

1.7.X5 bonus issue 3,200,000

19,200,000 3/12 4,800,000

1.10.X5 Purchase (500,000)

31.3.X6 in issue 18,700,000 6/12 9,350,000

18,950,000

Basic EPS for 20X6 £480,000/18,950,000 = £0.0253

Comparative for 20X5 = £0.022 × 5/6 = £0.0183

(b) Diluted EPS calculation

Equity earnings: £000 As for Basic EPS 480

The computation of Basic and Diluted EPS is as follows:

Per share Earnings Shares Net profit for 20X6 £480,000

Weighted average shares during 20X6 18,950,000

Basic EPS (£480,000/18,950,000) £0.02533

Number of shares under option 200,000

Number that would have been issued

At fair value (200,000 × £1.00)/ £1.10 (181,818)

Diluted EPS £0.0253 £480,000 18,968,182

Page 257: elliottv10

Barry Elliott and Jamie Elliott: Financial Accounting and Reporting (tenth edition) – Instructor’s Manual

257 © Pearson Education Limited 2006

(c) Usefulness of EPS figures

It is helpful to users to have a standardised EPS figure. This is provided by applying the IIMR calculation as follows.

IIMR headline EPS

Headline earnings per share are based upon the headline earnings figure stated in accordance with the Institute of Investment Management and Research Statement of Practice No. 1 The Definition of Headline Earnings and accordingly exclude profit on sale of the major operation. £000 Equity earnings:

Profit after tax and extraordinary items 580

Exclude capital items such as profit on sale of a major operation:

£120,000 less tax £38,000 (82)

IIMR Headline EPS 498

Less: preference dividend (100)

398

Even when standardised the ASB considers that there is too much emphasis on a single profit figure and encourages users to refer to the information set as a whole when appraising perform-ance and predicting future earnings. Nevertheless, the EPS figure has remained an important figure in the eyes of many investors and analysts.

Page 258: elliottv10

Barry Elliott and Jamie Elliott: Financial Accounting and Reporting (tenth edition) – Instructor’s Manual

258 © Pearson Education Limited 2006

Chapter 26: Question 4 – Simrin plc

(a) Calculation of Basic EPS

As per IAS 33:

Profit after tax, Minority interest, EPS = Preference dividends and extraordinary items

Number of ordinary shares

= £79,000 – £9,000 100,000

Basic Earnings per Share = 70p per share

(b) Calculation of the Diluted EPS

£ Subscription monies received = £1.28 × 50,000

= 64,000

Notional number at fair value:

£64,000/£1.1 (fair value of a share) = 58,182

Notional number at no value = 5,818

64,000

Profit after tax and preference dividend

MI & extraordinary items 70,000

Number of shares: At 1 January 20X0 100,000

From warrants at no value 5,818

Total number of shares 105,818

Diluted EPS = £70,000/105,818 = 66.15p per share

(c) (i) Need to disclose DEPS

• Company able to finance projects using convertible securities which carried fixed interest rate and also future benefits causing dilution of shares in the future on conversion.

• Trend revealed by diluted EPS is more meaningful to shareholders as it enables them to identify the final effect on company’s EPS by using convertible debt.

Page 259: elliottv10

Barry Elliott and Jamie Elliott: Financial Accounting and Reporting (tenth edition) – Instructor’s Manual

259 © Pearson Education Limited 2006

(ii) Relevance to shareholders

• Relevance is questionable.

• It shows dilution of future earnings per share and it is reasonable that existing shareholders should be given a warning of the potential dilution.

(d) Reliance on EPS as single most important indicator of financial per-formance

• There is no one correct answer for this, but discussion of the Institute of Investment Man-agement and Research headline figure is required.

Page 260: elliottv10

Barry Elliott and Jamie Elliott: Financial Accounting and Reporting (tenth edition) – Instructor’s Manual

260 © Pearson Education Limited 2006

Chapter 26: Question 5 – Gamma Ltd

There are two steps in arriving at the Diluted EPS, namely:

Step 1 Determine the increase in earnings attributable to ordinary shareholders on conversion of potential ordinary shares

Step 2 Determine the potential ordinary shares to include in the Diluted Earnings per Share

(a) Convertible preference shares receive a dividend of £2.50

Step 1 Determine the increase in earnings attributable to ordinary shareholders on conversion of potential ordinary shares

Increase in Increase in number Earnings per earnings of ordinary shares incremental share Convertible preference shares Increase in net profit

50,000 shares × £2.50 125,000

Incremental shares

50,000/1 50,000 2.50

10% Convertible bond Increase in net profit

£250,000 × 0.10 × (1 – 0.4) 15,000

Incremental shares

250,000/1000 × 500 125,000 0.12

Step 2 Determine the potential ordinary shares to include in the computation of diluted earnings per share

Net profit attributable to Ordinary continuing operations shares Per share As reported 5,000,000 1,000,000 5.00

10% Convertible loan 15,000 125,000

5,015,000 1,125,000 4.46 dilutive

Convertible preference shares 125,000 50,000

5,140,000 1,175,000 4.37 dilutive

Page 261: elliottv10

Barry Elliott and Jamie Elliott: Financial Accounting and Reporting (tenth edition) – Instructor’s Manual

261 © Pearson Education Limited 2006

(b) Convertible preference shares receive a dividend of £6 per share.

Step 1 Determine the increase in earnings attributable to ordinary shareholders on conversion of potential ordinary shares

Earnings per Increase in Increase in number incremental earnings of ordinary shares share Convertible preference shares Increase in net profit

50,000 shares × £6.00 300,000

Incremental shares 50,000/1 50,000 6.00

10% Convertible bond Increase in net profit

£250,000 × 0.10 × (1 – 0.4) 15,000

Incremental shares

250,000/1000 × 500 125,000 0.12

Step 2 Determine the potential ordinary shares to include in the computation of diluted earnings per share

Net profit attributable to Ordinary Per share continuing operations shares As reported 5,000,000 1,000,000 5.00

10% convertible loan 15,000 125,000

5,015,000 1,125,000 4.46 dilutive

Convertible preference shares 300,000 50,000

5,315,000 1,175,000 4.52 anti-dilutive

• Since the Diluted EPS is increased when taking the convertible preference shares into ac-count (from 4.46p to 4.52p) , the convertible preference shares are anti-dilutive and are ignored in the calculation of Diluted EPS.

• The lowest figure is selected and the Diluted EPS will, therefore, be disclosed as 4.46p.

Page 262: elliottv10

Barry Elliott and Jamie Elliott: Financial Accounting and Reporting (tenth edition) – Instructor’s Manual

262 © Pearson Education Limited 2006

Chapter 26: Question 6 – Delta NV

(a) Calculate theoretical ex-rights value of a share

Market value of a share prior to rights issue was €1.10.

4 shares at €1.10 per share = 4.40 1 share at 60p = .60 5 shares = 5.00 Theoretical ex-rights value = 1.00

(b) Bonus issue factor =110/100

(c) BEPS 20X8

440,000/(4,000,000 × 11/10) = €0.10

previously calculated as:

440,000/4,000,000 = €0.11

(d) BEPS 20X9

Uplift shares prior to issue by 110/100 4,000,000 × (110/100) × 6/12 months = 2,200,000

Weight shares after issue: 5,000,000 × 6/12 months = 2,500,000 Total shares for BEPS calculation = 4,700,000

BEPS = €500,000/4,700,000 = €0.106

Page 263: elliottv10

Barry Elliott and Jamie Elliott: Financial Accounting and Reporting (tenth edition) – Instructor’s Manual

263 © Pearson Education Limited 2006

Chapter 26: Question 7 – X Ltd

(a) Ordinary shares Profit EPS Effect Net profit after tax 18,160

Less preference dividend (160)

40,000 18,000 45p

Options (W1) 400 40,400 18,000 44.6p dilutive

Convertible preference shares 3,200 160

43,600 18,160 41.7p dilutive

Convertible loan stock Interest [6% × £20m ×s 0 .67] 804 Discount 200 Shares converted [(20m/200) × 23] 2,300 _____ 45,900 19,164 41.8p anti-dilutive

• Since the loan stock is anti-dilutive it is ignored in the calculation of Diluted Earnings per Share

• Diluted EPS will be reported as 41.7p.

W1 Fair value of one ordinary share £1.50

Number of options 2,000,000

Exercise price £1.20

Proceeds from exercise of options £2,400,000

Number of shares assumed to be issued at fair value 1,600,000

Number of shares issued for no consideration (2m – 1.6m) = 400,000

(b)

• An option is treated as if – there was an issue of shares for full market value/fair value; and – an issue for no consideration (a bonus issue). – The bonus element is treated as being the dilutive effect.

• IAS 33 is saying that by issuing options to directors/employees the company is making a bonus issue of shares plus a full issue of shares, the latter being as-sumed not to have a dilutive effect.

Page 264: elliottv10

Barry Elliott and Jamie Elliott: Financial Accounting and Reporting (tenth edition) – Instructor’s Manual

264 © Pearson Education Limited 2006

• Only potential ordinary shares that would dilute EPS should be taken into account and any anti-dilutive potential ordinary shares will be ignored.

• This procedure essentially means that certain categories of potential

• ordinary shares will not be used in the calculation.

• Thus the calculation will be based on the concept of prudence rather than on the substance of what is realistically going to occur. All items of income or expense that would cease on conversion are to be added back.

• Prudent disclosure.

As regards the ranking of potential ordinary shares from most to least dilutive and the subse-quent calculations, an alternative solution would be to disclose both the fully diluted EPS and the maximum dilution of EPS. This would essentially mean that the more realistic calculation and the prudent calculation of IAS 33 would be disclosed.

Page 265: elliottv10

Barry Elliott and Jamie Elliott: Financial Accounting and Reporting (tenth edition) – Instructor’s Manual

265 © Pearson Education Limited 2006

C H AP T E R 2 7

Chapter 27: Question 1 – Example Ltd

£000 £000 Cash flows from operating activities Net profit before tax 500

Adjustments for:

Depreciation 102

Profit on sale of plant (13)

Interest expense 20

Operating profit before working capital changes 609

Increase in trade and other receivables (260)

Increase in inventories (400)

Decrease in trade payables (40)

Cash generated from operations (91)

Interest paid (20)

Income taxes paid (220)

Net cash used in operating activities (331)

Cash flows from investing activities Purchase of property, plant and equipment (560)

Proceeds from sale of equipment 241

Payment to acquire government securities (20)

Net cash used in investing activities (339)

Cash flows from financing activities Proceeds from issuance of share capital 300

Redemption of debentures (50)

Dividends paid (120)

Net cash from financing activities 130

Net increase in cash and cash equivalents (540) Cash and cash equivalents at the beginning of the period 72

Cash and cash equivalents at the end of the period (468)

Page 266: elliottv10

Barry Elliott and Jamie Elliott: Financial Accounting and Reporting (tenth edition) – Instructor’s Manual

266 © Pearson Education Limited 2006

Chapter 27: Question 2 – Martel plc

(a) £000 £000

Cash flows from operating activities Net profit before tax 427

Adjustments for:

Depreciation 292

Profit on sale of plant (8)

Interest expense 52

Operating profit before working capital changes 763 Increase in trade and other receivables (132)

Increase in inventories (174)

Increase in trade payables 46

Cash generated from operations 503

Interest paid (52)

Taxes paid (79)

Net cash used in operating activities 372

Cash flows from investing activities Purchase of property, plant and equipment (714)

Proceeds from sale of equipment 20

Purchase of government securities (40)

Net cash used in investing activities (734)

Cash flows from financing activities Proceeds from issuance of share capital 150

Proceeds from 9% debenture issue 82

Dividends paid (76)

Net cash from financing activities 156

Net increase in cash and cash equivalents (206) Cash and cash equivalents at the beginning of the period 22

Cash and cash equivalents at the end of the period (184)

Note 1: Cash and cash equivalents 20X1 20X0 Bank – 22

Overdraft (184) –

Cash and cash equivalents (184) 22

Page 267: elliottv10

Barry Elliott and Jamie Elliott: Financial Accounting and Reporting (tenth edition) – Instructor’s Manual

267 © Pearson Education Limited 2006

(b) Martel plc has invested heavily in fixed assets during the year and although it has raised addi-tional capital it has had to rely on a bank overdraft. The acid test ratio is lower in the current year (304 : 642).

However, we do not have information on the projected cash flows that supported the capital in-vestment decisions – this is where narrative information within the annual report could be helpful in identifying the company’s strategic planning for future years, e.g. new markets, new products, greater productive efficiency.

Page 268: elliottv10

Barry Elliott and Jamie Elliott: Financial Accounting and Reporting (tenth edition) – Instructor’s Manual

268 © Pearson Education Limited 2006

Chapter 27: Question 3 – Flow Ltd

Cash flow Statement for the year ended 31 December 20x6 for Flow Ltd

Net cash inflow from operating activities 191,025 Cash flows from investing activities

Payment to acquire non-current assets (265,500)

[1,983,750 + 25,500–1,743,750]

Receipts from sale of non–current assets 6,225

Net cash paid on investing activities (259,275) Cash flows from financing activities

Issue of common shares 180,000

Dividends paid

(could be shown as operating cash flow) (45,000)

Net cash inflow from financing activities 135,000 Net increase in cash and cash equivalents

[75,000–8,250] 66,750 Net cash inflow from operating activities:

Profit (176,625 –(387,000–300,000–45,000)) 134,625

Non cash items

Depreciation [619,125–551,250 +(25,500–9,375)] 84,000

Loss on disposal [9,375–6,225] 3,150 Changes in working capital

Decrease in inventory 15,750

Increase in trade receivables (22,500)

Decrease in trade payables (24,000)

Net cash inflow from operating activities 191,025

Page 269: elliottv10

Barry Elliott and Jamie Elliott: Financial Accounting and Reporting (tenth edition) – Instructor’s Manual

269 © Pearson Education Limited 2006

Chapter 27: Question 4 – Blue Ting plc

(a) Cash Flow Statement for the year ended 31 May 20X5 £m £m Cash flows from operating activities Profit before tax 96

Adjustments for:

Depreciation expense 37

Amortisation of development expenditure 1

Interest income (3)

Interest expense 7

Loss (gain) on disposal of property, plant and equipment (2)

Operating profit (loss) before working capital changes 136 Increase in accounts receivable (20)

Decrease in inventories 15

Increase in payables 89

Cash generated from (used in) operations 220 Interest paid (4)

Interest income 3

Interest element in finance lease rental payment (3)

Tax paid [10 + 22 – 16] (16)

Dividends paid [12 + 8] (20)

Net cash from operating activities 180

Cash flows from investing activities: Proceeds from disposal of property, plant and equipment 21

Purchase of property, plant and equipment (104)

Net cash used in investing activities (83)

Cash flows from financing activities: Proceeds from issuing shares 14

Share issue costs (1)

Purchase of own shares (12)

Capital repayments under finance leases (7)

Net cash used in financing activities (6)

Net increase (decrease) in cash and cash equivalents 91 Cash and cash equivalents at beginning of year 3 Cash and cash equivalents at end of year 94

(Note: Dividends paid have been treated in their optional position in operating activities and the cash and liquid resources have been assumed to be cash and cash equivalents.)

Page 270: elliottv10

Barry Elliott and Jamie Elliott: Financial Accounting and Reporting (tenth edition) – Instructor’s Manual

270 © Pearson Education Limited 2006

(b) Published forecast cash flow information: advantages and disadvantages to users

Advantages

• Provides the benefit of: management’s knowledge of future cash flow their views as to future cash flows committing management to future planning committing to considering going concern issues.

• Reduces the benefits of insider dealing as information would be in the public domain.

• Makes it more possible to evaluate managerial performance

• Aids investors and creditors to assess the ability of the company to meet its obligations in the future.

Disadvantages

• They are uncertain.

• They are subjective as based on the opinions of management.

• They can be manipulated by management although a poor history of accuracy will become apparent over time.

Page 271: elliottv10

Barry Elliott and Jamie Elliott: Financial Accounting and Reporting (tenth edition) – Instructor’s Manual

271 © Pearson Education Limited 2006

Chapter 27: Question 5 – Carver plc

Cash Flow from Operations for the year ended 30 September 20X4

£000 £000 Profit before tax

1,985

Adjustments for:

Depreciation W1 325

Investment income (155)

Interest payable 150

Share of profit of associate (495)

Profit on sale of machinery (100)

Operating profit before working capital changes 1,710 Increase in inventory W2 (943)

Increase in trade receivables W3 (547)

Increase in trade payables W4 152

Cash generated from operations 372 Interest paid (100)

Taxes paid W5 (250)

Net cash from operating activities 22

Cash from investing activities Purchase of machinery W6 (1,085)

Sale of machinery 500

Purchase of subsidiary W7 98

Dividends received from associate W8 250

Dividends received from fixed asset investments 155

Cash used in investing activities (82)

Cash flow from financing Proceeds from the issue of share capital W9 2,453

Issue of loan stock W10 920

Capital payments under finance leases W11 (270)

Dividends paid to group shareholders (300)

Dividends paid to minority interests W12 (48)

Page 272: elliottv10

Barry Elliott and Jamie Elliott: Financial Accounting and Reporting (tenth edition) – Instructor’s Manual

272 © Pearson Education Limited 2006

Cash flow from financing 2,755 Increase in cash and cash equivalents 2,695

Cash and cash equivalents brought forward 1,820

Cash and cash equivalents carried forward 4,515

Workings

W1: Depreciation charge Buildings 125

Machinery

Closing aggregate amount 1,200

Less: Opening aggregate amount(1,100)

100

Add: depreciation on disposal 100 200

325

W2: Inventory Closing balance 1,975

Less: Opening balance 1,000

Arising on acquisition 32 (1,032)

943

W3: Trade receivables Closing balance 1,850

Less: Opening balance 1,275

Arising on acquisition 28 (1,303)

547

W4: Trade payables Closing balance 500

Less: Opening balance 280

Arising on acquisition 68 (348)

152

W5: Tax

Opening balances

Income tax 217

Deferred tax 13

Transfer from profit and loss account 495

Closing balances

Page 273: elliottv10

Barry Elliott and Jamie Elliott: Financial Accounting and Reporting (tenth edition) – Instructor’s Manual

273 © Pearson Education Limited 2006

Income tax (462)

Deferred tax (30)

233

Acquisition tax 17

250

W6: Investment in machinery Cost at 30.9.20X4 3,000

Less: Cost at 1.10.20X3 (1,400)

1,600

Add: Disposal 500

2,100

Less: Arising from acquisition (165)

Leased (850)

Cash outflow 1,085

W7: Cash Cash acquired from acquisition 112

Less: Cash consideration (14)

Cash inflow 98

W8: Dividends received from associate Opening balance 1,000

Add: Share of profit 495

Less: Tax (145) 350

1,350

Closing balance (1,100)

Cash inflow 250

W9: Shares

Closing balances

Shares 3,940

Premium 2,883

6,823

Less: Opening balances

Shares (2,000)

Premium (2,095)

Page 274: elliottv10

Barry Elliott and Jamie Elliott: Financial Accounting and Reporting (tenth edition) – Instructor’s Manual

274 © Pearson Education Limited 2006

Non-cash consideration

Shares (220)

Premium (55)

Cash inflow 2,453

W10: Loans Closing balance 1,460

Less: Opening balance (500)

960

Less: Increase finance cost (40)

Cash inflow 920

W11: Lease – capital payments Opening balances [200 + 170] 370

Add: new lease commitment 850

1,220

Less: Closing balances [240 + 710] (950)

Cash outflow 270

W12: Minority interests Opening balance –

Add: Profit for year 100

Arising from acquisition 63

163

Closing balance (115)

Cash inflow 48

Page 275: elliottv10

Barry Elliott and Jamie Elliott: Financial Accounting and Reporting (tenth edition) – Instructor’s Manual

275 © Pearson Education Limited 2006

C H AP T E R 2 8

Chapter 28: Question 1 – Saddam Ltd

(a) Profitability – ROCE

• Camel Ltd is the most profitable of the three companies.

• An inspection of the secondary ratios shows that this is due to efficient utilisation of assets since its net profit ratio is well below that of the other two companies.

• Examination of gross profit percentages confirms the observation that Camel Ltd seems a high volume, low margin business compared with the others.

Liquidity

• Ali Ltd has a current ratio which is out of line with the other two, being very much higher suggesting surplus investment in working capital.

• The acid test ratio reinforces this view and also indicates that Baba Ltd appears to have a liquidity problem with current liabilities considerably greater than cash and debtors (de-spite having the greatest number of weeks’ debtors outstanding of the three companies).

• Baba Ltd also has considerably more weeks of stock outstanding than the other two com-panies which may be linked with the high level of creditors.

• Ali Ltd also has stock levels well in excess of Camel Ltd explaining in part at least the high current ratio.

Dividends

Camel Ltd is paying a higher proportion of profits out in dividends, which may have the effect of raising shareholder loyalty and the bid price.

Conclusion

• Baba Ltd appears to have considerable liquidity problems arising out of excess investment in stock.

• Camel Ltd is a lean enterprise able to survive on a lower gross profit margin due to superior asset utilisation. Why is the gross profit margin low?

Before a final decision is made the absolute figures in the financial statements should be studied and questions raised such as:

• Are the activities of the firms really the same?

• What are the relative turnovers?

• What is the growth over a period of years?

• What are the trends of all the ratios?

• How old are the assets?

• Are asset ages distorting ROCE comparisons between the companies?

Page 276: elliottv10

Barry Elliott and Jamie Elliott: Financial Accounting and Reporting (tenth edition) – Instructor’s Manual

276 © Pearson Education Limited 2006

Also need to assess managerial skills, product potential etc. which are not shown in the financial statements.

(b) Why balance sheet is unlikely to show the true market value of the business

The accounting policy in the UK is to state fixed assets at cost less depreciation or at historical cost modified by revaluation of all or selected classes of fixed assets.

The true market value of a listed company is available from the market capitalisation figure based on current share prices.

The true market value of an unquoted company is not readily available and would require the future cash flows to be evaluated.

Page 277: elliottv10

Barry Elliott and Jamie Elliott: Financial Accounting and Reporting (tenth edition) – Instructor’s Manual

277 © Pearson Education Limited 2006

Chapter 28: Question 2 – Esrever Ltd

Forecast profit and loss account for year ended 30 June 20X1

£ £ Turnover [87,007 × 100/32] (S3) 271,897

Opening stock 22,040

Purchases (S5) 194,205

216,245

Closing stock [184,890 × 61.9/365] (S5) 31,355

Cost of sales [271,897 × 68%] (S4) 184,890

Gross profit [20,290 × 100/23.32] (S2) 87,007

Depreciation

– buildings [132,000 × 2%] (S6) 2,640

– fixtures etc. [96,750 × 20%] (S6) 19,350

Loan interest [50,000 × 12%] (S7) 6,000

Credit expenses (balancing figure) (S8) 33,655

61,645

Profit before tax 25,362

Corporation tax [20,290 × 20/80] (S9) 5,072

Profit after tax [181,808 × 11.16%] (S1) 20,290

Dividends [200,000 × 2.5p] (S10) 5,000

Profit retained (S11) 15,290

Profit retained b/f (S12) 66,518

Retained profit c/f (S13) 81,808

Forecast balance sheet as at 30 June 20X1

£ £ Fixed assets (NBV)

Land and buildings [132,000 – 2,640] 129,360

Fixtures, fittings [96,750 – 19,350] 77,400

(S14) 206,760

Current assets Stock (S15) 31,355

Debtors [(271,897 × 42.6/365) × 1.15] (S16) 36,494

67,849

Page 278: elliottv10

Barry Elliott and Jamie Elliott: Financial Accounting and Reporting (tenth edition) – Instructor’s Manual

278 © Pearson Education Limited 2006

Creditors: amounts falling due in less than one year Bank overdraft (a balance figure based on note 2) (S20) 9,756

Creditors [(194,205 + 33,655) × (29.7/365) × 115%] (S17) 21,321

Other creditors [5,072 tax + 5,000 dividends

+ 1,652 VAT] (S18, S19) 11,724

42,801

Net current assets 25,048

Total assets less current liabilities (per Note 3) 231,808

Creditors: amounts falling due in more than one year 12% loan (S23) 50,000

181,808

Ordinary shares (S21) 100,000

Profit and loss account (balancing figure) (S22) 81,808

181,808

VAT: Output tax [271,897 × 15%] 40,785

Input tax [(194,205 + 33,655) × 15%] 34,179

Net amount for year 6,606

6,606 × 0.25 1,652

Approach to Esrever profit and loss account

(S1) Start with post-tax profit i.e. 11.16% of (231,808 – 50,000) per notes 3 & 4 = £20,290 (S2) From post-tax profit 20,290 derive gross profit as 100/23.32 × 20,292 based on Note 4 = £87,007 (S3) Next, derive turnover as 100/32 × 87,007 based on Note 6.

Cost of goods sold = 68% of turnover. Therefore turnover = 100/32 × gross profit = £271,897

(S4) From sales and gross profit derive cost of goods sold as 271,897 – 87,007 = £184,890

(S5) You can now find components of cost of sales (£184,890) as: £

(a) Opening stock 22,040 (given in question) (b) Purchases 194,205 (balance figure)

216,245 (c) Closing stock (31,355) (61.9 × 184,890)

365 Total costs of goods sold 184,890

Note: Start with closing stock 61.9 days based on Note 7; all other figures are derived and the opening stock is given as £22,040.

Page 279: elliottv10

Barry Elliott and Jamie Elliott: Financial Accounting and Reporting (tenth edition) – Instructor’s Manual

279 © Pearson Education Limited 2006

(S6) Depreciation: 2% × 132,000 for buildings = £2,640 20% × 96,750 for fixtures etc. = £19,350

based on Note 1 and opening asset given (S7) Loan interest is 12% of 50,000 = 6,000 (S8) Expenses – this is a balancing figure as we already have all the other figures in the profit

and loss account = 33,655 (S9) Taxation charge is 20/80 × 20,290 based on Note 5 = 5,072 (S10) Dividend – see Note 9 (200,000 × 2.5p) = 5,000 (S11) Retained profit = 15,290

(S12) Retained profit b/forward is a balancing figure = 66,518

(S13) Retained profit c/down (see S22 below) = 81,808

Approach to Esrever balance sheet

Projected balance sheet as at 30/6/20X1 is built up as follows:

(S14) Fixed assets are derived from the opening figure less depreciation = 206,760

(S15) Stock has already been computed at = 31,355 (S16) Debtors, based on Note 10, assuming 42.6 days’

credit, are 42.6/365 × 271,897 = 31,734 × 1.15 to cover VAT = 36,494

(S17) Creditors, assuming credit of 29.7 days, are 29.7/365 × 227,860 × 1.15 = (21,321)

(S18) Other creditors (dividends 5,000 + tax 5,072) = (10,072) (S19) VAT 15% net of sales – purchases and expenses is

15% (271,897 – 194,205 – 33,655) x 0.25 = (1,652) (11,724)

(S20) Overdraft is balancing figure based on Note 2 = (9,756) Current liabilities 42,801

Total assets less current liabilities per Note 3 231,808 (S21) Share capital given in question 100,000 (S22) Retained profit (balancing figure) 81,808 (S23) 12% loan 50,000

231,808 Note: Retained profit is the balancing figure to make up £231,808. The bank overdraft of £9,756 is the overall balance sheet balancing figure.

Page 280: elliottv10

Barry Elliott and Jamie Elliott: Financial Accounting and Reporting (tenth edition) – Instructor’s Manual

280 © Pearson Education Limited 2006

Chapter 28: Question 3 – Amalgamated Engineering plc

(a) Cash flow statement for year ended 31 December 20X6

£000 £000 Net cash inflow from operating activities 495

Returns on investing and servicing of finance

Interest paid 195

Net cash outflow 195

300

Taxation

Tax paid 375

(75)

Capital expenditure

Payment to acquire plant (450)

Receipt from sale of investments 300

(150)

Equity dividends paid (225)

Net cash outflow before financing (450)

Financing -

Increase in overdraft (450)

Reconciliation of operating profit to net cash inflow from operating activities

Operating profit 795

Depreciation 300

Increase in stocks (375)

Increase in debtors (300)

Increase in creditors 75

495

(b) 20X5 20X6

Liquid ratio

Current assets – inventory 1,125 1,125

Current liabilities 1,125 1,575

= 1 = 0.71

Page 281: elliottv10

Barry Elliott and Jamie Elliott: Financial Accounting and Reporting (tenth edition) – Instructor’s Manual

281 © Pearson Education Limited 2006

Interest cover Profit before interest 885 795

Interest charge 135 195

= 6.56 times = 4.08 times

Return on average shareholders’ funds Profit after tax …….375 …………..300

Av. shareholders funds (4,575 + 4,425) / 2 (4,650 + 4,575) / 2

(Could revalue property) = 8.3% = 6.5%

Gearing ratio Long-term loans 1,500 1,500

Shareholders’ funds 4,575 4,650

= 32.8% = 32.3%

or

Long-term loans 1,500 1,500

Long-term loans and shareholders’ (1,500 + 4,575) (1,500 + 4,650)

funds

= 24.7% = 24.4%

Stock turnover ratio Cost of sales 4,410 4,680

Inventory 1,125 1,500

= 3.92 times = 3.12 times

(c) Main points in report should cover the following. Most important points are with an asterisk.

Profitability *(1) Given unchanged sales volume (NB cannot tell from historical cost accounts without

date on specific price movements), price rises have been below the level of general in-flation (4.8%). Is this deliberate policy or just poor management? If deliberate appears not to have improved sales.

(2) Cost of materials and labour also increased below level of inflation (5% and 5.6% respec-tively).

More efficient use? (3) Overheads increased 10% – in line with inflation (both production and administrative) – led

to falling margins (gross and net). (Further information by product might help see if one particular area is a problem – or if it is right across the board.)

(4) Increased interest has caused profit before tax to fall 20% although interest cover still looks OK. (NB Is this relevant? Interest is paid from cash.)

Page 282: elliottv10

Barry Elliott and Jamie Elliott: Financial Accounting and Reporting (tenth edition) – Instructor’s Manual

282 © Pearson Education Limited 2006

*(5) Trends are worrying – falling margins and rising interest seem to indicate problems soon. How long can firm continue to hold the dividend? (Need more years’ data – long-term picture. Is this a recent trend or not?)

Solvency/liquidity (6) Working capital rising – trade receivables and inventories are up a lot.

*(7) Reflected in worsening liquid ratio – quite a large fall. (Again, need more years’ data. What is norm?)

*(8) Inventory turnover is getting worse – 3.85 months’ inventory on hand (20X5 3.06). Need more information here – slow-moving inventory? Or is it just poor management of working capital?

*(9) Trade receivables’ turnover ratio has got worse (20X5 7.64; 20X6 5.87). In their state they need to be collecting more quickly. Is there one or a few debts causing this, or is it general sloppiness?

(10) Flow of funds – company is investing in new equipment so is presumably not contract-ing operations. Need information as to use equipment is being put to, and future capital expenditure plans.

*Purchases of assets (+ payment of tax + dividend) have been partly paid for by selling off short-term investments. This is a one-off – bad sign.

Could use previous 5 years’ funds flow statements – trends quite important.

*(11) The increased overdraft is financing the increased stocks and debtors. (12) Gearing ratio is OK – but the problem is one of liquidity at the moment.

Could argue the overdraft appears to be a permanent feature of this firm. The gearing ra-tio looks worse if the overdraft is included (+ an overdraft of 1,500,000 makes it look even more unhealthy). (Gearing ratios calculated using book values may not be too use-ful – could recalculate using market values of debt and equity, where quoted.)

General points *(13) Why does firm want to increase the overdraft? Seems to be to finance working capital.

Could be risk for the bank if the firm’s profitability is in a long-term decline (Does not mean don’t lend – could charge more interest.)

*(14) Or could secure the overdraft – market value of the land and buildings is well in excess of the debentures.

*(15) How will firm pay off the overdraft? Need to ask for cash forecasts for next few years (firm should have – if not, poor management). (NB Historical cost accounts generally of little help with respect to forward-looking data.)

(16) More data on management. Old, young? Likely to let firm stagnate? Also need to see strategic plans – in what direction is firm going? Do they know?

Page 283: elliottv10

Barry Elliott and Jamie Elliott: Financial Accounting and Reporting (tenth edition) – Instructor’s Manual

283 © Pearson Education Limited 2006

(d) Response to director

(i) Debt Service Coverage Ratio

• This ratio requires the figures for interest, tax, depreciation and amortisation charge to cal-culate EBITDA.

• The ratio gives the bank an indication of the company’s ability to meet its capital debt re-payments as well as annual interest payments from its cash flow from operations.

(ii) Cash flow from operations to current liabilities

• This ratio requires the cash flow from operations figure in the cash flow statement.

• The ratio gives additional information to the current and acid test ratios which are static in the sense that both the numerators and denominators are based on year-end figures which are capable of manipulation or management e.g. running down stocks or exceptional cash receipts at the year-end.

(iii) Cash recovery ratio

• This ratio requires the figures for cash flow from operations and proceeds of sale of fixed assets from the cash flow statement.

• The ratio gives an indication of the payback time i.e. how quickly the company will recoup its investment in fixed assets from its cash flow. The manager would naturally regard a shorter period as less risky.

Page 284: elliottv10

Barry Elliott and Jamie Elliott: Financial Accounting and Reporting (tenth edition) – Instructor’s Manual

284 © Pearson Education Limited 2006

Chapter 28: Question 4 – Sally Gorden

(a) EPS Ruby Sapphire

Earnings: Profit £280,000

Pref. div. (90,000)

190,000 Number of shares:

1.7.X3 – 30.9.X3: 1,500,000 × 3/12 = 375,000

1.10.X3 – 31.6.X4: 2,000,000 × 9/12 = 1,500,000

1,875,000

£190,000 / 1,875,000 × 100 = 10.13p

£240,000 / 3,000,000 × 100 = 8p

(b) Price/earnings ratio

475/10.13 = 47 times 480/8 60 times

(c) PE ratio of Sapphire plc is almost twice that of Ruby plc.

• This would reveal that there is much higher demand for shares in Sapphire.

• This in turn indicates greater confidence the investing public has in that company.

• This confidence may be based on

• the type of industry

• growth potential, growth rate

• track record of past performance

• diversity of its products

• quality of management

• customer attachment and so on.

(d) Other matters that should be considered

(i) What Sapphire’s EPS would have been if there had

been no bonus issue: 240,000/2,000,000 × 100 12p Sapphire appears better than Ruby

Page 285: elliottv10

Barry Elliott and Jamie Elliott: Financial Accounting and Reporting (tenth edition) – Instructor’s Manual

285 © Pearson Education Limited 2006

(ii) Return on capital employed (ROCE)

PBIT/capital employed × 100 588,000 / 2,710,000 × 100 = 21.7% 445,000 / 2,450,000 × 100 = 18.2%

(iii) Return on equity capital (ROEC)

PBT less pref. Div./equity cap + reserves 354,000/1,310,000 × 100 = 27% 385,000/1,950,000 × 100 = 19.7% • Ruby provides a better return on equity • but its EPS is not quite so favourable • it is more geared, and • had borrowed at a significantly higher cost than Sapphire.

(iv) Gearing ratio

Prior charge capital/total CE × 100 1,400,000/2,710,000 × 100 = 51.66% 500,000/2,450,000 × 100 = 20.4%

• Though both companies are geared, Ruby is highly geared. This means that any fall in profit will affect equity shares more than in proportion.

(e) Advantages of gearing

• Equity shareholders benefit if the return on investment exceeds the cost of borrowing.

• There is no dilution of the existing shareholders’ interest if funds are raised by borrowing rather than by an issue to new shareholders.

• Loan interest is allowable for tax relief.

• Lenders normally obtain some form of security in the form of either a charge on assets or prior rights on liquidation. This means that their risk is lower and therefore their required rate of interest is lower.

Disadvantages of gearing

• Impact on company’s funding if loan covenants are breached e.g. may be required to rene-gotiate the loan at a higher rate of interest or even by issuing additional ordinary shares to the lenders in recognition of their increased risk.

• Impact on company’s funding if equity shareholders perceive that there is a greater risk to equity funds if there is high gearing and as a result require a higher return on their invest-ment.

• Adverse impact on amount available for distribution to shareholders if profits fall.

Page 286: elliottv10

Barry Elliott and Jamie Elliott: Financial Accounting and Reporting (tenth edition) – Instructor’s Manual

286 © Pearson Education Limited 2006

Chapter 28: Question 5 – Segmental Reporting

(i) The case for segmental reporting – two arguments for

• It will reveal in more detail how well management has performed.

• Management will not be able to hide its failures behinds its successes.

• Both will be disclosed and shareholders will be better able to judge the performance of di-rectors.

• In addition, disclosure of segment results may encourage management to exercise greater care when making investment decisions and be more positive in correcting any mistakes.

• The first argument is, then, that segmental reporting will result in improved managerial per-formance.

• The data provided by segmental reporting will be more useful for the investors.

• This is because many financial statement users have said that consolidated financial infor-mation, while important, would be more useful if supplemented with disaggregated information to assist them in assessing those uncertainties that surround the timing and amount of expected cash flows.

• This would allow them, therefore, to assess the risks related to a personal investment in or a loan to an enterprise that may well operate in different industries or in different areas of work.

• The results of a diversified enterprise are composed of the results of its parts and the finan-cial users consequently regard financial information on a segmental basis as also important.

(ii) The case against segmental reporting

• The case against segmental reporting arises from a consideration of cost and reliability.

• An important consideration in assessing the desirability of disclosing segmental data is a comparison of the benefits arising from and the costs incurred by any such disclosure.

• If the benefits exceed the costs, then the disclosure is desirable.

• This comparison is difficult to make in practice because the benefits are enjoyed by the us-ers while the costs are incurred by the statement providers.

• It is not surprising that users express a need for segmental data because it costs them noth-ing.

• Equally, it is not surprising that the statement providers do not want to incur the costs of disclosing segmental data because

• they are unlikely to receive any benefits and,

• even worse, they run the risk of their managerial deficiencies being revealed.

• The costs that may be incurred by the statement providers include

• the costs of collecting and processing the information,

• the costs of audit,

Page 287: elliottv10

Barry Elliott and Jamie Elliott: Financial Accounting and Reporting (tenth edition) – Instructor’s Manual

287 © Pearson Education Limited 2006

• the costs of disseminating it to those who must receive it and

• the costs of disclosure in the form of a loss of competitive advantage vis-à-vis trade competitors or trade unions with a consequent effect on wage demands.

• It follows that a comparison of the private costs incurred by the providers and the private benefits enjoyed by the users is likely to be inconclusive.

• A more fruitful, but again difficult, approach would be to compare the social costs with the social benefits. The social costs would be the resources consumed in the gathering, process-ing and publication of the segmental data. The social benefits would be the improved allocation and more efficient use of resources.

• The second major objection to the provision of segmental data is their reliability.

• It is argued that segmental data are not sufficiently reliable to justify disclosure. If this is true the unreliable data may be just as misleading as no segmental data at all.

• The unreliability is due to the fact that there is the necessity to make arbitrary allocations of both costs and revenues amongst the various segments of the business.

• The degree of arbitrariness will depend upon the nature and size of the reporting segments and the amount of detail disclosed for each segment.

There are other specific objections to the disclosure of segmental data that may be made. These include:

Investors invest in a company and not its individual segments. Whilst this is correct it cannot be denied that data about the operations of individual segments may permit investors to make better informed decisions about investments.

The data are difficult to interpret and may confuse readers or be misunderstood with inappropriate inferences being drawn. It is usually assumed, however, that the statement users are technically competent and able to understand accounting data.

Segmental data cannot be prepared with sufficient reliability and it is beyond the scope of external financial reporting to provide such analytical or interpretive data. It is true then that there are reliability problems with producers of segmental data, but whether those problems are sufficient to warrant non-disclosure of the data is a matter of judgement. It is sometimes maintained that the disclosure of segmental data constitutes analysis and interpretation and is, therefore, beyond the scope of financial reporting. However, this is a matter of opinion. Whilst analysis and interpretation do usually involve the study or reordering of existing published data, segmental reporting provides additional data not otherwise available. It is difficult to argue, therefore, that the provision of segmental data constitutes analysis and interpretation.

There may be a negative impact on corporate innovation and experimentation. If mistakes are disclosed, management may be inclined to minimise risk to avoid mis-takes, and innovation may suffer. This argument is difficult to assess. In the long run, of course, a lack of innovation will lead to poor performance and dissatisfaction with man-agement. It seems likely that investors will be sufficiently sophisticated to realise that continued success requires innovation, which means that some risks must be taken. The costs of providing segmental information are too high.

Page 288: elliottv10

Barry Elliott and Jamie Elliott: Financial Accounting and Reporting (tenth edition) – Instructor’s Manual

288 © Pearson Education Limited 2006

The objection relates primarily to a fear that disclosure of segmental data may weaken the firm’s competitive position. This objection has been fairly widely researched and the general conclusion seems to be that researchers found that companies rarely ‘if ever, en-counter(ed) any real loss of competitive advantage as a result of segment reporting’.

Page 289: elliottv10

Barry Elliott and Jamie Elliott: Financial Accounting and Reporting (tenth edition) – Instructor’s Manual

289 © Pearson Education Limited 2006

(b) Chapter 28: Question 6 – Filios Products plc

(a) Refer to Chapter 28: Question 5.

(b) Segmental statement (£m)

Classes of business Beer & pub Hotel Other drinks Total business & leisure operations Turnover

Turnover 508 152 368 1,028

Profit

Segment profit (W1) 85 45 18 148

Common costs 15

Operating profit 133

Interest 14

Published net profit 119

Net assets

Segment net assets (W2) 1,127 391 403 1,921

Unallocated assets (W2) 82

Published net assets 1,839

Workings

W1 Sales 508 152 368

Cost of sales 316 81 287

Administration 43 14 18

Distribution costs 64 12 25

423 107 350

Segment profit 85* 45 18

W2 Net assets Fixed assets book value 890 332 364 77

Stocks and debtors 230 84 67

Bank 73 15 28 12

303 99 95 12

Less Current liabilities 66 40 56 31

Net current assets 237 59 39 (19)

Page 290: elliottv10

Barry Elliott and Jamie Elliott: Financial Accounting and Reporting (tenth edition) – Instructor’s Manual

290 © Pearson Education Limited 2006

10% Debentures (140)

Net assets 1,127* 391 403 (82)

* Excluding interest and interest bearing loans as appropriate

(c) Analysis calculations

Accounting ratios: Operating divisions Competitors Beer & Hotel Other Dean Clarke Pub business % % % % % Operating profit % 16.7 29.6 4.9 13.3 40.0

Asset turnover 45.1 38.9 91 46.0 50.0

Return on assets 7.5 11.5 4.5 6.2 20.0

(i) Possible comments:

• The best performing segment, based on the primary accounting ratio, the return on assets, is the hotel division.

• The superior performance of the hotel division is attributable to the fact that it is able to generate higher operating margins than either of the other segments, and this outweighs the fact that it has the lowest asset turnover.

• It would appear that Filios’ beer and pub division pursues a policy of higher selling prices and margins while also endeavouring to maintain asset turnover.

• It would appear that the hotel business division is performing poorly both in terms of cost control and use of assets, and each of these areas requires detailed investigation.

• The third division – other drinks and leisure – is making a contribution to profit of £18m but performance is mediocre by all measures, including a return on net assets of just 4.5%.

(ii)

• In order to interpret effectively the performance of the company, the results achieved by each division need to be compared with its direct competitor.

• The beer and pub division achieved a return on assets higher than that of its competitor, Dean. The beer and pub division has achieved a far higher profit margin that has more than compensated for the marginally inferior asset turnover.

• Filios’ hotel business division performs poorly compared with its competitor, producing a return on assets of not much more than half of that achieved by Clarke. The division’s profit margin and rate of asset turnover are both lower than those of its competitor.

Page 291: elliottv10

Barry Elliott and Jamie Elliott: Financial Accounting and Reporting (tenth edition) – Instructor’s Manual

291 © Pearson Education Limited 2006

• The performance of each of the two divisions for which competitor information is available is not encouraging. The position is even worse when it is recognised that there are unallo-cated common costs of £15m and interest of £14 million. All the indications are that Filios is not being managed in the most effective manner.

• The rate of return on assets earned by Filios Products as a whole (6.5%) is marginally better than that achieved by its competitor Dean, but far below that of Clarke.

Page 292: elliottv10

Barry Elliott and Jamie Elliott: Financial Accounting and Reporting (tenth edition) – Instructor’s Manual

292 © Pearson Education Limited 2006

Chapter 28: Question 7 – Chaldon District Council

Report To Client Services Committee From Accountant Date Subject Roofing Contract: Financial Appraisal of Tenderers

1 Introduction

1.1 Four tenderers, including CDS, have been short-listed for appraisal. The ten-ders have been submitted by:

Tender Name A Nutfield & Sons B Chaldon Direct Services (CDS) C Tandridge Tilers Limited D Redhill Roofing Contractors plc

Objective – to determine to whom the roofing contract should be awarded.

Basis of appraisal Tenderers will be appraised on financial and qualitative grounds. Accounting ratios will be em-ployed to assess profitability, solvency (long and short term), speed of cash collection and payment. Details are provided in Appendix A.

Reference to limitations of approach:

• analysis is indicative only, not definitive

• analysis is based on historical information

• need for several years’ figures in order to consider trends.

2 Interpretation of ratios

2.1 Profitability

Despite having the lowest profit margin, A’s ROCE is the highest at 77% due to its very high asset turnover of 13 times per annum. This is probably a reflection of the nature of the business – a small family concern; this probably also accounts for the firm’s relatively low stockholding. The asset turnover of the other two companies is similar, and C’s higher ROCE is due to its higher margins. D’s stock turnover is considerably higher than its competitors’ which could be a cause for concern.

2.2 Long-term security

A has no long-term debt and, therefore, does not bear any interest charges. The other two com-panies are highly geared with C’s long-term debt being equivalent to its equity finance which is a cause for concern.

Page 293: elliottv10

Barry Elliott and Jamie Elliott: Financial Accounting and Reporting (tenth edition) – Instructor’s Manual

293 © Pearson Education Limited 2006

2.3 Short-term security

D has the best current ratio, although the quick ratio of the three businesses is similar, C having the lowest. Interest cover is only relevant for C & D and does not appear problematic in either given current profit levels.

2.4 Cash flow ratios

A takes longer to settle its creditors and collect from its debtors than the other two companies whose ratios are similar.

3 Other factors

3.1 An analysis of the make-up of the tenders is as follows

Labour Materials Overheads % % %

A 59 35 6

B 63 25 12

C 75 20 5

D 57 34 9

The variation between the components of the various tenders does not provide for any meaning-ful comparison, although CDS (B) does have the highest proportion of its bid for overheads and profit.

Nutfield and Sons (A) have been employed by the Council for small contracts which they have performed satisfactorily. However, this contract is substantially larger than others they have won and, given a workforce of only six, they may not be able to fulfil a contract of this scale.

CDS (B) is obviously well known to the authority and its management have striven to improve its financial position recently in order to achieve a satisfactory rate of return this year.

Tandridge Tilers Limited (C) have not performed satisfactorily on other contracts that they have carried out for the Council.

Redhill Roofing Contractors plc have not been employed by this authority and the standard of their work is not known.

4 Conclusions

Although the financial standing of Nutfield and Sons (A) does not give cause for concern, and although it has submitted the lowest tender bid, there are doubts as to whether it is capable of carrying out a contract of this scale.

Include a comparison of uses in private and public sector.

Main points should include:

Page 294: elliottv10

Barry Elliott and Jamie Elliott: Financial Accounting and Reporting (tenth edition) – Instructor’s Manual

294 © Pearson Education Limited 2006

Private sector

• Weaknesses of historical cost (HC) accounts in times of high inflation; undervaluing assets; overstating profits; not providing for maintenance of capital.

• Can give a better indication of actual profits earned, separates holding gains from earned profits.

• Can give better indication of value of individual assets to the business.

• Based on concept of providing useful information for users of accounts. Not accepted in public sector; accountants not able to agree on bases and methods of adjustments required, or capital maintenance to use.

Public Sector Need to show effective use of public assets. Real-terms measure seen as more appropriate:

• Many public sector organisations have very long lived assets, HC is particularly misleading as result.

• Financial objectives of many public sector bodies are stated in real terms and test discount rates used to evaluate capital projects based on real rates of inflation.

HC is objective and services stewardship function. HC can provide information to enable users of accounts to make their own adjustments, comparisons etc., but fuller disclosure of informa-tion would be required.

There is a wide range of external information available to users of private sector company ac-counts. This is not the case with many public sector bodies.

CCA-adjusted figures argued to be more useful bases of assessing performance.

Does this imply the government views performance evaluation as more important for public sector bodies than investors do for private companies?

CDS (B) submitted the second lowest tender. There is no reason to suspect that it will not be able to deliver the contract to the appropriate standard.

The longer-term financial security (gearing) of Tandridge Tilers Limited (C), the second highest tenderer, and the quality of its work give major causes for concern.

The highest bid was submitted by Redhill Roofing Contractors plc and, although its financial standing does not cause concern, its quality is unknown.

Page 295: elliottv10

Barry Elliott and Jamie Elliott: Financial Accounting and Reporting (tenth edition) – Instructor’s Manual

295 © Pearson Education Limited 2006

5 Recommendation

5.1 It is recommended that the contract be awarded to CDS.

Appendix A: Accounting ratios

Profitability A B C Return on capital employed (%) (ROCE) 77.1 21.2 16.5

Profit margin (%)(PM) 6.0 19.5 17.3

Asset turnover (times p.a.)(AT) 12.8 1.1 1.0

Stock turnover (days) (ST) 17 46 94

Long-term solvency Gearing (%) 0.0 50.0 37.0

Short-term solvency Interest cover (times) n/a 4.0 5.6

Current ratio 0.9:1 1.3:1 1.9:1

Quick ratio 0.7:1 0.6:1 0.7:1

Cash flow ratios

Creditors’ settlement period (days) 59 37 40

Debtors’ settlement period (days) 41 27 29

Notes ROCE = (Operating profit/net assets) × 100 PM = (Operating profit/sales) × 100 AT = (Sales/net assets) ST = (Stock and WIP × 365)/direct costs Gearing = (Non-equity finance/equity finance and non-equity finance) × 100 Interest cover = Net profit before tax and interest/interest payable Current ratio = Current assets: current liabilities Quick ratio = (Current assets – stock and WIP): current liabilities Creditors’ settlement period = (Creditors × 365)/operating costs Debtors’ settlement period = (Debtors × 365/sales)

A C D £000 £000 £000

Turnover 612 1,741 3,080

Operating profit 37 339 534

Net assets (total assets less

Current liabilities) 48 1,600 3,241

Non-equity finance – 800 1,200

Page 296: elliottv10

Barry Elliott and Jamie Elliott: Financial Accounting and Reporting (tenth edition) – Instructor’s Manual

296 © Pearson Education Limited 2006

Equity and non-equity finance 48 1,600 3,241

Interest payable – 85 96

Direct costs 410 1,191 1,735

Stock and WIP 27 149 449

Operating costs 575 1,402 2,546

Current assets 96 290 690

Current liabilities 104 232 356

Current assets excl. stock 69 141 241

Creditors 93 141 280

Debtors 69 131 241

Page 297: elliottv10

Barry Elliott and Jamie Elliott: Financial Accounting and Reporting (tenth edition) – Instructor’s Manual

297 © Pearson Education Limited 2006

Chapter 28: Question 8 – Chelsea plc

(a)

Profitability: ROCE

• Wimbledon outperformed: the industry by 8% and Kensington by 6%. Profit margins:

• Wimbledon follows a high volume/low profit pricing policy

• Low profit evidenced by extremely low profit margin.

• High volume evidenced by the asset turnover figures of 12 and 4 for fixed assets and total assets.

• Kensington and the industry, in contrast, have achieved 2.3 and 1.5; and 5.1 and 2.5 re-spectively. Kensington perhaps moving up market with lower volume/ higher margin.

Cost control:

• Wimbledon’s 7% (12 – 5) shows lower overhead costs as a percentage of sales compared with Kensington and industry averages of 14% and 13%.

Liquidity:

• Wimbledon has a lower debtor collection period and stockholding period – suggests better working capital management than in Kensington.

• Kensington’s acid test ratio of 0.5 appears low compared with 0.9 in Wimbledon and the industry average of 1.3. This appears dangerously low when taking into account the long debtor collection period.

Overall, Wimbledon appears the better investment:

• making better use of assets

• better cost control

• well managed working capital

• potential for borrowing to gear up

• return on equity is healthy.

(b) Matters to be investigated before a final decision can be made:

• Check if activities of companies are actually similar.

• Obtain the absolute figures (£) for turnover, profits, assets etc.

• Determine unexpired economic lives of fixed assets in each company.

• Check quality of management and confirm whether likely to remain.

• Confirm management’s strategy – increased markets or diversification.

• Obtain details of date of redemption of debt.

Page 298: elliottv10

Barry Elliott and Jamie Elliott: Financial Accounting and Reporting (tenth edition) – Instructor’s Manual

298 © Pearson Education Limited 2006

Chapter 28: Question 9

(a) North is achieving a higher profit margin than South or East, with East achieving the lowest margin at 3%.

North South East

(i) Profit/Sales × 100 5% 4% 3%

(ii) Asset turnover 5 times 3 times 4 times

ROCE (i) × (ii) 25% 12% 12%

Whilst North has the highest profit to sales, the effect of the differences in the rate of asset turnover on the comparative performance of South and East means that their ROCE is the same at 12%. The ROCE indicates that North is performing better than South and East which have the same return of 12%. However, there are different levels of gearing as shown by the financial multiplier and when this is taken into account the position is as follows:

North South East

(i) Profit/Sales × 100 5% 4% 3%

(ii) Asset turnover 5 times 3 times 4 times

ROCE (i) × (ii) 25% 12% 12%

Financial leverage 2 4 5

ROE 50% 48% 60%

For a lender North has the lowest gearing and less risk (assuming that there are no other con-tra-indications such a s solvency or liquidity problems), for a minority investor East shows the highest ROE. However, as a measure of management performance, although East has the highest ROE it is underperforming at an operational level. If it were to achieve North’s performance (perhaps under new management) then its ROE would increase to 175% (5% margin × 5 times asset turnover × 5 times financial multiplier).

(b) Decision usefulness of consolidated accounts

Pros:

• Consolidated accounts give an overview of the group’s results and financial position.

• Shareholders in parent company can see how their funds have been invested:

Page 299: elliottv10

Barry Elliott and Jamie Elliott: Financial Accounting and Reporting (tenth edition) – Instructor’s Manual

299 © Pearson Education Limited 2006

• How much of the net assets belongs to the minority shareholders.

• The amount of profit attributable to their shareholding.

• The amount of profit expressed as EPS with implication for share price movements.

• Impossible for a shareholder in parent of a complex group to obtain such information with-out group accounts which are prepared:

• using uniform accounting policies across all group companies

• eliminating inter-group transactions.

Cons:

• Group accounts might combine very disparate companies with different levels of profitabil-ity, liquidity and risk profiles.

• Detailed information on an individual company may be disguised e.g. excessive gearing.

• Some group companies may be making losses – this will only become apparent if the parent decides to sell the loss-making subsidiary and it is reported under discontinued operations.

• Also not possible to identify any extremely profitable subsidiary although this might be-come apparent from the segmental report.

• The volume of intra-group trading and intra-group indebtedness by each company will not be apparent.

• Where there is a minority interest, unrealised profit on intra-group sales will be eliminated 100% although it could be considered that the proportion relating to the minority interest has been realised.

Page 300: elliottv10

Barry Elliott and Jamie Elliott: Financial Accounting and Reporting (tenth edition) – Instructor’s Manual

300 © Pearson Education Limited 2006

Chapter 28: Question 10

PROFILE 31/07/2004 31/07/2003 31/07/2002 31/07/2001

12 months 12 months 12 months 12 months

Turnover 787,126 730,913 601,295 483,968

Profit (Loss) before Taxation

46,316 56,139 53,568 44,317

Net Tangible Assets (Liab.)

677,710 680,989 660,447 575,223

Shareholders Funds

288,954 318,628 310,133 273,839

Profit Margin (%) 5.88 7.68 8.91 9.16

Return on Share-holders Funds (%)

16.03 17.62 17.27 16.18

Return on Capital Employed (%)

6.83 8.24 8.11 7.70

Liquidity Ratio 0.22 0.24 0.24 0.29

Gearing Ratio (%) 143.19 121.51 120.96 110.06

Page 301: elliottv10

Barry Elliott and Jamie Elliott: Financial Accounting and Reporting (tenth edition) – Instructor’s Manual

301 © Pearson Education Limited 2006

C H AP T E R 2 9

Chapter 29: Question 1 – Wandafood Products plc

• Profit to sales was low in 20×7.

• Decline in return on assets in 20×7.

• Consider the profit to turnover and asset turnover movements.

• Interest and dividend cover is falling from 20×6.

• Further decline could create problems.

• There is an upward move in the level of borrowings.

• Debt to equity as indicated by the relationship between the net borrowings and the sum of net borrowings and shareholders’ funds is 1:10 in 20×6 and 1:2.5 in 20×9 indicating in-creased borrowing.

• Liquidity as indicated by the trend in the liquid and current ratios is levelling out.

Asset ratios

• Sales to working capital ratio shows a steadily increasing trend. May indicate more efficient use of working capital.

• Assets per share are increasing indicating that the company is ploughing back profits.

Page 302: elliottv10

Barry Elliott and Jamie Elliott: Financial Accounting and Reporting (tenth edition) – Instructor’s Manual

302 © Pearson Education Limited 2006

Chapter 29: Question 2 – Bouncy plc

(a) Ratios for a potential shareholder

20×6 20×5

(i) Return on equity Profit after tax and Preference divi-

dends /Ordinary share capital +

Reserves

1,300/6,700

= 19.4%

900/5,650

= 15.9%

(ii) Earnings per

share

Profit after tax and Preference divi-

dends/No.of Ordinary shares

1,300/6000

= 21.67p

900/6,000

= 15p

(iii) Dividend cover Equity profits/Proposed dividend 1,300/250

= 5.2 times

900/250

= 3.6 times

(iv) Gearing Debt capital/Debt + Equity 1,500/8,200

= 18.3%

1,500/7,150

= 21.0%

(b) Solvency ratios for a potential lender

(i) Debt equity Debt:Equity 1,500:6,700 = 1:4.5

1,500:5,650 = 1:3.8

(ii) Solvency Current assets:Current liabilities 3,810:1,960 = 1.9:1

3,610:2,060 = 1.8:1

(iii) Interest cover Profit before interest:Interest 2,200/170 = 13 times

1,570/150 = 10 times

(iv) Liquidity Current assets – Stock:Current li-abilities

1,710:1,960 = 0.87:1

1,540:2,060 = 0.75:1

(c) Comments from potential shareholder’s viewpoint

The return on equity has improved by approximately 25%. The dividend is well covered and has improved in 20×6 from 3.6 in 20×5 to 5.2 in current year. The EPS figure is in line with the re-turn on equity and is acceptable. The gearing is low at 18.3% so that the business enjoys lower earnings risk.

Comments from viewpoint of lender

The current ratio at 1.9 and acid test ratio at 0.87 are both improving and interest is well covered at 13 times. Gearing is low and coupled with the improving return on equity and sound interest cover means that the company is able to increase its long-term borrowing.

The increase in the share price over the last three years is understandable given the picture pre-sented by the ratios.

Page 303: elliottv10

Barry Elliott and Jamie Elliott: Financial Accounting and Reporting (tenth edition) – Instructor’s Manual

303 © Pearson Education Limited 2006

(d) Advising on scheme to choose

It is interesting to assess the schemes from their impact on earnings per share and return on equity. Assuming a rights issue £000 £000 Profit before interest and tax 2200

Interest expense currently (170)

Less: Debenture interest (10% of £1.5m) 150

Bank charge interest (20)

2180

Taxation (730)

Loss of interest allowance 40% of 150,000 (60)

(790)

Revised profit after tax 1390

Earnings per share:

Shares in issue £3,000,000/£0.5 = 6,000,000

New shares £6,000,000/£1.5 = 4,000,000

10,000,000

EPS = £1,390,000/10,000,000 = 13.9p

Return on Equity = 1,390/(6,700+6,000) × 100 = 11%

13% Debentures

£000 £000 Profit before interest and tax 2,200

Interest expense (6,000 × 13%) (780)

1,420

Taxation 730

Less tax savings on loan interest (780 – 170) × 40% (244)

486

Revised profit 934

EPS = 934/6,000 = 15.6p

Return on Equity = (934/6,700) × 100 = 14%

The decision based on EPS and return on equity supports the loan funding scheme.

Other factors to be taken into account:

Consider the increase in gearing from 18.3% to 47.2% (6,000/12,700)

Page 304: elliottv10

Barry Elliott and Jamie Elliott: Financial Accounting and Reporting (tenth edition) – Instructor’s Manual

304 © Pearson Education Limited 2006

Chapter 29: Question 3 – Liz Collier

(a) Option 1 Year 1 Year 2

£ £ Profit [21,000 × 140/100] 29,400 29,400

Less interest at 10% per annum 1,000 1,000

28,400 28,400

Comments:

(i) The loan of £10,000 is paid out of incremental cash flow generated by profit in 14.5 months.

(ii) It is likely that some benefit will continue after the end of year 2 and marginally improve her lifestyle.

(iii) It is assumed that the 40% increase is a reasonable and feasible forecast.

Option 2 Year 1 Year 2

£ £ Partnership profit 39,600 39,600

[profit is £33,000 × 120/100]

Less cost of amalgamation (6,870)

Less salaries

Liz – 2% of £126,000 (2,520) (2,520)

Joan – 2% of £72,000 (1,440) (1,440)

28,770 35,640

Profit share:

Liz 3/5 (17,262) (21,384)

Joan 2/5 (11,508) (14,256)

Comment:

(i) Liz will receive

Salary 2,520 2,520

Profit share 17,262 21,384

19,782 23,904

Liz is worse off in year 1 by £1,218 [21,000 – 19,782] and better off by £2,904 in year 2. Her share of the initial investment is £4,122 i.e. 3/5 of £6,870. This investment will be repaid in 2.5 years and the benefit will accrue in perpetuity. From year 2 onwards it generates a ROCE of 70% i.e. £2,904/£4,122. It seems a good proposal assuming the figures are reliable and that the partners are able to work in harmony. There is potential for expansion with synergy effect.

Page 305: elliottv10

Barry Elliott and Jamie Elliott: Financial Accounting and Reporting (tenth edition) – Instructor’s Manual

305 © Pearson Education Limited 2006

Joan will derive a benefit in year 1 and a higher return in subsequent years i.e. £3,696/£2,748 × 100 = 134% in year 2. This might indicate that the profit-sharing ratio is unfair to Liz and should be reviewed if this option is selected. Option 3 Year 1 Year 2 Profit [£21,000 × 8/10] 16,800 16,800

Franchise profit 15,000 17,250

Less interest @ 10% per annum (8,000) (8,000)

Total profit 23,800 26,050

Comments:

(i) Incremental profit compared with

present position is: £2,800 £5,050

(ii) Franchise projected profit:

Year 3: £19,838; Year 4: £22,813; Year 5: £22,813

It will take 6 to 7 years to repay £80,000 from incremental cash flows. After year 8 it could be a very profitable proposition.

(b) Option 1 gives a 40% increase over two years. It is unlikely that this increase can be main-tained in year 3 and subsequent years without additional expense on advertising etc. The initial outlay is moderate and is repaid quickly from additional cash flow. Liz will maintain her inde-pendence and improve somewhat her standard of living/lifestyle.

Option 2 shows a reduction in profit in year 1 compared with the present and a £2,900 increase thereafter in year 2 and subsequently. The initial outlay is moderate and there may be longer-term prospects without additional expense after year 3. There is however a loss of independ-ence as a partner. There may be hidden costs not provided for and high opportunity costs.

Option 3 requires substantial investment of £80,000 which may be repaid until about year 7 out of incremental cash flows.

(c) Reservations

Option 1

• The ability to increase turnover by 40% and the maintenance of the level of sales after year 2.

Option 2

• The ability to work amicably with Joan in the partnership.

• Risk of poor decisions by the other partner which then bind the firm.

• Possibility of administration costs not included in the estimates given.

• Basis of profit-sharing ratio seems to be biased in favour of Joan.

Page 306: elliottv10

Barry Elliott and Jamie Elliott: Financial Accounting and Reporting (tenth edition) – Instructor’s Manual

306 © Pearson Education Limited 2006

Option 3

• Need to reduce existing sales.

• Involvement with franchise constitutes a refocusing of the business with attendant risks.

• Reliability of the estimates particularly after the first 2 years.

Page 307: elliottv10

Barry Elliott and Jamie Elliott: Financial Accounting and Reporting (tenth edition) – Instructor’s Manual

307 © Pearson Education Limited 2006

Chapter 29: Question 4 – Chekani plc

(a) Principles of share valuation:

• No one method of valuing shares.

• Accounting values only a guide as to starting point and/or maximum value for negotiating between parties.

• Share value is the amount that the buyer is willing to pay and the vendor willing to accept.

Methods or bases of valuation:

• Dividend yield basis.

• Price/earnings ratio.

• Net asset value based on a going concern or break-up value.

• Valuation of an unlisted company of this type needs to take account of the difficulty associ-ated with selling the shares and the increased risk of the investment, by adjusting quoted company yields or PE ratio.

• Minority holdings usually valued on dividend yield basis, which is not appropriate for ma-jority holdings of this type.

• Majority holdings valued on PE basis, using adjusted quoted company PEs; the net asset value, as a going concern, gives a minimum valuation.

(b) Alternative valuations (see appendix for calculations)

Dividend yield basis.

For calculations, see appendix. Discussion of investor’s required rate of return, quoted com-pany yields and possible uplift % for an unlisted company.

Appendix shows £1.28 (with 30% adjustment for unquoted status). Method not appropriate here.

Net asset basis

For calculations, see appendix. Discussion of treatment of:

• contingent liability

• purchased goodwill unamortised

• book value v current market value of assets

• relating goodwill and current market values.

Page 308: elliottv10

Barry Elliott and Jamie Elliott: Financial Accounting and Reporting (tenth edition) – Instructor’s Manual

308 © Pearson Education Limited 2006

Preference shares

• These could be deducted at their nominal value as the liability due to outside interests.

• Alternatively, the better student could argue that the approach to be adopted should be to value the company as a whole, e.g. Gross asset value, the amount Chekani would pay for the whole enterprise, including the preference shares, and then to deduct the value of the prefer-ence shares, i.e. total value less the amount that will not be acquired.

Debentures

• Either the premium is an extra cost to Chekani and is ignored in the valuation,

• or it is included on the basis that it is part of the value of the company’s net assets.

Earnings basis

Calculations per appendix

• Comment on use of P/E ratios for quoted companies and discount rate applied.

• Treatment of exceptional item, maintainable earnings.

(c) Appropriate strategy

• Normally do not sell below net asset value, based on revalued amounts. This is the highest price here, £3.02, and may not be acceptable to the purchaser due to factors such as:

• falling profits over last three years, return on investment may be more important than asset value

• value includes estimate of current values which may not be realisable

• future prospects and forecasts are not given and cannot be taken into account, but they could provide evidence to support a higher (upturn in prospects expected) or lower (no improvement or downturn forecast) price.

• The final price is likely to be between the two net asset basis prices, the revalued asset basis figure £3.02 and the value based on net assets per balance sheet, £2.175.

• Opening bid should be around the higher figure, say £3.00, leaving room to negotiate downwards, possibly with a view to agreeing a final price around £2.60.

Appendix

Dividend yield

Net dividend on ordinary shares £3m

Dividend per share £3 million/40 million 7.5 pence

Investors’ required rate of return, based on average gross yield of the two quoted companies given, (4.9 + 4.1)/2 = 4.5%

Page 309: elliottv10

Barry Elliott and Jamie Elliott: Financial Accounting and Reporting (tenth edition) – Instructor’s Manual

309 © Pearson Education Limited 2006

Value per share: No allowance for different risk (£1 × 7.5/4.5) = £1.67 per share Adjusted yields, allowing for increased risk of unquoted share, using 30% (Note: other adjust-ment % are permissible), (4.5% × 1.3) = 5.85%, so £1 × 7.5/5.85 = £1.28 per share.

Asset valuation basis Assets at balance Assets revalued/

sheet values goodwill written off etc. Goodwill 15,000 0

Property 30,000 56,250

Plant 60,000 60,000

Investments 15,000 22,500

Net current assets 12,000 12,000

Contingent liabilities (3,000) (3,000)

10% Debentures (30,000) (33,000)

Preference shares (12,000) (10,800)

87,000 103,950

Value per share 87,000/40,000 = £2.175 103,950/40,000 = £2.60

Other variations could be: Goodwill not written off 103,950 + 15,000 = 118,950

118,950/40,000 = £2.97

Debentures and preference shares 103,950 + 1,800 = 105,750

at nominal value 105,750/40,000 = £2.64

Goodwill not written off and 103,950 + 15,000 + 1,800 = 120,750

debenture/preference at nominal 120,750/40,000 = £3.02

value (maximum valuation of assets)

Page 310: elliottv10

Barry Elliott and Jamie Elliott: Financial Accounting and Reporting (tenth edition) – Instructor’s Manual

310 © Pearson Education Limited 2006

Valuation on an earnings basis Capitalised using PE ratio of similar listed companies:

Average of 11.3 and 8.2 = 9.75 Discount at 30% = 6.825

Earnings:

Profit before interest and tax 21,000

Interest (3,000)

18,000

Tax 5,550

12,450

Preference dividends 840

11,610

EPS = 11,610/40,000 = 29.025 pence per share

29.025 × 9.75/100 = £2.83

Discounted

29.025 × 6.825/100 = £1.98

Page 311: elliottv10

Barry Elliott and Jamie Elliott: Financial Accounting and Reporting (tenth edition) – Instructor’s Manual

311 © Pearson Education Limited 2006

Chapter 29: Question 5 – Johnson Products Ltd

(a) (i) Sale of shares to Sonar Products Ltd

The 75% holding constitutes a controlling interest and can be valued on an earnings basis to in-dicate the amount that the buyer could offer and reasonably assume would be acceptable to the seller.

A valuation on an earnings basis gives a value of approximately £300,000 or 37p per share. The value is computed using the following formula:

Value = Earnings/% earnings yield required

For this part of the question we need to estimate the amount of the earnings that are to be capi-talised and the percentage earnings yield required from the information given in the question. The earnings could be based on the final year figure or perhaps a weighted average. For the purpose of illustration the weighted average is being used in this solution, calculated as follows:

Earnings Weight Product £ £ 79,400

(27,600)

56,500 1 56,500

88,300 2 176,600

97,200 3 291,600

6 524,700

Average earnings = £524,700/6 = £87,450

Note that in part (c) of the question there is a further discussion required of the principal matters that need to be taken into account when assessing future maintainable earnings.

The percentage earnings yield required is based on the information provided in the question about the three other companies:

Gross dividend Retention Earnings % yield % % yield Eastron 15 25 20

Westron 10.5 16 12.5

Northron 13.4 20 16.75

The average percentage earnings yield = (20 + 12.5 + 16.75)/3 = 16.4%.

Based on the estimated average earnings which are regarded as maintainable and the estimated percentage earnings yield required, the valuation of the 75% shareholding is as follows:

Page 312: elliottv10

Barry Elliott and Jamie Elliott: Financial Accounting and Reporting (tenth edition) – Instructor’s Manual

312 © Pearson Education Limited 2006

Value of company = £87,450 × 100/16.4 = £533,232

Value of 75% = £533,232 × 75/100 = £399,924

Less say 25% for lack of marketability £99,981

£299,943

This value is an estimate of the amount that would be acceptable to R. Johnson.

(ii) Sale of shares to the staff

• The possible sale to the staff would result in a widely held share capital with no single person holding in excess of 4% of the share capital.

• Consequently it is felt that the shares should be valued on a dividend basis using the formula that the value of a share would be the dividend divided by the percentage dividend yield required less 25% for lack of marketability.

• The dividend is assumed to be 5p per share and the percentage dividend yield re-quired is estimated at 12.97 being the average of the yields for the three comparator companies.

The value of a share = 5/12.97 × 100 = 38.55p

Less 25% 9.64

Value per share 28.91

Value of 810,000 shares £234,171.00

(iii) Sale to Divest plc

The realisable value of the business is:

Land 480,000

Premises 630,000

Equipment 150,000

Stock 98,000

Debtors 168,000

Cash 70,000

Creditors (335,000)

Non-current creditors (158,000)

Realisable value 1,103,000

Less 16.7% (based on the need to obtain 20% return) 183,833

Value of business 919,167

Value of 75% 689,375

The three values are therefore:

(i) Sale to Sonar Products Ltd £299,943

(ii) Sale to minority interests £234,171

(iii) Sale to Divest plc £689,375

Page 313: elliottv10

Barry Elliott and Jamie Elliott: Financial Accounting and Reporting (tenth edition) – Instructor’s Manual

313 © Pearson Education Limited 2006

(b) Maximum that would be offered by Sonar Products Ltd on basis of information provided in the question

The valuation would be calculated on a return on capital basis to indicate the maximum amount the buyer would be prepared to offer.

The maximum that Sonar Products would be prepared to pay may be estimated by reference to the rate of return that they presently achieve. Given that they currently achieve a rate of return on capital employed of 12.5% the amount they would regard as maximum is £524,700 calcu-lated as follows:

Average Capitalised at % Holding Maximum earnings % return on value capital

£87,450 × (100/12.5) × (75/100) = £524,700

(c) Principal matters to take into account when estimating future main-tainable earnings

There are a number of matters that could be mentioned and in this answer a selection of relevant matters is given. There are others that could be put forward as satisfactory replies to this ques-tion.

(i) Past performance

• Past performance, i.e. past earnings, is the main indicator of future potential.

• One cannot merely carry out an extrapolation of the past three to five years

• But it is an indication of how well the company has operated in the past in comparison with other companies within the same industry.

• This means that one would need to obtain information about the earnings of the three com-parator companies over say the past three to five years and assess how well Johnson Products Ltd has fared in comparison with these.

• One could pay attention to the compound annual growth rates in sales and operating profits and profits for the year, and look at the implication of financial and operating gearing.

(ii) Forecast for the industry

• It is important to form a view on the possible growth or decline within the industry sector in the future.

• Although the past earnings are the base from which accountants start, they also needs to have regard to the expected movements within the industry in the future.

• We are attempting to estimate future maintainable earnings and clearly the rate of growth in the industry is important.

Page 314: elliottv10

Barry Elliott and Jamie Elliott: Financial Accounting and Reporting (tenth edition) – Instructor’s Manual

314 © Pearson Education Limited 2006

(iii) Changes in the activities undertaken

• The activities that generated the past earnings will be known.

• It is important to identify the extent to which these will be varied.

• There are various indicators that will be apparent from an examination of the accounts themselves, such as

• research and development expenditure

• new fixed assets

• capital investment contracts outstanding at the balance sheet date, and even

• surplus funds that are not currently invested within the business because they indicate the capacity to move into new activities or to expand the level of existing operations.

(iv) Rationalisation

• Consideration needs to be given to the likelihood of the acquirer selling off parts of the ac-quired company in order

• to improve performance or

• to release cash for the payment of interest or for other purposes.

(v) Management and staff

• These are an important component for success in any business.

• It is possible to gain an impression from the accounts and filed documents of average wage levels and the stability of the board.

• However, to obtain more detailed information, it would be necessary to have the co-operation of the company because one would be seeking more detailed information on

• service contracts

• performance-related pay

• the rate of labour turnover.

• It is clearly more fruitful if it is possible to obtain the co-operation of management to obtain these data.

(vi) Accounting policies

• If it is assumed that the new owners will be able to control the accounting policies then clearly it is of interest to identify how the past policies will be varied. For example, there are the areas such as depreciation and long-term contracts where the company might follow a more or less conservative accounting policy.

Page 315: elliottv10

Barry Elliott and Jamie Elliott: Financial Accounting and Reporting (tenth edition) – Instructor’s Manual

315 © Pearson Education Limited 2006

(vii) nterim accounts

• If it is possible to obtain access then the interim accounts, management accounts, budgets and forecasts will give an indication of the company’s strategy and its success over the im-mediate past few months.

• This could give a more current feel for the company’s progress.

(viii) Ratio analysis of balance sheet

• The matters referred to in paras (ii)–(vii) are specifically towards the future.

• Whilst the emphasis is on the future we also need to refer to the last balance sheet to pick up items such as

• high gearing,

• poor liquidity

• references to post balance sheet events, or

• contingent liabilities that might impact on the future prospects of the company.

• In conclusion therefore the question is looking for a recognition that the valuer needs to be forward looking, identifying as clearly as possible the future maintainable earnings.

Page 316: elliottv10

Barry Elliott and Jamie Elliott: Financial Accounting and Reporting (tenth edition) – Instructor’s Manual

316 © Pearson Education Limited 2006

Chapter 29: Question 6 – Business Risk Management

(a) Identification and prioritisation of risks

All types of risk are relevant to an existing or potential shareholder including both downside risks (possible losses) and volatility risks (possible gains or losses). Shareholders are not pro-tected if they only receive details of downside risks and sell their shares inappropriately.

Developments to date have been aimed at addressing a particular problem e.g. SSAP 25 Seg-mental Reporting. This has meant that companies have had prescriptive requirements which might not have reflected the actual risks which are relevant to their company.

Risk that may be relevant include:

• product or service failure

• new regulations

• product development with heavy R&D costs before cash flows in.

Internal risks include:

• process risks e.g. arising from employees such as risk of losing key staff, suppliers and manufacturing process whereby products are not delivered on time or to correct specifica-tion.

• financial risks e.g. price, liquidity and credit risks.

External risks include:

• social, political and economic forces, e.g. risk of new employee protection regulations

• financial risks, e.g. exchange rate movements.

Risk prioritisation

The normal materiality criterion applies and attention should be drawn to risks in accordance with their significance.

(b) Managing risk

There are different views on the nature of the disclosure. One view is that it is sufficient to con-firm that the company has complied with the Combined Code.

There is also the view that there should be detailed disclosure of particular steps taken e.g. in-surance, hedging, outsourcing.

(c) Measuring risk

There is a wide range of measures that could be applied to measuring risk and it is important not to concentrate only on deterministic data e.g. potential losses on exchange, but also to consider how to report on strategic risks.

Page 317: elliottv10

Barry Elliott and Jamie Elliott: Financial Accounting and Reporting (tenth edition) – Instructor’s Manual

317 © Pearson Education Limited 2006

Accounting measures already exist internally, e.g. reporting provisions and contingencies and producing ratios such as gearing and liquidity, trend analysis and benchmarking.

Accounting measures also exist externally, e.g. bond rating by credit agencies, benchmarking.

Non-accounting measures are also important, e.g. price competitiveness, delivery times, level of warranty claims.

Page 318: elliottv10

Barry Elliott and Jamie Elliott: Financial Accounting and Reporting (tenth edition) – Instructor’s Manual

318 © Pearson Education Limited 2006

C H AP T E R 3 0

Chapter 30: Question 1

The IFRS Taxonomy has the element Inventories identified as bearing the name of Inventories, being of the type monetaryItemType and that it is expected to have a debit balance <element id="ifrs-gp_Inventories" name="Inventories" type="xbrli:monetaryItemType" substitutionGroup="xbrli:item" xbrli:periodType="instant" xbrli:balance="debit" nillable="true" /> As the explanations in this text do not go in detail as to the different structures of all the files used in XBRL, it is not expected that students would be able to reproduce this type of coding. Students may have visited the example at http://www.xbrl.org/Example1/ and come up with the following: <ifrs-gp:Inventories ContextRef=”Current_AsOf” UnitRef=”U-Euros” Decimals=”0”>100000</ofrs-gp:Inventories> Although this is an example of XBRL works, the coding really is from an Instance Document and not from the underlying schemas etc. If the students followed the naming conventions used (see http://xbrl.org.au/training/ XBRLNamingConventions.pdf) then it is probably more likely that students come up with: CurrentAssets Inventory Or: CurrentAssets CashAndCashEquivalents Inventory The placement of Inventory depends thus on the standard’s position as to Inventory belonging to the class of Cash and Cash Equivalents or to Other Current Assets. Agricultural assets may well be split between Current and Non Current (Timber Trees or planta-tions) Another good overview example of the application of the IFRS can be found at http://xbrl.iasb.org/int/fr/ifrs/gp/2005-01-15/Samples.htm

Page 319: elliottv10

Barry Elliott and Jamie Elliott: Financial Accounting and Reporting (tenth edition) – Instructor’s Manual

319 © Pearson Education Limited 2006

Chapter 30: Question 2

List the steps in the approval process for a taxonomy to be considered to have ‘final’ status. (Hint: look at www.xbrl.org ) Students would have looked at http://www.xbrl.org/FormingAJurisdiction/ and perused the zip file and found that there are 2 different classifications for Jurisdictions as listed in the table below: Established Jurisdiction A non-profit organisation, involved in business reporting, that col-

lects dues from ten or more members and pays dues to XBRL

International.

Provisional Jurisdiction A non-profit organisation involved in business reporting that has a

group of individuals (often called a working party) creating an

XBRL Taxonomy or other XBRL work products, and that pays dues

to XBRL International.

Jurisdictions are normally established within a country where a particular accounting standard is applicable. Should a country adapt the IFRS then, technically, it may not need to form a jurisdiction. Should there be a need to adapt (XBRL refers to this as ‘extend’) then there will be a need if the country needs to use XBRL.

Page 320: elliottv10

Barry Elliott and Jamie Elliott: Financial Accounting and Reporting (tenth edition) – Instructor’s Manual

320 © Pearson Education Limited 2006

C H AP T E R 3 1

Chapter 31: Question 1 – BC

(a) Provision at 31.12.×0 = no. of shares × option price × probability of achieving the option × 1/3 = 1,000,000 × 0.05 × 0.6 × 1/3 = 10,000

Provision at 31.12.×1 = no. of shares × option price × probability of achieving the option × 2/3 = 1,000,000 × 0.10 × 0.7 × 2/3 = 46,667

Provision at 31.12.×2 = no. of shares × option price × probability of achieving the option = 1,000,000 × 0.15 × 0.85 = 127,500

Provision at 31.12.×3 = no. of shares × option price × probability of achieving the option = 1,000,000 × 0.30 × 1.0 = 300,000

Final cost at 30.6.×4 = no. of shares × option price × probability of achieving the option = 1,000,000 × 0.35 × 1.0 = 350,000

Year ended Provision P&L a/c charge 31 Dec 20×0 10,000 10,000

31 Dec 20×1 46,667 36,667

31 Dec 20×2 127,500 80,833

31 Dec 20×3 300,000 172,500

31 Dec 20×4 50,000

Total charge 350,000

There will be no provision in the financial statements for the year ended 31 December 20×4, as the option will have been awarded to the director. The total cost of the option at 30 June 20×4 will be charged as directors’ remuneration and credited to the share premium account.

Page 321: elliottv10

Barry Elliott and Jamie Elliott: Financial Accounting and Reporting (tenth edition) – Instructor’s Manual

321 © Pearson Education Limited 2006

(b) The total cost of the share option using the different methods will be: Method Total cost IAS 37 £350,000

US FAS123 & IFRS 2 £30,000

UK recommendation £300,000

(c) (i) The IAS 37 method gives a cost for the option which is consistent with the International Accounting Standard and the IASC’s Framework for the Preparation and Presentation of Financial statements. Taking the value of the option and the probability of the option being granted tends to give an uneven charge for the option, with a higher charge in later years. This example tends to exaggerate this uneven charge, but the charge will tend to be higher in later years because the probability of the option being granted is higher near the end of the option entitlement period (and less in earlier years). Also, there is an element of discounting in determining the option price, so the option price will be lower in earlier years than later ones. In addition, the option price will be lower in earlier years as it is less certain that the share price will be above the option price in earlier years than later ones.

(ii) In this example, FAS 123/IFRS 2 gives a charge to the profit and loss account of only 8.6% of the cost of the IAS 37 method (and only 10% of the UK recommendation). This charge occurs in the year ended 31 December 20×0, and there is no subsequent charge in the profit and loss account.

(iii) The UK recommendation gives a charge of 86% of the IAS 37 method, which is a more realistic figure than using the second method. However, the charge only occurs at 31 December 20×3, which is 3 years after the option was granted to the director. It would seem more appropriate if the charge was made during the period the director earned the entitlement to the share option.

Page 322: elliottv10

Barry Elliott and Jamie Elliott: Financial Accounting and Reporting (tenth edition) – Instructor’s Manual

322 © Pearson Education Limited 2006

Chapter 31: Question 2 – Rolls Royce

(a) The composition of the Remuneration Committee is appropriate as it comprises entirely non-executive directors. It is good that they meet regularly and receive appropriate profes-sional advice. However, the statement that they ‘meet regularly’ is vague and could be more specific, stating the number of meetings held in the year ended 31 December 1999.

There appears to be a slight conflict in that the Chairman and Chief Executive attend the meetings. They are virtually members of the Remuneration Committee. However, the Re-muneration Committee need relevant information on the performance of the executive directors, and one good way of obtaining this is from the Chairman and Chief Executive. Also, the Chairman and Chief Executive may have relevant information on comparable salaries for directors in other similar companies.

However, there may be problems with the Chairman and Chief Executive attending the meeting, as the Remuneration Committee may want to discuss information provided by the Chairman and Chief Executive but with both the Chairman and Chief Executive being ab-sent from the meeting. For instance, it would not be possible for the non-executive directors to discuss the point ‘do we believe what the Chairman and Chief Executive are saying’ when those people are present at the meeting. Thus, the non-executive directors should have the opportunity to discuss matters alone. The note to the financial statements suggests that either the Chairman or Chief Executive would be present at the meeting. The likely close relationship between the Chairman and Chief Executive could result in the Chairman disclosing to the Chief Executive confidential matters relating discussions by the Remunera-tion Committee on the Chief Executive’s performance and remuneration.

One would be more confident about the independence of the Remuneration Committee if the financial statements disclosed that the non-executive directors of the Remuneration Committee have the right to discuss matters without the Chairman and Chief Executive be-ing present.

(b) Like Diageo plc, the statement about ‘Base Salary’ is qualitative rather than quantitative. It appears that the ‘median-level base salary’ would be given even if the performance of the directors is poor. Also, mention of ‘performance-related schemes’ does not seem relevant and appropriate in a section discussing ‘Base Salary’.

As explained in the main body of the Chapter, it is difficult to determine the effectiveness of directors’ performance. The financial performance of a company may be poor because the directors’ performance is poor, or the performance may be poor because of a recession in the market for the company’s products. Also, directors may work very hard when the com-pany is performing poorly to overcome the problems it is experiencing. Directors should be paid on how hard and effectively they work, and financial performance may be a poor measure of their performance.

(c) This paragraph explains that the annual performance award scheme gives a performance award of up to 60% of basic salary. It talks about a ‘reducing scale of maximum percent-ages for senior employees’ - are these non-directors or do they include directors? It is unclear about whether the performance award is graduated, so that the achievement of the full performance requirement obtains 60% of salary, with reducing figures for lower achievement of the performance. Also, at what point is no performance award made?

Page 323: elliottv10

Barry Elliott and Jamie Elliott: Financial Accounting and Reporting (tenth edition) – Instructor’s Manual

323 © Pearson Education Limited 2006

On the award itself, it says that a third of the award is paid in Rolls-Royce shares. How and when is the charge for these shares made in the financial statements? It would appear that the charge should be made when the shares are purchased. However, the charge could be avoided at that stage by including the shares at cost in the balance sheet. If a third of the award is made in shares, then two-thirds must be made in ‘cash’. When is the charge for the cash part of the award made? This charge is probably when the payment is made, whereas it would be better to make the charge during the period when the director works to achieve the performance target.

The final statement in this paragraph says the performance award ‘provides a culture of share ownership amongst the Group’s senior management’. From the company’s financial statements, it is very difficult to see whether this is happening. The directors held more (or the same number of) shares at 31 December 1999 than they did a year earlier, but there was no information on the number of shares issued to directors in that period.

(d) Once again, the directors can obtain an award of 60% of salary under the LTIP. This seems like ‘double counting’ as they can also obtain 60% of their salary from the annual perform-ance award scheme (APAS). This gives a total possible bonus of 120% of the base salary.

The wording of the paragraph is very similar to that for the APAS, so the comments are similar. There is maximum award of 60% of salary, but is this proportionally reduced when the full target is not met?

There is some explanation of how the award is calculated, and no award is made if the com-pany’s performance is below average (i.e. 10th out of 19 or below). The timing of the charge for the LTIP to the profit and loss account is uncertain, in a similar way to the APAS. It is probable that the cost is charged when the shares are purchased, but the cash portion is only charged when the payment is made to the director, rather than during the pe-riod when the director is working to achieve the target.

The last paragraph is clear in explaining that no award was made in the year ended 31 De-cember 1999, and none was realised (i.e. paid).

(e) The ‘Combined Code’ recommends that rolling contracts should be no more than a year, whereas the company is providing a rolling contract of two years to most of its executive di-rectors. The ‘rolling contract’ means that if a director is dismissed, he/she is paid one (or two) years’ salary on termination.

One would have to look at the financial statements of other companies to see if they are us-ing a one-year rolling contract for directors, or a longer period. It appears that most companies now comply with the one-year term of the ‘Combined Code’, so Rolls-Royce’s terms are generous to its directors.

On new directors who are initially given a two-year rolling contract, there is no explanation of the ‘initial period’ after which the contract is reduced to one year.

(f) The first paragraph under ‘Compensation and mitigation’ seems reasonable, as it appears to prevent incompetent directors from receiving a large termination payment. Also, if a direc-tor was close to the retirement age it would be unreasonable to give the director a termination payment which extends beyond the retirement age. For example, if the director was exactly 64 and the retirement age was 65, then the termination payment should be lim-ited to 1 year, so it only covers the period until he is 65. So, the first paragraph seems reasonable.

Page 324: elliottv10

Barry Elliott and Jamie Elliott: Financial Accounting and Reporting (tenth edition) – Instructor’s Manual

324 © Pearson Education Limited 2006

The second paragraph gives an example of the application of the first paragraph. The com-pensation paid to the director seems generous and there are many unanswered questions. Although the individual was a director until the company’s year end of 31 December 1999, he is not included in the list of directors and their biographies. So, there is no disclosure of the age of the director at 31 December 1999 and his position in the company. One would have to look at the previous year’s financial statements for this information, and, like many other shareholders, I have thrown away the 1998 financial statements! Thus, the financial statements should give biographical details of all directors who served the company during the year. There is a need for such a requirement under the Companies Acts or Stock Ex-change codes.

The other unanswered question is ‘why did the director retire early?’ The Chairman’s statement only thanks the retiring director for his services, and the only other information on this payment is given in this question. How long had the director served the company and in what positions?

In practice, it is unlikely that companies will disclose reasons why directors leave the com-pany or retire early. Sometimes, they express appreciation for the director’s contribution to the company. However, there may have been heated disagreements between the directors with the losing directors leaving the company. These ‘losing directors’ may be given a gen-erous termination payment on the understanding they will ‘keep quiet’ about the dispute. There is likely to be little or no mention of these problems in the company’s financial state-ments. Also, full disclosure of the problems by the company is likely to lead to expensive legal action by the ‘losing directors’ against the company.

It is probably much better to obtain information about the director’s resignation from the fi-nancial press than from the company’s annual financial statements.

From the figures given in the question, it is apparent that the retiring director has been given two years’ basic salary in compensation payment. This is very generous. Also, there is al-most no disclosure of information about the director in the financial statements, so it is impossible to determine whether there should be any reduction in the termination payment, using the rules in the first paragraph of the statement.

It appears that many UK companies are very generous in their termination payments to di-rectors. Very few UK employees who are not directors are given two years’ termination payment when they retire early.

Some directors can earn large sums from termination payments. For instance, a director working for company A may obtain a job at company B in six months’ time. Then, he makes a nuisance of himself at company A which results in him being dismissed from com-pany A with a generous compensation payment. He then moves on to company B, obtains a job at company C, makes a nuisance of himself at company B and is dismissed and so on!

Page 325: elliottv10

Barry Elliott and Jamie Elliott: Financial Accounting and Reporting (tenth edition) – Instructor’s Manual

325 © Pearson Education Limited 2006

Chapter 31: Question 3 – Risk-Averse Auditors

Explanations of why auditors perform more work than is necessary include:

(d) being sued involves a lot of scarce senior management time

(e) it is very worrying to be sued, so auditors would want to keep the number of claims to a minimum

(f) being sued damages the reputation of the audit firm, which is likely to reduce the number of new audits acquired and losing some existing audits

(g) if the auditor is found to be negligent, he/she will have to pay out damages. Most of this would be covered by Professional Indemnity Insurance (PII insurance). If there are many claims, the PII insurance company is likely to increase the insurance premiums

Page 326: elliottv10

Barry Elliott and Jamie Elliott: Financial Accounting and Reporting (tenth edition) – Instructor’s Manual

326 © Pearson Education Limited 2006

Chapter 31: Question 4 – Audit Firms and Consultancy

(a) An audit firm can provide significant help to the audit client through consultancy work. The audit firm will know the audit client from the audit, so this will reduce the learning time when carrying out consultancy work, thus reducing its cost. Also, audit firms have knowl-edge in specialised areas, which would benefit the audit client. Audit firms have been developing financial and related business services as part of their consultancy business, so there are many areas where the audit firm can help the audit client. The audit firms have specialised skills in information and computer systems, and E-commerce and Internet appli-cations. Also, audit firms have always had specialists who deal with company and personal taxation, and accountants’ skills in this area are probably better than those of any other business.

So, it can be seen that auditors can provide high quality services to audit clients, often at a lower cost than could be provided by other consultants.

(b) The provision of consultancy services creates independence problems for the audit firm:

(i) The auditor may be reporting on his/her own work, such as when reporting on financial statements prepared by the auditor, or accounting systems which have been recom-mended by the audit firm. If the auditor both prepares and audits the financial statements, the quality of the audit will be less than if these two functions were under-taken by different people. This is because one is poor at checking the accuracy of one’s own work. An independent person is much better at detecting errors in another person’s work. Also, if the auditor finds errors in work carried out by the firm, he/she will be reluctant to highlight these errors, as they could be a sign to the audit client of the low quality of the audit firm’s work. So, the audit firm may give an unqualified audit report when the audit report should have been qualified (modified) because of material errors in the financial statements.

(ii) the second problem is that with the higher fee from the combined audit and consultancy work, the auditor will be reluctant to qualify the audit report, as this could result in a loss of both the statutory audit and the consultancy work. The profitability of the non-audit work for the FTSE 100 listed companies is six times the profits from audit work.

(iii) The ethical rules of most of the accounting bodies, the IFAC and the US SEC say the auditor should avoid making management decisions when performing consultancy work. However, there are problems in defining the situations when the auditor would be making management decisions, and it is difficult for third parties and regulators to detect whether audit firms are carrying out management decisions for the client com-pany. The easiest solution to this problem is to prohibit auditors from carrying out consultancy work for audit clients.

(c) Audit firms want to continue to perform consultancy work because of the high profits from this work. If an audit firm has acquired a new audit, it is both a regular annual income stream from the audit, and, being auditor, the firm has a greater chance of being selected for consultancy work than competitor consultants. This is because the audit firm’s consultants will be able to avoid some of the learning costs and the audit client will know the audit firm (and probably have a good relationship with the audit firm) so they will feel more confident in awarding the consultancy work to the audit firm than to a consultant who they have no experience of.

Page 327: elliottv10

Barry Elliott and Jamie Elliott: Financial Accounting and Reporting (tenth edition) – Instructor’s Manual

327 © Pearson Education Limited 2006

Chapter 31: Question 5 – Auditor Accountability

(a) Where the auditor reports to shareholders, the work the auditor carries out is determined by the information required by the shareholders. In a statutory audit (i.e. one governed by the country’s legislation) the work the auditor needs to carry out is determined by the informa-tion the country’s law requires the auditor to report on. In addition, it is common for the statutes to give the auditor the right of access to the company’s accounting records and to obtain explanations from the company’s staff, including the directors. In this situation, it is the shareholders or the country’s statutes which determine the work the auditor is required to carry out. In this situation, the directors cannot limit the work the auditor is required to carry out. This is quite different from situation (b).

(b) Where the auditor is providing consultancy services for the client company, the directors specify the work the audit firm is required to carry out. Thus, the audit firm’s responsibility is to the directors of the company. For consultancy work, the directors can prevent the audit firm from looking at parts of the company’s business. This limitation of the auditor’s work is not allowed for the statutory audit. So, for consultancy work, the auditor’s work is con-trolled by the directors. For the audit, the auditor’s work is determined by statute (or the shareholders) and it cannot be limited by the directors.

If the directors tried to limit the auditor’s work in carrying out an audit, the auditor would probably give a qualified (modified) report on the financial statements, stating the way the directors have restricted the auditor’s work and its possible effects on the financial state-ments.

Page 328: elliottv10

Barry Elliott and Jamie Elliott: Financial Accounting and Reporting (tenth edition) – Instructor’s Manual

328 © Pearson Education Limited 2006

Chapter 31: Question 6 – Auditors as Shareholders

There is a concern that auditors should be impartial and also be seen to be impartial in carrying out their duties as auditors. This means that they should have no personal pressure to influence the reported income of the business. Whilst it is of course possible for an independent auditor to ignore the personal implication of the shareholding and to act professionally and objectively, the public might well take a critical view. Consequently the professional bodies prohibit an auditor from holding shares in a client company.

Page 329: elliottv10

Barry Elliott and Jamie Elliott: Financial Accounting and Reporting (tenth edition) – Instructor’s Manual

329 © Pearson Education Limited 2006

C H AP T E R 3 2

Chapter 32: Question 1 – Plus Factors Group plc

(a) Value added statement for year ended 30 September 20X9

20X9 20X8

£000 % £000 %

Turnover 8,613.6 7,560.1

Bought-in materials and services 4,815.4 4,096.4

Value added by group 3,798.2 3,463.7

Share of profits of Associate 10.9 - 10.7 -

3,809.1 100.0 3,474.4 100.0

Applied the following ways:

To employees 2,193.5 57.6 2,153.6 62.0

To providers of capital 735.7 19.3 566.5 16.3

To government 464.7 12.2 527.9 15.2

For asset maintenance and expansion 415.2 10.9 226.4 6.5

3,809.1 100.0 3,474.4 100.0

(b)

20X9 20X8

£ £

Sales per employee 43,974 37,612

Value added per employee 20,379 17,232

Average remuneration per employee 11,201 10,714

(c) Problems arise because there is no standard defining the terms e.g. turnover gross or net of VAT, treatment of minority interests, should VAT appear in the government section?

Page 330: elliottv10

Barry Elliott and Jamie Elliott: Financial Accounting and Reporting (tenth edition) – Instructor’s Manual

330 © Pearson Education Limited 2006

Working

20X9 20X8

£000 £000

Turnover Sales including VAT 9,905.6 8,694.1

VAT at 15% 1,292.0 1,134.0

8,613.6 7,560.1

Bought-in materials and services Cost of materials

Creditors at end of year 1,244.2 1,109.1

Add: Payments in year 3,622.9 2,971.4

4,867.1 4,080.5

Less: Creditors at start of year 1,109.1 987.2

Materials purchased in year 3,758.0 3,093.3 Add opening stock 804.1 689.7

Less closing stock (837.8) (804.1)

Add: Bought-in services

Auditors’ remuneration 12.2 11.9

Hire charges 66.5 367.3

Other overheads 1,012.4 738.3

4,815.4 4,096.4

Employees £ £

Benefits 109.9 68.4

Pensions 319.8 222.2

Salaries and wages 1,763.8 1,863.0

2,193.5 2,153.6

Providers of capital Debenture interest [11% of £600,000] 66.0

Debenture interest [11% of £550,000] 60.5

Discount on debentures 4.0

Dividends

Preference [7% of £200,000] 14.0

Preference [7% of £500,000] 35.0

Ordinary [8m at 4.28p] 342.4

Ordinary [10m at 4.69p] 469.0

Minority interest 167.2 144.1

735.7 566.5

Page 331: elliottv10

Barry Elliott and Jamie Elliott: Financial Accounting and Reporting (tenth edition) – Instructor’s Manual

331 © Pearson Education Limited 2006

Maintenance and expansion of assets Profit before tax 1,437.4 1,156.4

Less: Taxation 464.7 527.9

Minority interest 167.2 144.1

Dividends 504.0 356.4

Retained profits 301.5 128.0

Depreciation 113.7 98.4

415.2 226.4

Page 332: elliottv10

Barry Elliott and Jamie Elliott: Financial Accounting and Reporting (tenth edition) – Instructor’s Manual

332 © Pearson Education Limited 2006

Chapter 32: Question 2 – David Mark

(a) Your proposal to close the branch is ill advised. Apart from the so-cial implications of closure referred to below, the loss accruing to your organisation based on 20X4 figures would be approximately £2,800 made up as follows:

£ £ Contributions/gross profit lost 95,700

Expenses/costs save:

Salaries and wages (all) 78,540

Rates (all) 2,865

Advertising (specified) 1,320

Delivery van expenses (all) 5,280

General expenses assuming all relate to branch 1,188

Telephone (specified) 1,056

Wrapping materials 2,640 92,889

Loss if closed 2,811

(b) Increased turnover if Peter’s suggestion is followed

To cover £125,500 expenses (including presumably the extra staff required) will need additional gross profit of (125,500 – 111,237) £14,263 thus requiring

£ Sales (14,263 × 4) of 57,052

But current expenses of £111,237 are not

covered by currently generated gross profit

because a loss of £15,537 occurs. If this is

to be absorbed then additional turnover is

necessary (15,537 × 4) 62,148

Total additional turnover 119,200

This assumes that the branch will be expected to absorb existing fixed charges i.e. salary of D. Mark £10,560, advertising £1,320 and telephone of (1,584 – 1,056) £528 and, if demanded, the delivery charge attributable to Arton of £5,280.

[Total costs estimated of £125,500 have presumably allowed for additional wages and the van charge; or additional wages, having deducted the van charge. One way or the other the wages figure will have compensated for the van be it a plus or minus. If van is included then wage fig-ure will be incorrectly budgeted in the data of the question, because it should have been excluded.]

Page 333: elliottv10

Barry Elliott and Jamie Elliott: Financial Accounting and Reporting (tenth edition) – Instructor’s Manual

333 © Pearson Education Limited 2006

Extra part-time workers necessary per formula = 119,200 = 4

30,000

If figures are to be based on costs specific to the branch of £92,889 then the additional turnover will still be £119,200 because Peter’s expenses of £125,500 remain unadjustable for fixed ex-penses, be they included or excluded, in this solution. i.e. (125,500 – 92,889) × 4 = 130,444

But the 92,889 already includes a contribution

of 2,811 via sales (×4) of 11,244

So extra turnover is: 119,200

Or required turnover of 25% gross profit content

to generate absorption of Peter’s estimated costs

of £125,500 = × 4 = £502,000

Current level per accounts = £382,800

Additional turnover = £119,200

(c) Comments on social implications of closure

1. Loss of a local shopping amenity in village. 2. Inconvenience to local residents travelling to nearest supermarket. 3. Loss of employment for 8 people and loss of the benefit of their disposable income if they

are local residents. 4. Impact on family life with parents having to work.

Comments on social implications of Peter’s recommendation. This would avoid the prob-lems referred to above.

Page 334: elliottv10

Barry Elliott and Jamie Elliott: Financial Accounting and Reporting (tenth edition) – Instructor’s Manual

334 © Pearson Education Limited 2006

Chapter 32: Question 3 – Hythe plc

(a) Value added statement – year ended 31 December 20X6

20X6 20X5

£000 % £000 %

Turnover 5,124 4,604

Bought-in materials and services (W1) 3,275 2,770 ___

Value added 1,849 100 1,834 100

Applied in the following way:

To pay employees

Wages and salaries (W2) 810 43.8 796 43.4

To pay providers of capital

Interest on loans 168 9.1 151 8.2

Preference shareholders’ dividend 24 1.3 24 1.3

Equity shareholders’ dividend 288 15.6 256 14.0

480 431

To pay government

Corporation tax 402 21.7 393 21.4

To provide for maintenance and expansion

of assets

Depreciation 155 8.4 144 7.9

Retained profits 2 0.1 70 3.8

157 214

£1,849 100.0 £1,834 100.0

Value added per employee 46,225 43,667

Sales per employee 128,100 109,619

Average earnings per employee 20,250 18,952

Page 335: elliottv10

Barry Elliott and Jamie Elliott: Financial Accounting and Reporting (tenth edition) – Instructor’s Manual

335 © Pearson Education Limited 2006

20X620X5

Workings £000 £000

(1) Bought-in materials and services

Materials consumed 2,934 2,482

Fuel consumed 290 242

Hire of plant and machinery 41 38

Auditors’ remuneration 10 8

3,275 2,770

(2) Wages and salaries

Wages 607 598

Salaries 203 198

810 796

(b) A value added statement is a measure of the wealth created by a business. It is the amount of value added by manufacturing, distribution and other businesses to the cost of raw materials, products and services purchased. It shows the total wealth created and how it was distributed, taking into account the amounts retained and reinvested in the group for the replacement of as-sets and development of operations.

Financial statements have been regarded as primarily intended for equity investors whose inter-est has been focused on profitability, capacity to adapt and solvency. The value added statement has perhaps been seen as of more interest to staff who have had little recognition by standard setters. Even in 2004 when there is a growing interest in social, environmental and ethical issues there is no financial reporting standard relating to human asset accounting in the balance sheet or value added statements.

There is a further argument that the data already appears within the existing primary reporting statements and that there is consequently little point in producing yet another statement.

Page 336: elliottv10

Barry Elliott and Jamie Elliott: Financial Accounting and Reporting (tenth edition) – Instructor’s Manual

336 © Pearson Education Limited 2006

Chapter 32: Question 4 – Gettry Doffit

1 Quantities of chemicals received by the company for disposal on site represent a liability for costs of disposal at the year-end. The work would be undertaken by Gettry Doffit plc on a con-tractual basis and, clearly, income from contracts (short-term as defined in the original SSAP 9) should not be credited to profit and loss account until the work has been completed i.e. on the completion of the contract. Therefore, it would appear that such quantities should be carried in the balance sheet as a liability at the higher of:

(i) invoice cost to the customer or

(ii) estimated cost of disposal.

Applying these principles to (A) axylotl peroxide and (B) pterodactyl chlorate:

Re (A) This contract will give rise to certain revenue of £87,179 i.e. 170 million won @ 1,950 to the £. This is because the invoice value in won has been ‘sold forward’ at the stated rate of the forward contract. It is therefore appropriate, and permissible per SSAP 20, to use the forward rate as the transaction value in the books at all dates, and given such treatment, no exchange differences will arise.

There should be a debtor and creditor for this amount in the balance sheet i.e. the debtor for the certain amount receivable should not be dealt with as income until the contract is completed. Any profit arising would be dealt with in the year to 31 March 20X6.

Dr Debtors £87,179

Cr Creditors – accruals and deferred income £87,179

It is possible that the company could choose as a matter of accounting policy to use rates on the date of the transaction and then retranslate on settlement/balance sheet date giving rise to ex-change differences. The alternative numbers arising are dealt with below. The costs incurred up to the year-end will be dealt with as follows

Dr Creditors £60,000

Cr Bank £60,000

The creditor balance would be debited with the estimated further costs to completion of £15,000 in 20X5/X6 leaving the company with profit of £12,179 in 20X5/X6.

Also in 20X5/X6 – 1.5.X5 – the company would receive 170 million won and realise, per the terms of the forward contract, £87,179, thus eliminating the debtor. Had actual rates been used:

Dr Debtors 170 million won @ 1,900 = £89,473

Cr Creditors £89,473

The balance on creditors in 20X5/X6 will then be a profit of £14,473. However, the debtor would have to be retranslated at the 20X5 year-end – 170 million won @ 2,000 to the £ = £85,000 giving rise to a loss in that year of £4,473. On settlement the debtor will realise £87,179 giving rise to a gain of £2,179.

Page 337: elliottv10

Barry Elliott and Jamie Elliott: Financial Accounting and Reporting (tenth edition) – Instructor’s Manual

337 © Pearson Education Limited 2006

In total, £14,473 plus £2,179 less £4,473 = £12,179 (as when the forward rate was used to start with) would be credited to profit and loss account, though in this case partly in 20X4/X5 and partly in 20X5/X6.

Re (B) As this is disposed of per the terms of this contract, neither a debtor nor a liability arises. The point where revenue should be recognised is the date of processing, and it is clear per the terms of the contract that no loss can arise. The costs of the break-down should therefore be carried forward as work-in-progress, perhaps reduced for the worth of the by-products.

2 The won forward contract has been exhaustively dealt with above. As the contract to buy dollars is to be used to finance trade purchases overseas, the transaction poses no problems pro-vided the dollars will be used to purchase stocks whose realisable amount is greater than (70,000 @ 1.60) = £43,750. Indeed, it would make sense not to reflect such a contract in the accounts, it being more appropriate to disclose the detail under commitments. There are, how-ever, other pertinent points to be made.

If the dollars are not to be applied towards a trade purchase, the company would have surplus dollars which may only be converted back to sterling at a loss. Such a loss should be recognised in accordance with the prudence concept, although there may be mitigating factors such as an alternative use for dollars.

3 Given the raising of the irrevocable letter of credit, all that the Nigerian supplier has to do is to ship the goods specified in the letter, present the bill of lading as proof of shipment, and await payment. Thus, the company must pay for goods supplied in accordance with the contract terms, and cannot cancel. Therefore, a liability of (130 – 90)/130 × £65,000 = £20,000 should be recog-nised immediately, unless a variation can be negotiated with the supplier or an alternative use found for the chemical.

4 The spillage is a post balance sheet event. No liability should be recognised in the accounts unless the going concern concept is threatened.

However, the potential liability is so material as to require disclosure under SSAP 17.

• In a normal joint venture the companies trade as partners, with joint and several liability.

• The precise apportionment of the liability may require a contribution from Dumpet Andrunn plc.

• If they cannot pay, it is likely that Gettry Doffit plc will have to.

• The likelihood of a liability crystallising, the likely amount, and any recovery from Dumpet Andrunn plc, must be assessed, and full details given in the notes to the accounts and re-ferred to in the directors’ report.

• As it is likely that the company will resist the claim, the maximum payable should probably be disclosed as a contingent liability.

• The possibility of an insurance recovery should also be examined.